Sie sind auf Seite 1von 114

APPART - Academy of Proficient Professionals for Aptitude Research and Training

THEORY BOOKLET

An Orientation

This booklet contains all the concepts of Maths, Logical Reasoning which are important from point of view of any recruitment test. 
 
All the major concepts are discussed along with solved examples. 
 
A sincere student must go through this booklet 2-3 times to ensure the clarity of the topic. 
 
In case of any query, please write us at info@apparteducation.com . 
 

ALL THE BEST!

www.apparteducation.com 1 info@apparteducation.com
APPART - Academy of Proficient Professionals for Aptitude Research and Training

INDEX/CONTENTS
TOPICS PAGE NO.
MATHS
NUMBER SYSTEM ..................................................................................................................................................................... 3 
PERCENTAGE .......................................................................................................................................................................... 16 
PROFIT & LOSS & PARTNERSHIPS ......................................................................................................................................... 17 
SIMPLE & COMPOUND INTEREST ......................................................................................................................................... 18 
RATIO AND PROPORTION ...................................................................................................................................................... 26 
AVERAGES AND MIXTURES .................................................................................................................................................... 32 
MIXTURE (ALLIGATION) ......................................................................................................................................................... 33 
TIME SPEED AND DISTANCE .................................................................................................................................................. 39 
TIME AND WORK .................................................................................................................................................................... 48 
PERMUTATIONS & COMBINATION ....................................................................................................................................... 54 
PROBABILITY ........................................................................................................................................................................... 61 
ANSWER KEY ........................................................................................................................................................................... 67 

LOGICAL REASONING
CODING – DECODING............................................................................................................................................................. 69 
NUMBER SERIES ..................................................................................................................................................................... 70 
ALPHABETIC SERIES ................................................................................................................................................................ 72 
BLOOD RELATIONS ................................................................................................................................................................. 73 
DIRECTIONS ............................................................................................................................................................................ 75 
CUBES ...................................................................................................................................................................................... 77 
SET THEORY ............................................................................................................................................................................ 80 
SYLLOGISM.............................................................................................................................................................................. 83 
DATA INTERPRETATION ......................................................................................................................................................... 86 
DATA SUFFICENCY .................................................................................................................................................................. 93 
MATCHING, SELECTION, ARRANGEMENT ............................................................................................................................ 96 
VISUAL REASONING .............................................................................................................................................................102 
CLOCKS ..................................................................................................................................................................................104 
CALENDARS ...........................................................................................................................................................................106 
INPUT & OUTPUT FLOW CHART ..........................................................................................................................................108 
PUZZLES ................................................................................................................................................................................110 
ANSWER KEY .........................................................................................................................................................................112 

www.apparteducation.com 2 info@apparteducation.com
APPART - Academy of Proficient Professionals for Aptitude Research and Training

MATHS

An Orientation

 This section of booklet contains all the concept of Maths which is relevant from the campus recruitment test point
of view along with some of the competitive exams (GRE/GMAT) point of view.
 All the major topics are covered & their concepts are discussed along with solved examples & explanations
 In case of any query, feel free to write us at info@apparteducation.com

www.apparteducation.com 3 info@apparteducation.com
APPART - Academy of Proficient Professionals for Aptitude Research and Training

NUMBER SYSTEM
Introduction:
This chapter is designed to give you a quick recap of what you had learnt during the school. A quick brush-up of the concepts shall 
help you in identifying and solving the problems at a greater pace.
Real Numbers: Numbers  which  can  be  commonly  seen  and  identified  and  can  be  represented  on  a  number  line.  e.g.: 
 10, 2.55, 0,1, 7  
Number Line: It is a line on which all the positive and negative numbers can be marked in a sequence. 
Imaginary numbers: Those numbers that cannot be represented on a number line are imaginary numbers. 

e.g.:   1 ,  5 , etc.   1 is represented by i.    

p
Rational Numbers: All numbers that can be expressed in  form, where p, q are integers and  q  0 . 
q
5 2
e.g.:  ,1  
7 3
p
Irrational Numbers: Those numbers that cannot be expressed in  form. 
q
e.g.:   , 2 , 3  1  
p
Fractions: All rational numbers which are in form, where p, q are integers and p is not a multiple of q. 
q
p is called numerator whereas q is known as denominator. 
•  Fractions are of the following types 
2 3
•  Proper :    p < q     e.g.,  , etc. 
7 8
6 5
•  Improper :    p    q     e.g.,  , etc. 
5 2
1 1
•  Mixed :  It is an integer plus a, fraction  e.g.,  3 , 7 etc. 
5 3
Integers :All the rational numbers that do not have any decimal or fractional part. 
 , ...,  3 ,  2 ,  1, 0 ,1,  2 ,  3 , ...,  
 
Whole Numbers :All non negative integers are whole numbers. W = {0, 1, 2, 3, ...}. 
Natural Numbers :Whole numbers, except zero, are called natural numbers. N = {1, 2, 3, 4, ...}. 
Odd Numbers : All natural numbers which are not divisible by 2 are odd numbers. 
    Such numbers are expressed as 2k ± 1 (k is any natural number). e.g.:1, 3, 5, 7, ... 
Even Numbers: All  natural numbers  that are divisible by 2 are  called  even numbers. Such numbers are  expressed as 2k  (k  is  any 
natural number). e.g.: 2, 6, 8, ... 
The box given below exhibits the types of numbers obtained while carrying out arithmetic operations between two type of numbers.  
 
odd  ± odd = even    odd ±even = odd    even ± even = even 
 
odd   odd = odd    odd    even = even    even    even = even 
 

www.apparteducation.com 4 info@apparteducation.com
APPART - Academy of Proficient Professionals for Aptitude Research and Training

Prime Numbers :All the natural numbers that are greater than 1, and are only divisible by 1 or the number itself, are called prime 


numbers. 
The box given below depicts some of the characteristics of the prime numbers. 

 There are 25 prime numbers upto 100 2, 3, 5, 7,11, 13,17,19, 23, 29, 31, 37, 41, 43, 47, 53, 59, 61, 67, 71, 73, 79, 83, 89, 97. 
 2 is the only even prime number. 
 97 is the only prime number from 90 to 100. 
 
Co-primes: Two numbers 'a' and 'b' are said to be co-prime if they don't have any common factor other than 1. e.g.: (3, 5), (7, 12), 
etc. 
Composite Numbers: Numbers greater than 1 that are not prime are called composite numbers. 
e.g.4, 6, 8, 9, .... 
1 is neither a prime number nor a composite number
Reason : A prime number has two factors, 1 and the number itself; whereas a composite  number has more 
than two factors. Since, 1 has only one factor i.e., 1, hence it is neither a prime nor a composite numbers. 
Table 1: Classification of numbers 
  No.  Real  Imaginary  Rational  Irrational  Even  Odd  Prime  Composite  Whole  Natural  Integer  Fraction 
  3  Y  N  Y  N  N  Y  Y  N  Y  Y  Y  N 

3   Y  N  N  Y  N  N  N  N  N  N  N  N 

7
  Y  N  Y  N  N  N  N  N  N  N  N  Y 
2
I  N  Y  N  N  N  N  N  N  N  N  N  N 

Perfect Numbers :A number ‘a’ is said to be perfect if the sum of its factors (excluding itself but including 1) is equal to `a'. 


e.g.: 6, 28, etc. 
The perfect number 6 has 1, 2 and 3 as its factors, which sum up to 6. 
Similarly, the perfect number 28 has 1, 2, 4, 7 and 14 as it's factors, which sum up to 28. 
Perfect square : 
A number is said to be a perfect square if and only if the square root of that number is an integer.   
Some important facts about perfect squares:  
(1) The square of an even number is always even.   
(2) The square of an odd number is always odd.   
(3) Square of an integer cannot end in 2, 3, 7 or 8.  
(4) The square of a real number (negative or positive) is always positive. 

www.apparteducation.com 5 info@apparteducation.com
APPART - Academy of Proficient Professionals for Aptitude Research and Training

Conversion of recurring decimal into fractions : 


p
What is the   form of 0.5555... (also represented as 0. 5 )?  
q
Let,  x  0.55555... 
   10x  5.55555 

   9 x  10 x  x  ( 5.5555...)  (0.55555...)  5  
5
 x  
9
If  x  0.232323...  
   100x  23.232323  

   99 x  100x  x  (23.232323...)  (0.232323...)  


 = 23 
For a purely recurring number (all digits after decimal point recur) we can identify the procedure as:  
p
The  form of a purely recurring number  
q
The recurring part written once

As many 9's as the number of digits in the recurring part  
In the number like 0.14333... i.e.  0.14 3  
Let x = 0.143333.. .  
    100x = 14.3333...  
    1000x = 143.3333... 
    900x = 1000x – 100x 
= (143.3333...) - (14.3333...) = 129  
129
 x
900  
Thus for any recurring number we can identify the procedure as: 
p
The  form of any recurring number  
q
(The non - recurring and recurring part written once) 
(the non - recurring part)
  
As many 9' s as the number of digits in the recurring part
followed by as many 0' s as digits in non - recurring part
Example 1: Express  0.643 as a fraction. 

Solution : Let  x  0.643 , then 1000  x  643.643

     1000x  x  643.643  0.643  

        999x = 643 
643
     x  
999

www.apparteducation.com 6 info@apparteducation.com
APPART - Academy of Proficient Professionals for Aptitude Research and Training

Example 2: Express  6.43  as a fraction. 

Solution : Let  x  6.43 , then 

    100 x  643.43  

     100 x  x  643.43  6.43  

     99x  637  
637
     x  
99
Example 3: What least number must be subtracted from 2000 to get a number which is exactly divisible by 17? 
Solution: On dividing 2000 by 17, we get 11 as remainder. 
       Required number to be subtracted = 11. 
Example 4: What least number must be added to 3000 to obtain a number exactly divisible by 19? 
Solution:On dividing 3000 by 19, we get 17 as remainder.  

       Number to be added = (19 – 17) = 2. 
Example 5: Find the number which is nearest to 3105 and exactly divisible by 21? 
Solution:On dividing 3105 by 21, we get 18 as remainder. 

       Number to be added to 3105 is (21 – 18) = 3. 
       3108 is the required number. 
Example 6: A number  when  divided  by 342 gives  a  remainder  47.  When  the  same  number  is  divided by  19,  what would  be  the 
remainder? 
Solution:On dividing the given number by 342, let k be the quotient and 47 the remainder.  
    Then, number = 342k + 47 
    =[(19x 18k)+(19x2+9)]=[19(18k+2)+9] 
    The given number when divided by 19 gives (18k + 2) as quotient and 9 as remainder. 
Alternate method:
342  is  a  multiple  of  19,  divide  the  remainder  by  the  second  dividend  to  get  the  remainder.  47  when  divided  by  19  gives  9  as 
remainder. 
How to find whether a number is prime or not?
For small numbers, we could find by checking, if that number is divisible by any other prime number till that number itself. 
But for the larger numbers like, say 631, there is an alternate method. 
Step 1: Find the approximate square root of the given number, i.e. 25. 
Step 2: Check if any prime number from 2 to 25 divides 631. 
The prime numbers from 2 to 25 are 2, 3, 5, 7, 11, 13, 17, 19 and 23. Since none of these numbers divide 631 exactly, 631 must be a 
prime number. 
Factorial: The continued product of first n natural number is called 'n factorial' and is denoted by n! or n.  
n !  1  2  3  ...  (n – 1)   n 
e.g.:  6 !  1  2  3  4  5  6  720  

By definition 0 ! = 1. 

www.apparteducation.com 7 info@apparteducation.com
APPART - Academy of Proficient Professionals for Aptitude Research and Training

Number of factors
N = (p + 1) x (q + 1) x (r + 1)   
where N is the number and p, q, and r are the prime factors  
Factor: Factors of a number are those numbers which when divide the original number, leaves no remainder. When talking about 
the factor we consider only the positive integral factor. 
Highest Common Factor (HCF) and Lowest Common Multiple (LCM):
HCF and LCM are one of the basic concepts of mathematics which is having a variety of applications in our daily life. 
For example, 
Factors of 20 = 20, 10, 5, 2, 1 
Factors of 100 = 100, 50, 25, 20, 10, 5, 2, 1 
Factors of a number are always countable.  
The numbers which divide the given number (say, N) completely, are called as factors of N. For example 1, 2, 3 and 6 are factors of 6. 
If N = a p × b q × cr… 
Where, a, b, c… are prime numbers, 
Multiple: Multiples of a number are those numbers which when divided by the number leaves no remainder: When talking about 
the multiples we consider the positive integral multiples. 
For example,
Multiples of 20 = 20, 40, 60, 80, etc. Multiples of 100 = 100, 200, 300, 400, etc. 
Understanding HCF:
Let us take two numbers 15 and 20 Factors of 15 are = 15, 5, 3, 1   
Factors of 20 are = 20, 10, 5, 1  ; 
Understanding LCM:
Let us take two numbers 15 and 20. 
Multiples of 15 = 15, 30, 45, 60, 75, 90, 105, 120,135, etc. 
Multiples of 20 = 20, 40, 60, 80, 100, 120, 140, etc. 
To find the LCM of these two numbers, check which is the lowest number common multiple to both the numbers. 
How to find HCF of two numbers?
There are two methods: 
a. Division method 
b. Prime factorisation method. 
 
a. Division method:
In this method divisor becomes dividend and remainder becomes divisor and this process continues till on, can divide. The last 
divisor is your answer.
For example –

www.apparteducation.com 8 info@apparteducation.com
APPART - Academy of Proficient Professionals for Aptitude Research and Training

Example 7: To find the HCF of 15 and 20.  Example 8: To find the HCF of 20 and 28. 

15 20 (1   20 28 (1  
–15        – 20          . 
  5) 15 (3          8) 20  (3 
–15            –16        . 
     0            4)  8  (2 
So, HCF of 15 and 20 is 5.               –8  
                 0. 
So, the HCF of 20 and 28 is 4. 
Example 9: To find the HCF of 20, 28 and 45 
    We have seen that HCF of 20 and 28 is 4.  
    So, we will take HCF of 4 and 45.  

4 45(11  
   –44          
      1) 4 (4 
        –4 
           0  
So, HCF of 20, 28 and 45 is 1. 
Note: The HCF of an odd number and an even number is always 1. 
b. Prime factorization method:
Write the number in terms of prime factors. 

20  2 2  5 1  
45  2 0  3 2  5 1  
For finding out their HCF, take the lowest power of all prime numbers.  

The HCF of 20 and 45 is  2 0  3 0  5 1 i.e. 5. 
How to find LCM of two or more numbers?
There are two methods 
i.  Division method 
ii. Prime factorisation method 
i. Division method: 
LCM of 18, 27 and 30. 
3  18,  27,  30 
3  6,  9,  10 
  2,  3,  5 
LCM   3  3  3  2  5  270  
 

www.apparteducation.com 9 info@apparteducation.com
APPART - Academy of Proficient Professionals for Aptitude Research and Training

ii. Prime factorisation method:


Take two numbers 20 and 45. 
Write the numbers in terms of prime factors. 

20  2 2  5 1  
45  2 0  3 2  5 1  
For finding out their LCM, take the highest power of all prime numbers.  

The LCM of 20 and 45 is  2 2  3 2  5 1 i.e. 180. 
Example 10: Find the HCF of 24 and 72. 
Solution: 24  2  2  2  3  

    72  2  2  2  3  3  
    HCF   2  2  2  3  24  
  Similarly, you can find the HCF of sets containing more than 2 numbers. 
Example 11: Find the largest number that can exactly divide 513, 783 and 1107. 
Solution:Required number = HCF of 513, 783 and 1107.  

    Now,  513  3 3  19 , 783  3 3  29 , 1107  3 3  41  

       HCF   3 3  27  
Hence, the required number is 27. 
Example 12: Find the least number which when divided by 6, 7, 8, 9 and 12 leaves the same remainder 1 in each case. 
Solution:Required number = (LCM of 6, 7, 8, 9, 12) + 1  

       LCM  3  2  2  7  2  3  504  
Hence, required number = (504 + 1) = 505 
Example 13: How many three-digit numbers are divisible by 6? 
Solution:There are 16 numbers before 100 which are divisible by 6. 
    There are 166 numbers before 999 which are divisible by 6. 
    Total three-digit numbers divisible by 6 are 166 -16 =150. 
Important results:
If 2 numbers a and b are given, and their LCM and HCF are L and H respectively,  
then L    H = a    b. 
LCM and HCF of fractions:
LCM of numerators HCF of numerators
LCM of fractions    HCF of fractions    
HCF of denominators LCM of denominators
25 35
e.g.: Find the LCM and HCF of  and  . 
12 18
LCM of 25 and 35 175 HCF of 12 and 18 5
LCM      HCF  
HCF of 12 and 18 6 LCM of 12 and 18 36  
Note : Do not directly apply the formula if the fraction are not in their simplest form. 

www.apparteducation.com 10 info@apparteducation.com
APPART - Academy of Proficient Professionals for Aptitude Research and Training

Example 14 :The HCF of two numbers is 11 and their LCM is 69; If one of the numbers is 77, find the other. 
11  693
Solution :The other number   99  
77
Unit's place digit of a number :
The digit at the unit's place of any number is the remainder when the number is divided by 10.  
For example, lets consider the number 364. The remainder when 364 is divided by 10 is 4. Hence ‘4’ is the unit's digit of the number 
364. 
To find the unit's digit of a number which is the product of two or more numbers, multiply the unit's digit of the numbers and find 
the units digit of the resultant number. For example, 19  64, the product of the units digit of 19 and 64 is 36 and the unit's digit of 
36 is 6, hence the unit's digit of 19  64 is 6. 
Unit's digit of higher powers of any number: 

21  2     2 2  4    23  8     2 4  16  
2 5  32   2 6  64   2 7  128   2 8  256  
2 9  512   2 10  1024   2 11  2048   2 12  4096  
We  can  see that  the unit's digit  of  2 1 , 2 5 , 2 9 is 2, units  digit of  2 2 , 2 6 , 2 10 is  4,  units digit of  2 3 , 2 7 , 2 11 is  8  and units digit of 

2 4 , 2 8 , 2 12 is 6. 
Therefore after every  four powers of 2, the units digit of the number starts repeating. Thus we say that cyclicity of unit's digit of 
higher powers of 2 is 4. 
Similarly the digits whose cyclicity is 4 are 2, 3, 7 and 8. The digits whose cyclicity is 2 are 4 and 9.  
Any power of numbers whose unit's digit 1, 5 or 6 always ends in 1, 5 and 6 respectively. 

For example,  11 2  121, 25 2  625 and 16 2  256 . 

Some important formulae used in simplification:

(1)  ( a  b ) 2  a 2  b 2  2 ab  

(2)  ( a  b ) 2  a 2  b 2  2 ab  

(3)  ( a  b ) 2  ( a  b ) 2  4 ab  

(4)  a 2  b 2  ( a  b ) ( a  b )  

(5)  a 3  b 3  ( a  b ) ( a 2  ab  b 2 )  

(6)  a 3  b 3  ( a  b ) ( a 2  ab  b 2 )  
3 3 3
(7)  ( a  b)  a  b  3ab(a  b)
 
3 3 3
(8)  ( a  b)  a  b  3ab( a  b)
 

www.apparteducation.com 11 info@apparteducation.com
APPART - Academy of Proficient Professionals for Aptitude Research and Training

527  527  527  183  183  183


Example 15 :Simplify   
527  527  527  183  183  183
( 527 ) 3  (183) 3
Solution:The given expression is equivalent to   
( 527 ) 2  527  183  (183) 2
a3  b3
    We know that,   
a 2  ab  b 2
    In the above example  a  527 and  b  183  

       The expression is equal to (527 + 183) = 710  

 (614  168) 2  ( 614  168) 2 


Example 16 :Simplify    
 614  168 
 
( a  b ) 2  ( a  b ) 2 4 ab
Solution: Let a = 614 and b = 168, then the expression becomes   4
ab ab
Example 17: Find the square of 1605.  

Solution: (1605) 2  ( 1600  5) 2  

     (1600) 2  2  1600  5  ( 5) 2  
    = 2560000 + 16000 + 25 = 2576025  

Example 18: Evaluate:  ( 57 ) 2  ( 43) 2  2  57  43  


2 2 2 2 2
Solution: a  b  2 ab  ( a  b )  ( 57  43)  100  10000  

2 2
Example 19: Simplify  (81)  (68)  2  81  68  

Solution: (81  68) 2  13 2  169


 
Example 20: Evaluate:  ( 313  313  287  287 )

1
Solution: a 2  b 2  [( a  b ) 2  ( a  b ) 2 ]  
2
    (where a  313  and  b   287) 
1
 [( 313  287 ) 2  ( 313  287 ) 2 ]  
  2
1

2
 
600 2  26 2  180338  
 

Rules of counting numbers :


n (n  1)
1. Sum of first n natural numbers    
2
2. Sum of first n odd numbers   n 2  

3. Sum of first n even numbers   n (n  1)  

www.apparteducation.com 12 info@apparteducation.com
APPART - Academy of Proficient Professionals for Aptitude Research and Training

n ( n  1) ( 2n  1)
4. Sum of the squares of first n natural numbers    
6
2
 n (n  1) 
5. Sum of the cubes of first n natural numbers      
 2 
5
Example 21 :If square root of 15 = 3.88, the value of square root of  is 
3
5 53 15 3.88
Solution :     1.29
3 33 3 3  
Example 22: A four-digit number divisible by 7 becomes divisible by 3, when 10 is added to it. Find the largest such number. 
Solution :Largest four-digit number is 9999. 
    On dividing 9999 by 7, we get 3 as remainder.  
    Largest four-digit number divisible by 7 is 9996.  
    Let 9996 – x + 10 be divisible by 3.  
    By trial and error, we find that x = 7  
    Required number = (9996 – 7) = 9989. 
Example 23 :A  three-digit  number  4a3  is  added  to  another  three  digit  number  984  to  give  the  four-digit number  13b7  which  is 
divisible by 11. Find the value of (a + b). 
Solution: 4 a 3  
              +  9 8 4  
                 1 3 b 7 
Here a + 8 = b, if 13b7 is divisible by 11 then (7 + 3) – (b + 1) = 0; b = 9 and a + 8 = bora = 1.  
Hence, a + b = 9 + 1 = 10 
 
Example 24: Of the three numbers, the sum of the first two is 45; the sum of the second and the third is 55; and the sum of the third 
and thrice the first is 90. Find the third number. 
Solution:Let the numbers be x, y and z. Then, x + y = 45; y + z = 55 and 3x + z = 90. 
y = 45 – x and z = 55 – y = 55 – (45 – x) = 10 + x 

       3x + 10 + x = 90 or x = 20  
y = (45 – 20) = 25 and z = (10 + 20) = 30 
       Third number = 30 
 
Advance Concepts:
No. of Zeroes:
Example - Find the total number of zeroes at the end of 100! 
100! = 1 * 2 * 3 * 4 * 5 * ………. * 10 * ………. * 20 * ………. * 30 * ………. * ………. * ………. * 100 
To find the number of zeroes, i.e. 10 = 2 * 5, you need to calculate the total number of 2s and 5s. The less of the two will be the total 
number of zeroes.  
Total 2s in 100! = 97 
Total 5s in 100! = 24 
Hence, the total number of zeroes is 24 

www.apparteducation.com 13 info@apparteducation.com
APPART - Academy of Proficient Professionals for Aptitude Research and Training

Class Exercise
Q.1  The LCM of two numbers is 5200 and their HCF is 40. If one of the numbers is 520, the other number is 
(1) 240    (2) 560    (3) 400    (4) 320 
Q.2    The sum of the squares of first ten natural numbers is 
(1) 281    (2) 385    (3) 402    (4) 502 
Q.3    The sum of first ten odd numbers is 
(1) 105    (2) 100    (3) 110    (4) 120 
3 12
Q.4    HCF of  and  is 
5 13
3 3 3 3
(1)      (2)    (3)      (4)   
65 130 5 13
Q.5    The lowest four-digit number which is exactly divisible by 2, 3, 4, 5, 6 and 7 is 
 (1) 1400    (2) 1300   (3) 1250   (4) 1260 
Q.6    Find the digit in the units place in the product  254  361  159  18  
(1) 1    (2) 6    (3) 4    (4) 8 
Q.7    The smallest number among the following is 
3
(1)  (7 )  
3
  (2)  (8.5)  
3
(3)  ( 4 )  
4
 
(4)  6 5 5  
Q.8    Find the sum of the first 50 even numbers. 
(1) 1275    (2) 2650   (3) 5100   (4) 2550 
Q.9    Find a if 7a4 is divisible by 9. 
(1) 6    (2) 5    (3) 4    (4) 7 

Q.10  Find the value of  4056 26 of  26  3 216  


(1) 6    (2) 0    (3) 26    (4) 104 
Q.11  Evaluate: 18   9 + 2.7 of 3  
(1) 101.2     (2) 27.32   (3) 10.1    (4) 11.1 
Q.12  Which one of the following is incorrect?  
(1) Square root of 5184 is 72      (2) Square root of 15625 is 125 
(3) Square root of 1444 is 38      (4) Square root of 1296 is 34 
Q.13  The least number which on division by 35 leaves the remainder 25 and on division by 45 leaves the remainder 35 and on 
division by 55 leaves the remainder 45 is 
 (1) 2515    (2) 3455   (3) 2875   (4) 2785 
Q.14  The sum of first 45 natural numbers is 
(1) 2070    (2) 1053   (3) 1280   (4) 1035 
Q.15  What least value must be assigned to * so that the number 451 *603 becomes exactly divisible by 9? 
(1) 2    (2) 7    (3) 8    (4) 5 
Q.16  What least value must be assigned to * so that the number 63576 * 2 is divisible by 8?  
  (1) 1    (2) 2    (3) 3    (4) 4 
 

www.apparteducation.com 14 info@apparteducation.com
APPART - Academy of Proficient Professionals for Aptitude Research and Training

Q.17  If the HCF of two numbers is 3 and their LCM is 24, find the two numbers. 
(1) 24, 1    (2) 3, 8    (3) 3, 24   (4) 6, 24 
Q.18  What is the least number which when divided by 6, 8 and 10 leaves remainder 4 every time? 
(1)84    (2) 124    (3) 244    (4) 484 
Q.19  What is the least number which when divided by 15, 24 and 36 leaves remainders 9, 18 and 30 respectively? 
(1) 354    (2) 366    (3) 714    (4) 184 
Q.20  Evaluate: 1399   1399 
(1) 1687401    (2) 1901541  (3) 1943211  (4) 1957201 
Q.21  Find the value of 397    397 + 104    104 + 2    397    104. 
(1) 250001    (2) 251001  (3) 260101  (4) 261001   
Q.22  What is the remainder when 27  is divided by 7 ? 
 (1) 2    (2) 4    (3) 1    (4) None of these 
1 1
Q.23  LCM of  and  is 
4 8
1 1 1
(1)      (2) 1    (3) 
    (4)   
2 4 8
3 2 7 4 4
Q.24  The greatest fraction among  , , , and  is 
7 5 13 7 9
4 4 2 7
(1)      (2)      (3)      (4) 
7 9 5 13  
Q.25 How many zeros are there at the end of product of first 15 prime numbers? 
(1) 0    (2) 1    (3) 15    (4) 5 
Q.26  What is the highest power of 5 that will divide 100!? 
(1) 100    (2) 10    (3) 24    (4) 11 
Q.27  In a city, there are three schools having 336, 210 and 294 students respectively. An examination committee has to arrange a 
sitting  arrangement  for  these  students  such  that,  each  classroom  should  be  occupied  to  its  capacity  and  students  from 
same school should sit in same classroom. What is the minimum number of classrooms needed? 
(1) 42    (2) 21    (3) 20    (4) 25 
Q.28  How many factors does 360 have? 
(1) 26    (2) 22    (3) 24    (4) 20 
Q.29  What is the unit’s digit of 372 25 × 72335? 
(1) 2    (2) 4    (3) 8    (4) 0 
Q.30  What is the unit’s digit of 3226 – 2335? 
(1) 4    (2) 7    (3) 2    (4) 5 
Q.31  If the product of three consecutive integers is 720, then their sum is 
 (1) 54    (2) 45    (3) 18    (4) 27 
Q.32  How many numbers between 200 and 600 are divisible by 4, 5 and 6 ? 
 (1) 5    (2) 6    (3) 7    (4) 8 
Q.33  What is the highest power of 8 that will divide 100!?   
 (1) 97    (2) 13    (3) 32    (4) 14 

www.apparteducation.com 15 info@apparteducation.com
APPART - Academy of Proficient Professionals for Aptitude Research and Training

PERCENTAGE
 
The word PERCENT is formed of two words, ‘PER’ which means ‘EVERY’ and ‘CENTUM’ which means ‘HUNDRED’. It is denoted by the 
sign ‘%’. 
A percentage can also be represented as a fraction or decimal. 
For e.g.: 50% is equal to ½ or 0.5
Important:
B
1) If A is increased/decreased by an amount B then the percentage increase/decrease in A is given by   100 %. 
A
A  (100  x)
2) If A is increased by x%, then the new value of A becomes . 
100
A  (100  x)
3) If A is decreased by x%, then the new value of A becomes . 
100
x
4) If A is X% more than B, then B is  % less than A. 
100  x
x
5) If A is X% less than B, then B is  % more than A. 
100  x
Example 1:
A's income is 70% of B's income. B's income is 50% of C's income, If C's income is Rs.1, 00,000, what is A’s income? 
Solution:

 50   Rs.100000 = Rs. 50,000 
B's income =   
 100 
 70 
A's income =     Rs. 50000 = Rs. 35,000 
 100 
Alternative Method:
50
B's income =   of C’s income 
100
70 70 50
A's income =   of B’s income     of C's income 
100 100 100
35
A’s income =   100000 = Rs.35, 000. 
100
Note:   10% increase on A signifies 1.1 times A. 
20% increase on A signifies 1.2 times A. 
10% decrease on A signifies 0.9 times A. 
20% decrease on A signifies 0.8 times A. 
Successive Percentage Changes
If a number is changed (increase/decrease) by a% and in the second step, this changed Number is again changes (increase/decrease) 
by b% then  
 ab
Net percent change =  a  b  % 
 100 
If a or b or both show decrease, then put a (-ve) sign before a and b, otherwise (+ve) sign will remain. 

www.apparteducation.com 16 info@apparteducation.com
APPART - Academy of Proficient Professionals for Aptitude Research and Training

Example 2:
If the price of an item is increased by 20% and then a discount of 10% is given on the increased price, what will be the effect on 
scale? 
Solution:
 a b 20  10
Using percent change =  a  b  %  20  10   8% (Increase) 
 100  100
Example 3:
The number of seats in a auditorium is increased by 25%. The price on a ticket is also increased by 12%. What is the effect on the 
revenue collected?  
Solution: 
 Let the initial number of seats be 100 and price per ticket be Re. 1 Then,  
Revenue = number of seats   price per ticket  
125
Increased number of seats =  100  125  
100
112
Increased price of a ticket    1  Rs. 1.12 
100
Increased revenue   125  1.12  Rs. 140  
 140  100 
Percentage increase in revenue      100  40%  
 100 
Short cut:
ab
Using % increase   a  b   
100
25  12
      Percentage increase in revenue   25  12   25  12  3  40%  
100
Example 4:
The length of a rectangle is increased by 10%. What will be the percentage decrease in its breadth so as to have the same area? 
Solution:
Let length and breadth of the rectangle be I and b respectively. 
Area = Ib 
Increased length and breadth = I’ and b'; Area = Ib 
110 11 11 10
I’= I I Ib'  Ib    b'  b 
100 10 10 11
10 1
Decrease in breadth = b - b'= b  b b 
11 11
b 100 1
Percentage decrease in breadth    100  9 %
11b 11 11

PROFIT& LOSS& PARTNERSHIPS


Whenever a person buys a certain article and sells it, there is a profit or a loss incurred in the whole transaction. The following basic 
terms should be known: 
Cost Price (C.P.): The price at which the article is bought. 
Marked Price (M.P.):  The  price  at  which  the  article  is  intended  to  be  sold  at  first.  It  is  also  known  as  MRP,  List  price,  Tag  price, 
Showroom price etc. 
Discount (D): It is the concession, which is offered on the marked price of the article. 
Selling Price (S.P.): The price at which the article is actually sold. 
M.P. – D. = S.P. 
www.apparteducation.com 17 info@apparteducation.com
APPART - Academy of Proficient Professionals for Aptitude Research and Training

Profit (P): When S.P. > C.P., there’s always a profit and it is equal to SP-CP. It also called gain. 
P.
Profit % =   100  
C.P.
100  P.%
S.P. =  C.P.   
100
Loss (L): When S.P. < C.P., there’s always a loss and it is equal to CP-SP. 
L.
Loss % =   100  
C.P.
100  L.%
S.P. =  C.P. 
100  
Note:
1) Profit or Loss is always calculated on Cost Price.
2) Discount is always calculated on Marked Price.
Formulae:
(S.P. CP.) Profit  S.P. 
1.  Profit percentage    100 =  100    1   100  
C.P. C.P.  C.P. 
(C.P. - S.P.) Loss  S.P. 
2.  Loss percentage     100   100   1    100  
C.P. C.P.  C.P. 
3.  If marked price is M. P and discount percentage is d, then  
M.P(100 - d) 100  S.P.
  S.P. = ; M.P   
100 (100 - d)
4.  M.P  Discount
   S.P.    C.P. 
Profit

P2
5.  If 2 items are sold, each at Rs. X, one gain of P % and the other at a loss of P% then overall loss percentage   %  and 
100
2 P2X
Loss (in rupees)  
100 2  P 2  
6.   If two items cost is same, one is sold at profit of P5 & other at a loss of P%, then overall there is neither profit nor loss. 
7.   If a vendor cheats his customer by selling the goods at CP, but using false weights instead of actual weights, then the profit 
earned is given by the formula: 

Profit % = 
 Actual weight – False weight  100  
 False weight 

SIMPLE & COMPOUND INTEREST


This is one of the two main areas in which the concept of percentages is applied. This is also a topic relevant to most people in real 
life - whether calculating bank interest or buying a car/ house on loan and calculating EMI. As the names suggest, simple interest is 
‘simple’ and compound interest is ‘complex’.  
Simple Interest
 Formula for Simple Interest (S.I) is =    × ×  
            100   
Where, P = principal or sum being borrowed.   

www.apparteducation.com 18 info@apparteducation.com
APPART - Academy of Proficient Professionals for Aptitude Research and Training

R = Rate of interest per year.  
T = Time period for which the amount is borrowed.   
Amount = Principal + Simple Interest  
(If T is not a whole number, then the period is represented as a fraction of year, i.e., 1 Month = (1/12)th of a year)  
The principal grows at a constant rate in absolute terms.  
Example 1: What shall be the interest to be paid on a principal of Rs. 14,000 borrowed at a rate of 15% per annum for a period of 3 
years and 6 months? 
P  R T
Solution: S.I.  
100
P = 14,000, R = 15 and T = 3.5 year 
(14000  15  3.5)
    So, S. I.    Rs. 7,350 
100
Example 2: At what simple rate of interest shall a sum of money double itself in 4 years?
Solution: Important point to be noted is that the amount received by the lender is double the amount given, which means 
Interest = Principal 
    So, if x is the Principal, then x is the Simple Interest. 
( x  R  4) 100
    Or,  x   Or,  R   25%  
100 4
Compound Interest: While computing  compound  interest  the amount  received at  the  end of 1st year becomes  principal for  2nd 
year, and so on. The principal grows at an increasing rate in absolute terms. The interest is calculated on the new principal at the end 
of every time period. Here for each time period principal keeps changing. The amount (A) for the previous time period becomes the 
principal (P) for the next time period. Formula for Compound Interest: 
n
 r 
(C.I.)   P  1   P 
 100 
n
 r 
Where, P = Principal; r = Rate of Interest; n = Time period and Amount   P  1    
 100 
Non annual compounding
  Compounding done in a  Interest added toprincipal after 
 
year  every 
Annually  1  1 year 
Semi – Annually  2  6 months 
Quarterly  4  3 months 
Monthly  12  1 month 
Note :If the word interest is given and nothing else is specified, the interest is considered as S.I.
If the interest is given by bank and nothing is specified, it is always C. I. 
Population growth is always taken on compounding basis. 

www.apparteducation.com 19 info@apparteducation.com
APPART - Academy of Proficient Professionals for Aptitude Research and Training

Example 3: What shall be the amount for a sum of Rs.1,000 at 10% for 3 years compounded annually?
Solution: Amount at the end of year 1 is
(P  R  T ) 1000  10  1
A1   P1   1000  Rs. 1,100 
    100   100
    This shall be the principal for year 2. 
(1100  10  1)
A2   1100  Rs. 1,210 
    100
(1210  10  1)
A3   1210  Rs. 1,331 
    100
So amount at the end of 3 years in case of Compound Interest (C.I.) is Rs. 1,331, while in the case of S. I., it shall be Rs. 1,300 (at 
10%). 
Alternative Method: Amount can also be calculated directly by using the formula.
n 3
 R   10 
      Amount   P  1    1000  1    Rs. 1,331 
 100   100 
      Where, P = Principal or sum being borrowed 
R = Rate of interest 
n = Time period for which the amount is borrowed 
Example 4: Find the C.I. on Rs. 5,000 at 8% p.a. for 2 years, compounded annually.
Solution: P = Rs. 5000, R = 8% and N = 2 years
2
 8 
5000  1    Amount = Rs. 5832. 
     100 
    C. I. = Amount –Principal = Rs. (5832 – 5000) = RS. 832 
Alternative method: The Compound Interest on the given sum is nothing but two successive increment of
8 8
      8%, i.e.  8  8 
 16.64%  
100
 xy 
using x  y  100 formula  
       
16.64
  Hence, Compound Interest    5000  Rs. 832 
100
Types of questions  Examples  Approach to the question 
Given Interest amount for n years at rate  A  certain  sum  earns  a  simple  interest  of  I
P  100  
r%. What is the principal?  Rs.250  in  4  years  at  5%  p.a.  Find  the  R T
250
  principal    100  Rs. 1250 
4 5
Which  option  would  lead  to  a  higher  Mr. Sharma wants to choose a investment  Compare 
1.     15 5  
amount?  plan 
    75% of interest on  
rs % simple interest for ts time   15% for 5 years (SI)         investment  
ri % compund interest for tc time  20% for 3 years (CI)  2.     (1.20) 3  1  1.728  1  

Which plan will result in higher amount?      72.8% of interest on  
        investment 
Thus option 1 will result in higher amount. 
The C.I. in nth year is Rs. X and C.I in (n +  C.I. earned in 7th year is Rs.600 and in 8th  YX
R  100  
X

www.apparteducation.com 20 info@apparteducation.com
APPART - Academy of Proficient Professionals for Aptitude Research and Training

1)th  year  is  Rs.  Y.  What  is  the  rate  of  year is Rs.660. Find the rate of interest  60
  100  10%  
interest?  600
The  total S.I  in  first  two years  is  Rs.X and  Total  S.I  for  first  two  years  is  Rs.  600.  If  Y X
R  100  
total C.I in first two years is Rs.Y. What is  same  amount  was  kept  at  C.I  at  same  X /2
the  principal  and  rate  of  interest,  if  they  rate,  total  C.I  would  have  been  Rs.660.  60
  100  20%  
are same for S.I and C.I?  What  is  the  principal  and  the  rate  of  300
interest?  Since interest for first years 
SI 600
   300 and if  
2 2
Principal is P, then  
20
P  1  300  
100
or P = 1500 
A principal amounts to X times in T years  Amount  becomes  3  times  in  5  years.  In  Y 1
Years    T  
at S.I. In how many years will it become Y  how  many  years  will  it  become  9  times?  X 1
times?  (Assume S.I.)    91
   5  20 years 
 31
Same as above but with C.I  Same as above but assume C.I.  Years  T  n where n is given 
n
by  X  Y  
Years   5  2  10  years  
Example 5: Find amount for Rs.80,000 at 20% per annum, compounded semi-annually for 2 years?
Solution : Here  n  (2 years)    2 = 4 years
20
    Similarly,  R   10% per time period 
2
    (As interest compounded semi-annually) 
    P = 80000 
    A = 80000 
4
 10 
1    80000  1.4641  Rs. 117128 
     100 
Example 6: Find C.I. on Rs. 10,000 at 10% for 9 months compounded quarterly
Solution : n = 3 periods, R = 2.5% per period and P = Rs. 10,000
3
 2.5 
    Amount = 10000   1   = Rs.10,769 (approx.) 
 100 
    C.I. = Amount – Principal = 10769 – 10000 = Rs. 769 

www.apparteducation.com 21 info@apparteducation.com
APPART - Academy of Proficient Professionals for Aptitude Research and Training

Example 7: The difference between the C.I. and S.I. on a certain amount at 10% per annum for 2 years, compounded annually is 
Rs.372. Find the principal
Solution : Let the principal be P.
2 P
    S.I.=  P  10   and   
100 5
2
 10  21 P
    C.I. = Amount – P = P  1   P   
 100  100
    C.I. – S.I. = Rs.372 
21 P P
  Rs.372 
    100 5
    P = Rs.37,200 
Alternative Method: You need to understand the fact that for 1 st period, S.I. = C.I.
The difference between the values of C.I. and S. I. is because of accumulated interest building on interest 
which  is  reinvested.  Therefore,  for  period 2,  the  difference  between  C.I.  and  S.I.  is  the  interest  on  the 
interest for period 1. 
In the above example, the difference being 372 is the interest generated on interest for period 1 on the 
principal.   
100
      Interest for period 1 = Rs.  372   Rs. 3,720 
10
100
      Therefore, Principal = Rs. 3720    Rs. 37,200 
10
Class Exercise 1(Percentage, Profit & Loss and Partnerships)
Q1.   What percent of 5/3 is 3/4? 
  (1) 50%    (2) 55%    (3) 40%    (4) 45% 
Q2.   In a container, 13 litres of milk was poured and it was still 35% empty. How many litres of water should be added to the 
container to fill it to the brim? 
  (1) 10 litres  (2) 9 litres  (3) 8 litres  (4) 7 litres 
Q3.  The price of a shirt is increased by 15% and then decreased by 15%. The final price of the shirt: 
  (1)decreases by 2.25 %    (2)does not change   
(3)increases by 2.25 %    (4) can’t be determined 
Q4.   If 8% of 80% of a number is 8 then the number is 
  (1) 512    (2) 200    (3) 225    (4) 125 
Q5.  The price of a commodity  increases by 20%. By what percentage the consumption should be reduced so that there is no 
change in the total expenses? 
  (1) 25%    (2) 16.66%  (3) 20%    (4) 30% 
Q6.   The S.P. of 12 notebooks is same as the C.P. of 13 notebooks. What is the gain percent? 
  (1) 1%    (2) 12.5%  (3) 8.33%  (4) 33.33% 
Q7.  A shopkeeper sells a chair at Rs. 350 and makes a profit of 25%. What was the C.P. of the chair? 
  (1) Rs. 437.50  (2) Rs. 280  (3) Rs. 300  (4) Rs. 250.50 

www.apparteducation.com 22 info@apparteducation.com
APPART - Academy of Proficient Professionals for Aptitude Research and Training

Q8.   A fruit vendor buys mangoes at the rate of 5 for Rs. 4 and sells them at the rate of 4 for Rs. 5. The profit percentage of the 
vendor is? 
  (1) 50%    (2) 56.25%  (3) 25%    (4) 33.33% 
Q9.   A  trader makes 20% profit by  selling a certain  quantity  of raw material.  How much profit  would he  make  if  he offers  a 
discount of 10%? 
  (1) 10%    (2) 8%    (3) 12%    (4) 15% 
Q10.   Bunty sold a chair to Bubbly at a profit of 20% and Bubbly sold it to Bobby at a loss of 10%. If Bobby paid Rs. 216 then how 
much did the chair cost to Bunty? 
  (1) Rs. 250  (2) Rs. 180  (3) Rs. 200  (4) Rs. 220 
Q11.  One shopkeeper offers a discount of 35% on an article whose marked price is Rs. 10000. Another   shopkeeper gives two 
successive discounts of 20% and 15% on a similar article marked at same price. What is the difference between both the 
selling prices? 
  (1) Rs. 275  (2) Rs. 300  (3) Rs. 328  (4) Rs. 248 
Q12.   A table sold at 7% loss would earn Rs. 64 more if sold at 9% profit. What is the cost price of the table? 
  (1) Rs. 352  (2) Rs. 422  (3) Rs. 400  (4) Rs. 450 
Q13.   Successive discounts of 10%, 20% and 40% are equal to a single discount of? 
  (1) 56.8%  (2) 70.2%  (3) 62.8%  (4) 53.4% 
Q14.  A retailer buys 260 eggs. He sells some of them at a profit of 20% and the remaining at a profit of 30%. If he gains 24% then 
how many eggs did he sell at the profit of 30%? 
  (1) 144    (2) 130    (3) 156    (4) 104 
Q15.  A reduction of Rs. 2 per meter enables a man to buy 4 more meters of cloth for Rs. 16. The cost of the cloth per meter is? 
  (1) Rs. 5    (2) Rs. 4    (3) Rs. 6    (4) Rs. 3 
Q16.  A fruit vendor buys 386 oranges. He sells half of them for Rs. 484 and gains 10% and he sells the remaining oranges at the 
same price of Rs. 484 but this time making a loss of 10%. The overall profit or loss of the vendor is? 
  (1) 5% loss  (2) 1% loss  (3) 1% profit  (4) No profit and no loss. 
Q17.   A seller sells meat at Rs. 44 per Kg thereby making a profit of 10%. If he makes a profit of Rs. 52,   how many Kgs of meat did 
he sell? 
  (1) 10 Kg   (2) 13 Kg   (3) 17 Kg   (4) 7 Kg 
Q18.   A dealer sold two shops for Rs. 6600 each. He made a profit of 10% on the first and a loss of 20% on the second. How much 
is his overall profit or loss? 
  (1)  30% profit  (2) 30% Loss  (3) 10% loss  (4) None of these. 
Q19.   Tarun sells a TV for Rs. 500 and a Recorder for Rs. 270 and makes an overall profit of 10% on both. Had he sold the TV for 
Rs. 380 and the Recorder at its cost price, he would have lost 10%. The cost price of the Recorder is? 
  (1) Rs. 280  (2) Rs. 255  (3) Rs. 250  (4) Rs. 265 
Q20.  A milkman bought 35 litres of pure milk at a rate of Rs. 15 per liter. He added 7 litres of water to it and sold the mixture at 
the rate of Rs. 17 per liter. His profit percentage is? 
  (1) 12%    (2) 26%    (3) 32%    (4) 36% 

www.apparteducation.com 23 info@apparteducation.com
APPART - Academy of Proficient Professionals for Aptitude Research and Training

Q21.  Sheila goes shopping and she spends 10% of her total amount on jewelry. She spends 20% of the remaining on household 
items and 25% of the rest on clothes. If after all these expenses she is left with Rs. 2700, find the initial amount she started 
with? 
  (1) Rs. 6000  (2) Rs. 4500  (3) Rs. 5000  (4) Rs. 3500 
Q22.  There are 38% girls in a class. If there are 144 more boys than girls then the total number of students in the class is? 
  (1) 300    (2) 500    (3) 600    (4) 400 
Q23.  Two numbers are respectively 10% and 40% less than a third number. How much percent is the second number less than 
the first number? 
  (1) 20%    (2) 30%    (3) 33.33%  (4) 40% 
Q24.  In  measuring  the  area  of  a  rectangle,  the  length  was  taken  5%  more  and  breadth  was  taken  4%  less  by  mistake.  The 
percentage error in the area is 
  (1) 0.5%   (2) 1%    (3) 1.2%   (4) 0.8% 
Q25.   A 6 litres of 5% salt solution was accidentally left in the sunlight. After sometime, the concentration increased to 6% due to 
evaporation. How much water should be added to it now to get back the 5% salt solution? 
  (1) 0.5 litres  (2) 2 litres    (3) 1 litre    (4) 1.5 litres 
Q26.  The  radius  of a  circle  is  increased  such  that  the  circumference of  the  circle  increases  by 7%.  The  area  of  the  circle  will 
increase by? 
  (1) 7%    (2) 10.5%  (3) 14%    (4) 14.5% 
Q27.  The  population  of  a  city  increases  by  20%  in  a  particular  year  and  decreases  by  15  %  in  the  year  following  it.  The  net 
percentage change in the population of the city in the two years is? 
  (1) 35%    (2) 2%    (3) 38%    (4) 5% 
Q28.  Raghu and Ram started a business in which Raghu invested Rs. 2000  initially and invested Rs. 1000 more at the end of 8 
months. Ram invested Rs. 750 initially and Rs 3000 more at the end of 4 months but withdrew Rs. 1300 at the end of next 3 
months. If they earned a profit of Rs. 1635 at the end of the year, what should be Raghu’s share? 
  (1) Rs. 840  (2) Rs. 820  (3) Rs. 740  (4) Rs. 780 
Q29.  A, B and C join a partnership contributing Rs. 2000, Rs. 1500 and Rs. 1250 respectively. What is A's share if total profit is Rs. 
3610?  
  (1) Rs. 1500   (2) Rs. 2290   (3) Rs. 1870   (4) Rs. 1520 
Q30.   A starts a business with Rs. 4000. B joins him after 3 months with Rs. 8000. C puts a sum of Rs. 12,000 in the business for 2 
months only. At the end of the year, the business gave a profit of Rs. 5,200. Find the share of B? 
  (1) Rs. 1,500   (2) Rs. 1, 800  (3) Rs. 2,600  (4) Rs. 4,000 
Q31.   If 60% of a number is added to 60, we get the same number again. The number is? 
  (1) 60    (2) 120    (3) 150    (4) 300 
Q32.  A student got 32% marks and failed by 7 marks whereas her friend got 44% marks and got 14 marks more than the passing 
marks. The maximum number of marks in the examination was? 
  (1) 150    (2) 200    (3) 175    (4) 225 
 

www.apparteducation.com 24 info@apparteducation.com
APPART - Academy of Proficient Professionals for Aptitude Research and Training

Q33.   A butcher sells meat at 70% profit. If he starts giving a discount of 20%, his profit would reduce by Rs. 68. The cost price of 
the meat is? 
  (1) Rs. 160  (2) Rs. 195  (3) Rs. 220  (4) Rs. 200 
Q34.   A milkman  sells milk at a  gain of 20%.  If he adds 1/10th water to it then  the percentage increase in  the profit would be 
equal to? 
  (1) 18%    (2) 20%    (3) 14%    (4) 12% 
Q35.  A person bought a cow and a buffalo. If he sold the cow at 10% loss and the buffalo at 20% profit, he will have no profit and 
no loss. If he sold the cow at a profit of 5% and the buffalo at a loss of   15%,  he  would  lose  Rs.  800  in  the  bargain.  How 
much did he pay for the buffalo? 
  (1) Rs. 13000  (2) Rs. 16000  (3) Rs. 9000  (4) Rs. 21000 
Q.36  In  an examination marks  obtained  by Shantanu  is  40%  less  than  the  marks  obtained  by  Kamal,  then  marks  obtained  by 
Kamal is how much percent more than the marks obtained by Shantanu? 
(1) 55 2/3    (2) 44 3/5  (3) 33 1/3  (4) 66 2/3 
Q.37  Due to an increase of 30% in the price of eggs, 6 eggs are less available for Rs 7.80. The present rate of eggs per dozen is  
(1) Rs 5.50  (2) Rs. 4.68  (3) Rs 6.49  (4) Rs 3.58 
Q.38  Ram sells book at a profit of 5%. If he had bought it at 10% less and sold it for Rs 6 more, he would have gained 20%. Find 
the cost price of the book. 
(1) Rs 200  (2) Rs 255  (3) Rs 250  (4) Rs 185 
Q.39  If the cost price is 95% of the selling price, what is the profit per cent? 
(1) 4    (2) 4.75    (3) 5    (4) 5.26   
Class Exercise 2 (Simple & Compound Interest)
Q.1  A sum of Rs. 3,500 is lent for 5 years at 5% p.a. The S. I. and amount respectively are 
  (1) Rs. 785, Rs. 4,375    (2) Rs. 875, Rs. 3,675   
  (3) Rs. 500, Rs. 4,375    (4) Rs. 875, Rs. 4,375 
Q.2  In what time, a sum of money will triple itself at the rate of 20% p.a., interest calculated as S. I. 
  (1) 5 years  (2) 10 years   (3) 15 years  (4) 20 years 
Q.3  What will be the C. I. on Rs. 1,000 for 3 yrs at 10% p.a.?   
(1) Rs. 331  (2) Rs. 330  (3) Rs. 300  (4) Rs. 361 
Q.4  If C. I. for a certain sum for 2 years at 2% p.a. be Rs. 1,010, what is the principal? 
  (1) Rs. 20,000  (2) Rs. 25,000  (3) Rs. 25,250  (4) Rs. 27,500 
Q.5  At what rate per cent, the interest on Rs. 1,125 will be Rs. 225 in 4 years? 
  (1) 4%    (2) 5%    (3) 6 3 %   (4) Can't be determined 
Q.6  In what time will Rs. 36 become Rs. 45 at 6.25% p.a. simple interest? 
(1) 2 years  (2) 3 years  (3) 4 years  (4) 8 years 
Q.7  The simple interest on Rs. 400 for 8 months at the rate of 5 paise per rupee per month is: 
  (1) Rs. 120  (2) Rs. 160  (3) Rs. 200  (4) Rs. 400 
Q.8  If Re.1 becomes Rs. 10 in 50 years at simple interest, the rate percent per annum is 
  (1) 15%    (2) 18%    (3) 20%    (4) 24% 

www.apparteducation.com 25 info@apparteducation.com
APPART - Academy of Proficient Professionals for Aptitude Research and Training

Q.9  The  difference  between  the  interests  received  from  two  different  banks  on  Rs.  500  for  2  years  is  Rs.  2.50.  Find  the 
difference between their rates. (Assume S. I.) 
  (1) 1%    (2) 2.5%   (3) 0.25%  (4) 0.5% 
Q.10  Find the difference between S.I. and C.I. on Rs. 700 at the rate of 10% for 3 yrs. 
  (1) Rs. 20.90  (2) Rs. 21.00  (3) Rs. 21.70  (4) Rs. 24.00 
Q.11  S.I. on a  sum of money  is  one fourth of principal. The number of  years is equal to  the  rate of  interest. Find  the rate  of 
interest. 
  (1) 2.5%   (2) 7.5%   (3) 6%    (4) 5% 
Q.12  What is the sum which when lent at 5% S.I. for 2 years would yield Rs. 154? 
  (1) Rs. 1,450  (2) Rs. 1,540  (3) Rs. 1,650  (4) Rs. 1,480 
Q.13  I  owe  you  Rs.  1,500  to  be payable  4 years  from  now.  What  is  the  equivalent  cash  payment  that  I  can make  now  (S.  I. 
prevailing being 6.25% p.a.)? 
  (1) Rs. 1,000  (2) Rs. 800  (3) Rs. 1,400  (4) Rs. 1,200 
Q.14  If I lend Rs. 5,000 for 3 years in two schemes: I. 11 % S.I., II. 10% C.I. 
  Which scheme is more profitable and by what amount? 
  (1) I, Rs. 150  (2) I, Rs. 50  (3) II, Rs. 5  (4) II, Rs. 50 

RATIO AND PROPORTION


Introduction: This is one of the most important topics in arithmetic from aptitude point of view. Ratio is an extension of the concept 
of fractions. These are fairly simple topics and will need a focused approach to refresh these concepts. 
Ratio: The concept of ratios is used for comparing two or more quantities of a similar kind. The definition of ratio is given below: 
The ratio of the quantity "A" to the quantity "B" is a relation that tells us what multiple or fraction the quantity "A" is of the quantity 
"B". 
A
The ratio of quantity A to quantity B is denoted by A:B and it is measured by the fraction  . 
B

Consider the two quantities A and B such that A: B = 2: 3. In what ways can we interpret this relation? Read the following. 
i.  The ratio B: A is 3: 2. 
2 rd
ii.  A is   part of B. 
3

iii.  B is 1.5 times that of A. 
iv.  B is 50% more than A. 
V  A is 33.33% less than B. 
If both the terms of a ratio are multiplied or divided by the same (non-zero) quantity, then the value of the ratio remains the same. 
In terms of notations: 
a ka
  (Here, k is any non-zero real number.) 
b kb

On the other hand, if both the terms of a ratio are added or subtracted by the same (non-zero) quantity, then the value of the ratio 
changes. Additionally, the relation between the new ratio obtained and the old ratio might also change, which will be discussed 
later. 

www.apparteducation.com 26 info@apparteducation.com
APPART - Academy of Proficient Professionals for Aptitude Research and Training

Example 1: Solve these problems 
I.  A and B got 175 and 225 marks respectively. What is the ratio of their marks? 
II.  X scored 105 marks out of 150 and Y scored 175 marks out of 200. What is the ratio of the percentage marks scored by 
each? 
175 7
Solution:  I.  A: B     7: 9  
225 9
105 200
    II.  X: Y    4:5  
150 175
Example 2: 5 kg of wheat flour is mixed with 500 gm of sugar extract. What is the ratio of sugar extract to the rest of the mixture 
after adding 1.5 kg of water? 
Solution:   We first need to express all quantities in a single unit. 
    Wheat flour = 5 kg  
    Water = 1.5 kg 
    Sugar extract = 500 gm = 0.5 kg  
    Total weight of the mixture = 7 kg 
0.5
Ratio of sugar extract to the rest of mixture    1: 13  
6.5
Example 3: Divide Rs. 1000 between A and B in the ratio of 7: 3. 
Solution:  Let us assume that A gets Rs. 7x and B gets Rs. 3x as 7x + 3x = 1000  
1000
      x  100  
10
      A gets Rs. 700 and B gets Rs. 300 
6 16
Example 4: What must be subtracted from the numerator and the denominator of the fraction  to give a fraction equal to  
7 21
6  x 16
Solution: Let the number subtracted be x.   On solving, we get x = 2.8 
7  x 21
Example 5: Ram's father is thrice as old as Ram was, 2 years ago. Five years from now, his father's age will be 6 years more than 
twice the Ram's age. What is Ram's present age? 
Solution: Let Ram's present age be X and his father's present age be Y. 
    Y = 3(X – 2) and (Y + 5) – 6 = 2(X + 5). Solving,  
    we get, X = 17 years and Y = 45 years. 
2
Example 6: A’s income is  rd of B's income. B's income is 75% of C's income. What is the ratio of C's income to A's income? 
3

3
Solution: B's income =    of A's income. 
2

4 4 3
    C's income =    of B's income     of A's income   Required ratio = 2: 1. 
3  3 2

X
Example 7: Let the ratio A: B is measured by the fraction    . If the quantities A and B are fractions then can X and Y be integers? 
Y 

 a  c  X ad 
Solution: Let A    and B   Now A: B       
 b  d  Y bc 
    As each of a, b, c and d are integers, ad and be are integers as well.  
    So, X and Y are integers. 

www.apparteducation.com 27 info@apparteducation.com
APPART - Academy of Proficient Professionals for Aptitude Research and Training

P p
Example 8:Let one or both of the two quantities P and Q (P  Q) are surds. If   then can p and q simultaneously be integers? 
Q q

Solution: If either or both of the quantities P and Q (P # Q) are surds then there exist no two integers p and q which can exactly 
P p
measure the ratio P: Q. In terms of the notations, if   then not both of p and q are integers.  
Q q

P
Choose arbitrary values for P and Q. Try to simplify the fraction  so that you can get both p and q as integers. You will never be 
Q

successful!!  
Comparison of Ratios
Question: Which of the following is/are correct:? 
113 13 87 27 27 13 15 5
(1)     (2)     (3)      (4)     
115 15 85 25 17 9 19 9

a c
To compare two ratios  and  , first make their denominators of the same sign. Now 
b d

a c
I.  If (ad – bc) > 0 then    
b d

a c
II.  If (ad – bc) < 0 then    
b d

a c
III.  If (ad – bc) = 0 then    
b d

Apply the above concepts for each option. You will see that (a), (b) and (d) are correct and (c) is incorrect. There are some other 
methods and shortcuts as well, to compare the ratios. Read the following concepts and understand the following examples carefully 
and make yourself comfortable with these methods as they are extremely helpful in Data Interpretation problems. 
A
Now, we will discuss some important properties of the ratios. Let us assume a ratio  . What happens when we add or subtract the 
B

same quantity from both the numerator and the denominator? The result, in fact, depends on whether 
A
 1 or< 1? We have summarized the results for both the cases as under:  
B

A
Case I: If   1 . 
B

A x A
1.   (x  0)  
B x B

A x A
2.   (x  0)  
B x B

A
Case II: If  1  
B

A x A
3.   (x  0)  
B x B

A x A
4.   (x  0)  
B x B

You can verify the above results by choosing any arbitrary values for A, B and x. Try to memorize these results as these are extremely 
helpful in Data Interpretation problems. 

www.apparteducation.com 28 info@apparteducation.com
APPART - Academy of Proficient Professionals for Aptitude Research and Training

13 15 11 12
Example 9: Which one is the greatest of  , , and  ?  
11 13 9 10
Solution:  Difference in the numerator and denominator is 2 in each case. Also, the fractions        
    are all more than 1. 
11
    Therefore, greatest fraction is  . 
9

    [Note: Here, Case I – (2) is used] 
Example 10: If a: b = 2:5, then find the ratio 2a – 3b: 5a + 7b.  
a b 2
2 3 2  3
2 2a  3b b b 5 11
Solution:    a:b       
5 5a  7b 5 a  7 b 5  2  7 45
b b 5
Proportion
 When two ratios are equal, the four quantities composing them are said to be proportionals. 
A C
In terms of the notations, if   then A, B, C and D are proportionals. 
B D

We use the symbol "::" to express it mathematically. So whenever we write A: B:: C: D, it is interpreted as A, B, C and D are 
proportionals. 
The terms A and D are called the extremes and the terms B and C are called the means. It is very easy to note that A x D = B x C. 
This result is more commonly expressed as: "Product of the extremes = Product of the means". 
a b c d e
Continued Proportion: a, b, c, d, e, f...are said to be in continued proportion if       ...  
b c d e f
When three quantities a, b and c are in continued proportion then b is called the mean proportional and c is'called the third 
proportional. 
a b
  b2  a  c  
b c
When four quantities a, b, c and c proportionals then d is called the fourth proportional. 
Example 11: As amount of Rs. 1,150 is to be divided among A, B and C such that the ratio of share of A to that of B is equal to 3: 2 
and share of B to share of C is equal to 3: 4. Find their individual share. 
Solution:     Here A: B = 3: 2 = 9: 6 and B: C = 3: 4 = 6: 8  
    Therefore, A: B: C = 9: 6: 8  
    Rs. 1150 can be divided between them as follows: 
1150  9
    A's share    Rs. 450 
23
1150  6
    B's share    Rs. 300 
23
1150  8
    C's share     Rs. 400 
23
Example 12: In the year 1996, the monthly allowances given to A, B and 0 were in the ratio of 5: 3: 1. If C's monthly allowance was 
Rs. 1000 then what was total allowance received by A, in that year? 
Solution:  C's share = Rs. 1,000 
    A's share: C's share = 5: 1 
    Therefore, A's monthly share = Rs. 5,000 
    A's shore for whole year = 5000    12 = Rs. 60,000 

www.apparteducation.com 29 info@apparteducation.com
APPART - Academy of Proficient Professionals for Aptitude Research and Training

Example 13: Find the third proportional to 3, 5. 
Solution:   Let the third proportional be "x" then 3: 5:: 5: x 
3 5 25
  x  
    5 x 3

A C
Operations on Ratios:Let   . Three very important results are derived from the following operations. 
B D

1.Componendo operation 
2.Dividendo operation 
3.Componendo and Dividendo operation 
A C
Componendo Operation:  As    
B D

A C AB C D
1  1    
      B D B D

This operation is called componendo. We will make se of this important result while solving problems o a variety of topics. 
A C
Dividendo Operation:   As    
B D

A C AB C D
1  1    
      B D B D

This operation is called dividendo. This is an equally rnportant result. 
A C AB C D
If   then    
B D AB C D

We will solve a few examples using the above results. 
Example 14: If p: q:: r: s then prove that 2p + 3q: 2p – 3q:: 2r + 3s: 2r – 3s. 
p r
Solution:  We have    
q s

2
    Multiplying both the sides by  we get 
3

2 p 2 r 2 p 2r
   or   
    3 q 3 s 3q 3s

    Using the componendo and dividendo property we get 
2p  3q 2r  3s
  
    2 p  3q 2r  3s

1 x  1 x
Example 15: Solve for  x , 2  
1 x  1 x

1 x  1 x
Solution:  2  
1 x  1 x

    Applying the Componendo and Dividendo: 
( 1  x  1  x ) ( 1  x  1  x) 2 1
  
    ( 1 x  1 x)( 1  x  1 x) 2 1

2 1x 3
    or   
2 1 x 1

www.apparteducation.com 30 info@apparteducation.com
APPART - Academy of Proficient Professionals for Aptitude Research and Training

1 x
Simplifying and then squaring on both sides we get:  9 
1 x

You can proceed to get 1 + x = 9 x (1 – x) and solve for x. 
Another way of proceeding from here is to apply componendo and dividendo, one more time. 
(1  x)  (1  x) 9  1 5 4
So that we get:       . This gives,  x  . 
(1  x)  (1  x) 9  1 4 5

a c
Example 16: If   then prove that 
b d

ab cd
a.     
b d

ab cd
b.    
a b c d

ab cd
c.    
a b c d

a c a c ab c d
Solution: a.     1   1  or    
b d b d b d

a c
    b.    
b d

ab c d
        k (say) 
b d

       (a  b)  bk and (c  d)  dk  

ab b
          … (i) 
cd d

  Similarly, 
ab b
         … (ii) 
c d d

  From (i) and (ii) 
ab cd
        
a b c d

a c
    c.    k , say 
b d

       a  bk and c  dk  

a  c bk  ck a
  So that,     k  
bd bd b

a c e
Example 17:If     K ,then prove that 
b d f

ac e
  a.  K  
bd  f

pa  qc  re
  b.  K     (p, q and r are not all zero) 
pb  qd  rf

1
 pa n  qc n  ren  n
  c.   n  K   (p, q and r are not all zero) 
 pb  qd n  rf n 
 

www.apparteducation.com 31 info@apparteducation.com
APPART - Academy of Proficient Professionals for Aptitude Research and Training

a c e
Solution:     K , given 
b d f

     a  bK , c  dK and e  fK  
Substituting, we get 
a  c  e bK  dK  fK
  a.   K   hence proved 
bd  f bd  f
pbK  qdK  rfK
  b.   K     hence proved 
pb  qd  rf
1
 p(a)n  q(c)n  r (e)n  n
  c.  
 pbn  qd n  rf n 
  
 
1
1
 p(bK )n  q(dK )n  r ( fK )n  n n n
    



 (K ) K  
 pbn  qd n  ef n 
Direct Proportion and Inverse Proportion
Direct Proportion: Let there be two variables A and B. They are said to be in direct proportion if the ratio A: B is constant for all the 
possible values of A and (the corresponding values of) B. We can understand this in terms of percentages also: Two quantities A and 
B are in direct proportion if change in the value of A, by a certain percentage, always corresponds to the same percentage change in 
the value of B. Direct Proportion is characterized by the following equation: 
A
   K , a constant 
B

B 1
Note: That   , again a constant. When A is in a direct proportion to B then B is in a direct proportion to A as well. 
A K

Inverse Proportion: Two variables A and B are said to be in inverse proportion if the product A  B is constant for all the possible 


values of A and (the corresponding values of) B. We can understand this in terms of percentages also: 
Two quantities A and B are in inverse proportion if increase/decrease in the value of A, by a certain percentage, always corresponds 
to the same percentage decrease/increase in the value of B. Inverse Proportion is characterized by the following equation: A    B = 
K, a constant 
Note: That when A is in inverse proportion to B then B is in inverse proportion to A as well. 

AVERAGES AND MIXTURES


An average or an arithmetic mean is the sum of all observations divided by the total number of observations. 
Conceptually, it is the number that represents the entire set statically.  
Example 18: The average score of Sachin after 25 innings is 46 runs per innings. If after the 26th innings, his average score increased 
by 2 runs, then what is his score in the 26th innings? 
Solution: Runs in 26th inning = Total runs after 26th innings - Total runs after 25th innings = 26 x 48 - 25 x 46 = 98 
Alternatively, this question can be done by the above given central value meaning of average. Since the average increases by 2 runs 
per innings, we can assume that 2 runs have been added to his score in each of the first 25 innings. Now, the total runs added in 
these innings have been contributed by the scored in the 26th inning, which must be equal to 25 x 2 = 50 runs. 
And after contributing 50 runs, his score in the 26th inning is 48 runs. 
Hence, runs scored in the 26th inning = new average + old innings x change in average = 48 + 25 x 2 = 98. 

www.apparteducation.com 32 info@apparteducation.com
APPART - Academy of Proficient Professionals for Aptitude Research and Training

Properties of Average: 
1.   Average always lies in between the maximum and the minimum value. It can be equal to themaximum or minimum value if 
all the numbers are equal. 
2.   Average is the resultant of net surplus and net deficit, as used in the central tendency  medhod. 
3.   When weights of different quantities are same, then simple method is used to find the average.   However, when   different 
weights of different quantities are taken then it is known as weighted   average. Here the method of weighted average is 
used to find the average. For exmaple, assume per capita income of  India is USD 500 and per capita income of US is USD 
200. Now if we merge India and US into one country then it is observed that per capita   income of this new country will not 
be euqal  to: 500+200/2 = USD 350. 
4.  If the value of each quantity is increased or decreased by the same value S, then the average will  also increase or decrease 
respectively by S. 
5.  If the value of each quantity is multiplied by the same value S, then the average will also be multiplied by S. 
6.  If the value of each quantity is divided by the same value S (S # 0) then the average will also be divided by S. 
Weighted Average
In simple average, the elements of the entire set put the same weight in the group. When the elements have different numbers 
assigned to them as weight, then the weighted average is given by  
(p1q1 + p2q2)     (q1 + q2) 
Where, p1 and p2 are the respective averages of the groups and q1 and q2 are the respective weights assigned to them. 
For example,  students 
There are two sections of a class with 36 and 44 students respectively. If the average age of first section is 40 years and that of the 
second section is 35 years, what’s the average weight of the entire class. 
Solution is given by the concept we discussed about weighted average. Here, the weights assigned are 36 and 44 respectively. So, 
the weighted average is  
(36 * 40 + 44 * 35)    
(36 + 44) 
i.e. 2980 
        80 
37.25 years 
Note: Mixtures and alligations are extended concepts of weighted average 
 

MIXTURE (ALLIGATION)
 These types of questions are very important and involve the theory of ratio proportion percentages, profit and loss. 
Generally questions on mixtures are based on either simple mixtures or compound mixtures. Simple mixtures involve mixing of only 
two items, while compound mixture may involve more than two items. 
Alligation helps us 
To find the mean or average values of mixtures, when the prices of two or more ingredients which may be mixed together and the 
proportion in which they are mixed are given. 
To find the proportion in which the ingredients at given prices must be mixed to produce a mixture at a given price. 
When quantities at two different prices are mixed in a certain ratio, then our aim is to find the price of the final mixture. 

www.apparteducation.com 33 info@apparteducation.com
APPART - Academy of Proficient Professionals for Aptitude Research and Training 

Quantity of Cheaper Dearer Price - Mean Price
  
Quantity of Dearer Mean Price - Cheaper Price

Where Pd& Qd, means the cost price and quantity of the dearer item; Pc&Qc means the cost price and quantity of the cheaper item 
and mean price Pm means the price of the final mixture. 
You can derive this formula from the weighted average formula discussed before.  
Sometimes in particular questions, profit and loss condition is given along with some other data. 
Example 19. In what proportion must a grocer mix one kind of tea at Rs. 45 per kg with another at Rs. 40 per kg so that the final 
mixture costs him Rs. 41.50 per kg? 
Solution: Mean price = Rs. 41.50. Dearer price = Rs. 45. 
    Cheaper price = Rs. 40. Putting the values in the formula 
Quantity of Cheaper 45  41.5 3.5 7
  
Quantity of Dearer 41.5  40 1.5 3  
 He should mix 7 parts of tea costing Rs. 40 per kg with 3 parts of tea costing Rs. 45 per kg to get the final mixture. 
 

Example 20. How many kg of tea selling at Rs. 10.40 per kg should be mixed with tea selling at Rs. 8.80 per kg to make a mixture 15 
kg at Rs. 146.40? 
Solution: Cost per kg of resulting mixture = 146.40/15 = Rs. 9.76 

    Putting the values in the formula 
Quantity of Cheaper 10.40  9.76 0.64 2
    
    Quantity of Dearer 9.76  8.80 0.96 3

    The two varieties should be mixed in the proportion 3: 2. (Dearer: Cheaper = 3:2) 
    In 15 kg of mixture there should be 9 kg of tea @ Rs. 10.40 and 6 kg of tea @ Rs. 8.80: 
Example 21: In what proportion must sugar costing Rs. 14 per kg and Rs. 17 per kg be mixed so that 20% profit is earned by telling 
the mixture at Rs. 18 per kg? 
Solution: Mean price = 18/ 1.2 = Rs. l5. 
    Dearer Price = Rs. 17. 
    Cheaper Price = Rs. 14. 
Quantity of Cheaper 17  15 2
Putting the values in the formula     
Quantity of Dearer 15  14 1

He should mix 2 parts of the cheaper sugar with 1 part of the costlier sugar to get the final mixture. 
Example 22: In two alloys, the ratio of zinc to tin are 3: 2 and 2: 3. If 7 kg of the first alloy and 21 kg of the second alloy are mixed 
together to form a new alloy, then what will be the ratio of zinc and tin in the new alloy? 
Solution:      Zinc  Tin 
    1st alloy    3  2 
nd
    2  alloy   2  3 
  Quantity mixed    Zinc  Tin 
  From I      4.2  2.8 
  From II      8.4  12.6 
  Total      12.6  15.4 

www.apparteducation.com   34  info@apparteducation.com  


APPART - Academy of Proficient Professionals for Aptitude Research and Training 

  Hence the ratio of zinc to that of tin = 12.6/15.4 = 9/11 
IMPORTANT: If a vessel contains "x" liters of milk and if "y" liters be withdrawn and replaced by water, then if "y" liters of mixture is 

withdrawn and replaced by water and the operation is repeated 'n' times in all, then   
 n
Milk left in the vessel after nth operation  x  y 
 
Initial quantity of milk in the vessel   x   
 
Example  23. 10 gallons are drawn from a vessel full of wine. It is then filled with water. 10 gallons of mixture are again drawn and 

vessel is again filled with water. The quantity of wine now left in the vessel to that of water bears a ratio of 49: 32. How much does 
the vessel hold? 
Solution: Since initially there was only wine in the vessel, hence finally the total quantity of wine and      
    water should be equal to the initial quantity of wine. 
    Let initially wine be X gallons. Applying the formula 
 2
Wine left in the vessel after nth operation  x  10 
 
Initial quantity of winw in the vessel   x   
    2
49 49  X  10 
  
49  32 81  X   Solving, we get X = 45 gallons. 
   
Example 24: Nine liters are drawn from a cask full of wine and it is then filled with water. Nine liters of the mixture are drawn and 
the cask is again filled with water. The quantity of wine now left in the cask to that of water in it bears a ratio of 16: 9. How much 
does the cask hold? 
n
A  q
Solution:  1  
  Q  Q 

    A = quantity of wine in the final mixture. 
    Q = volume of cask. 
    q = quantity removed. 
    n = number of times the operation is repeated. 
2
A 16 16  9 
    1
    Q 16  9 25  Q   

    => Q = 45 liters 
Class Exercise
Q. 1.  A: B = 3: 7 and the sum of A and B is 45. Find the value of B. 
   (1) 28      (2) 33.5      (3) 31.5      (4) 36 
Q. 2.  A fraction bears the same ratio to 3/7 as 1/27 does to 1/35. Find the fraction. 
   (1) 4/9      (2)1/ 3      (3) 3/5      (4) 5/9 
Q. 3.  Mean proportional between 8 and 72 is 
  (1) 24      (2) 40      (3) 16      (4) 32 
Q.4.  A sum of Rs 53 is divided among A, B and C in such a way that A gets Rs 7 more than what B gets and B gets Rs 8 more than 
what C gets. The ratio of their shares is 
  (1) 16:9:18    (2) 25:18:10    (3) 18:25:10    (4) 15:8:30 

www.apparteducation.com   35  info@apparteducation.com  


APPART - Academy of Proficient Professionals for Aptitude Research and Training 

Q. 5.  Three mixtures containing water and alcohol in the ratio of 5: 2, 6: 1 and 4:3 are mixed in equal quantities. The ratio of 
water to alcohol in the resulting mixture is: 
  (1) 5: 2      (2) 7: 3      (3) 6: 4      (4) 7: 4 
Q. 6.  The cost of type 1 sugar is Rs. 15 per kg and type 2 sugar is Rs 20 per kg. If both are mixed in the ratio 2:3, then the price per 
kg of the mixed variety is. 
  (1) 18.5      (2) 18      (3) 19      (4) 19.5 
Q. 7.  The ratio of 4 3.5: 2 5 is same as: 
   (1) 2: 1      (2) 4: 1       (3) 7: 5      (4) 7: 10 
Q. 8.  How much wheat at Rs.3 a kg must be added to 5 kg of wheat at Rs.6 a kg so that the mixture is worth Rs.4 a kg.? 
  (1) 5 Kg      (2) 8 Kg      (3) 10 Kg     (4) 9 Kg 
Q. 9.  What must be subtracted from each term of the ratio 68: 49 so that it becomes 3: 4? 
  (1) 45      (2) 20      (3) 5      (4)125 
Q. 10.  Rs. 3,960 is divided among A, B and C such that half of A's part, one third of B's part and one sixth of C's part are equal. Then 
B's part is 
  (1) Rs. 1,080    (2) Rs. 960    (3) Rs. 1,720    (4) Rs. 1,540 
Q. 11  8 litres are drawn from a cask full of wine and is then filled with water. The operation is performed three more times. The 
ratio of the quantity of wine now left in the cask to that of water is 16:65. How much wine did the cask hold originally? 
  (1) 18 litres    (2) 32 litres    (3) 42 litres    (4) 24 litres 
Q.12.  The average of the first four of five numbers is 40 and that of the last four numbers is 60. The difference of the last and the 
first number is: 
  (1) 400      (2) 200      (3) 80      (4) 40 
Q. 13  If 4x=3y=2z, then x:y:z is 
  (1) 4:3:2     (2)2:3:4      (3) 3: 4: 2    (4) 3: 4: 6 
Q. 14  The average age of 8 persons in a committee is increased by 2 years when two men aged 35 years and 45 years are 
substituted by two women. The average age of these two women is: 
(1) 50 years    (2) 56 years    (3) 44 years    (4) 48 years 
Q. 15.  In a mixture of 100 L, the ratio of milk and water is 3: 1. If 200 L of water is added in the mixture,  what will be the new ratio 
of milk and water? 
  (1) 1: 3      (2) 3: 1      (3) 2: 5      (4) 5: 2 
Q. 16  Tea costing Rs 126 per kg and Rs 135 per kg are mixed with a third variety in the ratio 1 : 1 : 2. If the mixture is worth Rs.153 
Per keg, the price of the third variety per kg will be: 
  (1) Rs 169.5    (2) Rs 175.5    (3) Rs 170    (4) Rs 180 
Q.17.  A container contains 40 litres of milk. From this container 4 litres of milk was taken out and replaced by water. This process 
was repeated further two times. Approximately, how much milk is now contained by the container?   
(1) 26      (2) 29      (3) 28      (4) 27 
Q.18.  A box containing a dozen ceramic mugs is dropped. Some of the mugs broke. Which of the following cannot be the ratio of 
broken and unbroken mugs? 
  (1) 2: 1      (2) 5: 7      (3) 7: 5      (4) 3: 2 

www.apparteducation.com   36  info@apparteducation.com  


APPART - Academy of Proficient Professionals for Aptitude Research and Training 

Q.19.  If (a + b): (b + c): (c + a) = 6: 7: 8 and (a + b + c) = 14, then the value of c is: 
  (1) 6      (2) 7      (3) 8      (4) 14 
Q.20.  The ratio of third proportional to 12 and 30 and the mean proportional between 9 and 25 is: 
  (1) 2: 1      (2) 5: 1      (3) 7: 15     (4) 9: 14 
Q. 21  The ratio of A's money to that of B's money is 4: 5 and B's money to C's money is 2: 3. If A has   Rs. 800,  then total 
amount of money among A, B and C is: 
  (1) Rs. 2,790    (2) Rs. 3,300    (3) Rs. 3,000    (4) Rs. 3,620 
Q. 22.  The number 68 is divided into two parts such that one-seventh part of the first is equal to one-tenth part of the second.Find 
the first part. 
  (1) 7      (2) 22      (3) 28      (4) 32 
Q. 23.  Rs. 9,700 has been divided among X, Y and Z such that if their shares are reduced  respectively by Rs. 30, Rs. 20 and Rs. 50, 
the balances are in the ratio of 3: 4: 5. What is Y's share? 
  (1) Rs. 3,180    (2) Rs. 3,220    (3) Rs. 3,253.33    (4) Rs. 3,200 
Q. 24.  The sides of a triangle are in the ratio of 1/2 : 1/3 : 1/4 and its perimeter 104 cm. The length of the longest side is: 
  (1) 52 cm    (2) 48 cm    (3) 32 cm    (4) 26 cm 
Q. 25.  The sum of Rs. 530 is divided among A, B and C such that A gets Rs. 70 more than B and  B gets Rs. 80 more than C. What is 
the ratio of the amount with A and C? 
  (1) 25: 18    (2) 18: 10    (3) 9: 5      (4) 5: 2 
Q. 26.  An amount of money is distributed amongst A, B and C such that A gets half that of B and B gets twice that of C. What is the 
ratio between the share of B to that of the sum of the shares of A and  B? 
  (1)2:5      (2) 2:3      (3) 3:2      (4) 4:3 
Q. 27.  In a class of 500 students. the number of boys equals the number of girls. If 1/5 th of the girls left the class and 25 boys 
joined in, what is the ratio of the number of boys to the number of girls, now? 
  (1) 3: 2      (2) 12: 7     (3) 11: 8     (4) 9: 8 
Q. 28.  The present ages of a man and his son are in the ratio of 7: 2. After 15 years, their ages would be  in the  ratio of 2: 1. What 
was father's age when the son was born? 
  (1) 25      (2) 30      (3) 35      (4) 42 
Q. 29.  Four years ago, a man's age was 6 times that of his son. 12 years from now, his age will be twice that of the son. What is the 
ratio of their present ages? 
  (1) 6: 1      (2) 7: 1      (3) 8: 2      (4) 7: 2 
Q. 30  A solution having milk and water in the ratio 2:3 is mixed with another milk and water solution. The resultant solution has 
milk and water in the ratio 4:5. Find the milk and water ratio in the second solution if the two solutions were mixed in the 
ratio 3:2.   
  (1) 21:23    (2) 23:22    (3) 22:23    (4) 23:21 
Q. 31.  A solution having milk and water in the ratio 2:3 is mixed with another milk and water solution. The resultant solution has 
milk and water in the ratio 4:5. Find the milk and water ratio in the second solution if the two solutions were mixed in the 
ratio 3:2.   
  (1) 21:23    (2) 23:22    (3) 22:23    (4) 23:21 
www.apparteducation.com   37  info@apparteducation.com  
APPART - Academy of Proficient Professionals for Aptitude Research and Training 

Q. 32.  Brass is an alloy of copper and zinc and has no other metal in it. In a sample of brass, copper and  zinc are in the ratio of 
13:7. How much copper will be there in a 500 kg sample of this alloy? 
  (1) 300 kg    (2) 325 kg    (3) 175 kg    (4) 150 kg 
Q. 33.  If it is given that x varies inversely as the square of y and that y = 2 for x = 1, then the value of x   for y = 6 will be: 
  (1) 3      (2) 9      (3)1/ 3       (4) 1/9 
Q. 34  The compounded ratio of (2: 3), (6: 11) and (11: 2) is: 
  (1) 1: 2      (2) 2: 1      (3) 11: 24    (4) 36: 121 
Q. 35.  Ratio of the earnings of A and B is 4: 7. If the earnings of A increases by 50% and those of B decrease by 25%, the new ratio 
of their  earnings become 8: 7. What is A's earning? 
  (1) Rs 21,000    (2) Rs 26,000    (3) Rs 28,000    (4) Data Inadequate 
Q. 36.  The average age of three boys is 25 years and their ages are in the proportion of 3: 5: 7. The age of the youngest boy is: 
  (1) 21 years    (2) 18 years    (3) 15 years    (4) 9 years 
Q. 37.  The value of a diamond varies directly as the square of its weight. If a diamond worth Rs. 10,000   is divided into 2 pieces 
having weight in the ratio of 4: 6, what is the loss in value? 
  (1) 52%      (2) 48%      (3) 36%      (4) None of these 
Q. 38  A bag contains 50p, 25p and 10p coins in the ratio of 5: 9: 4, amounting to Rs 206. Find the   number of 50p coins. 
  (1) 100      (2) 125      (3) 150      (4)200 
Q. 39.  How much amount of sugar costing Rs 9 per kg must be mixed with 27 kg of sugar costing Rs 7 per kg so that there may be 
gain of 10% by selling the mixture at Rs 9.24 per kg? 
  (1) 36 kgs    (2) 42 kgs    (3) 54 kgs    (4) 63 kgs 
Q. 40.  The volume of a sphere varies directly as the cube of its radius. If three cubes of radius 3cm, 4cm  and 5cm are melted and 
recast into a solid sphere, then find the radius of the sphere. 
  (1) 5.5 cm    (2) 6 cm     (3) 7 cm     (4) 7.5cm 
Q. 41.  The average age of the boys in the class is 16 years and that of the girls is 15 years. The average   age for the whole class is: 
  (1) 15 years    (2) 15.5 years     (3) 16 years    (4) Cannot be determined 
Q. 42.  The average score of a cricketer for ten matches is 38.9 runs. If the average score for the first six   matches is 42 runs, then 
find the average for the last four matches. 
  (1) 33.25    (2) 33.5      (3) 34.25    (4) 35 
Q. 43.  The average age of 8 men is increased by 2 years when two of them whose ages are 21 years and 23 years are replaced by 
two new men. The average age of the two new then is: 
  (1) 22 years    (2) 24 years    (3) 28 years    (4) 30 years 
Q. 44.  A pupil's marks were wrongly entered as 83 instead of 63. Due to that the average marks for the   class got increased by 
half. The number of pupils in the class is: 
  (1) 10      (2) 20      (3) 40      (4) 73 
Q.45   The average weight of 11 players of Indian cricket team is increased by 1 kg, when 1 player of the team weighing 55 kg is 
replaced by new player. The weight of new player is  
(1) 55 kg     (2)  64 kg     (3) 66 kg     (4) None of these  

www.apparteducation.com   38  info@apparteducation.com  


APPART - Academy of Proficient Professionals for Aptitude Research and Training 

Q.46  The average age of a family of 6 members 4 years ago was 25 years. Meanwhile a child was born in this family and still the 
average age of the whole family is same today. The present age of the child is  
(1) 2 yr      (2) 1.5 yr     (3) 1 yr       (4)  Data Insufficient  
Q. 47   A cat can take 5 leaps for every 4 leaps of a dog, but 3 leaps of the dog are equal to 4 leaps of the cat, what is the ratio of 
speeds of the cat to that of dog? 
(1) 11:15    (2) 15:11    (3) 16:15    (4) 15:16 
Q .48  600 g of sugar solution has 40% sugar in it. How much sugar should be added to make it 50% in the solution? 
(1) 160 g    (2) 120 g     (3) 130 g     (4) 140 g 

TIME SPEED AND DISTANCE


 
Relationship between Time, Speed and Distance: 
Speed is the rate of change of distance. 
Time, Speed and Distance are related as Distance = Speed   Time. 
It means that if a person is walking at 5 km/h and he walks for 2 hours, then he will be covering a total of 10 kms. 
From the relation distance (d) = speed (s)   time (t), we can deduce three important relations: 
Proportionality
1. When Distance (d) is constant, then s   t = k (constant) 
k 1
s  or s  
  t t
  Hence, more the speed, lesser is the time taken and more the time taken, lesser is the speed at which distance is travelled.  
d
2. When Time (t) is constant, then  = k (Constant)    d = ks or d   s. 
s
  Hence, higher  is  the  speed,  the  more  will  be  the  distance  covered and  lower  the  speed,  the  lesser  will  be  the  distance 
covered.  
d
3. When Speed (s) is constant, then   = k (Constant)   d = kt or d   t. 
t
  Hence, if a  person  is  running at a  constant  speed, then  the  ratio  of distance  covered  in one hour to  distance  covered  in 
three hours will be 1: 3. 
3 1
Example 1: Walking at  of his usual speed a man is  1 hr late in reaching his office from home. Find his usual travel time. 
4 2
Solution: Let usual time be ‘t’ hours. Since the distance travelled is the same, 
1
    Hence, s   
t
3
    New speed =   old speed (given) 
4
4 4 3
     New time =   old time    t  t  ; t  = 4.5 hr. 
3 3 2

www.apparteducation.com   39  info@apparteducation.com  


APPART - Academy of Proficient Professionals for Aptitude Research and Training 

3s 3
Alternative method: st=  (t  )
4 2
    4st = 3st + 4.5 s 
    t = 4.5 hr.  
Example 2: A man covers a certain distance between his house and office on scooter. Having an average speed of 30 km/hr, he is 
late by 10 min. However, with a  speed of 40 km/hr, he reaches his office 5 min earlier. Find the distance between his house and 
office. 
Solution: Let the distance be x km. 
x
    Time taken to cover x km at 30 km/hr =  hrs.  
30
x
    Time taken to cover x km at 40 km/hr =  hrs. 
40
1
    Difference between the time taken =15 min =  hrs. 
4
x x 1
   or, 4x - 3x = 30 or, x = 30  
    30 40 4
Hence, the required distance is 30 km. 
Average Speed
Total distance travelled
Average speed =  
Total time taken
Suppose, a man covers a distance d1 km at s1 km/hr and a distance d2 km at s2 km/hr, then 
d1  d 2
Average speed of the whole travel =   km/hr 
d1 d 2

s1 s2
dd 2s 1 s 2
If the distances are equal, then Average speed =   km/hr 
d d s1  s 2

s1 s2
Note:
Two cases of Average Speed when a man travels at speeds s1 and s2. 
1. When he travels for equal time, average speed is  
(s1 + s2) / 2 
2. When he travels for equal distance (as mentioned in the example, average speed is  
2s1s2 / (s1 + s2) 
Example 3: A car during its journey travels 30 minutes at a speed of 40 km/hr, another 45  minutes at a speed of 60 km/hr and 2 
hours at a speed of 70 km/hour. Find the average speed of the car.  

 30   45 
  40     60   (2  70)
60 60
Solution: Average speed        = 63 km/hr.  
30 45
 2
60 62

www.apparteducation.com   40  info@apparteducation.com  


APPART - Academy of Proficient Professionals for Aptitude Research and Training 

Relative Speed
If two bodies are moving (in the same direction or in the opposite direction), then the speed of one body with respect to the other is 
called its relative speed. Relative speed is a phenomenon that we observe every day. Suppose you are travelling in a train and there' 
is a second train coming in the opposite direction on parallel track, then it seems that the second train is moving much faster than 
actual. If both the trains were moving in the same direction on parallel tracks at same speeds, they  seem to be stationary  if seen 
from one of these trains, even though they might actually be at a speed of 100 km/hr each. So  what you actually observe is your 
speed relative to the other. 
Concepts:
1. If two objects are moving in opposite directions towards each other or away from each other on a straight- line at speeds u 
and v, then they seem to be moving towards each other or away from each other at a relative   speed = Speed of first + 
Speed of second = u + v. 
2.  If the two objects move in the same direction with speeds u and v, then 
 Relative speed = difference of their speeds = u -v. 
 This  is also the speed at  which  the  faster  object  is  either  drawing  closer  to the slower object  or  moving away  from  the 
slower object as the case may be. 
Example 4: A police officer at a distance of 200 m spots a thief. If the speed of the thief be 10 km/hr and that of the police officer be 
12 km/hr, then how far will the thief runs before the policeman catches him? 
Solution: Relative speed of the police officer = 2 km/hr 
    Time taken by the police officer to cover the additional  
200 1 1
    200 m =   hr    hr 
1000 2 10
1
    In this time, the thief covers = 10   = 1 km. 
10
Example 5: A train running at 54 km/hr takes 20 s to cross a platform and 12 s to pass a man walking in the same direction at a 
speed of 6 km/hr. Find the length of the train and the platform. 
Solution: Let the length of the train = x m. Let the length of the platform = y m.. 
40
    Speed of the train relative to the man = 48 km/hr    m/s 
3
    In passing the man, the train covers its own length with relative speed. 
40
    Length of the train =   12 = 160 m. Since speed of train = 54 km/hr = 15 m/s 
3
xy
 20 or x + y = 300 or y = 140. Length of the platform = 140 m. 
    15
1
Example 6: A hare makes 9 leaps in the same time as a dog makes 4. But the dog's leap is 2 m while hare's is only 1 m. How many 
3
leaps will the dog have to make before catching up with the hare if the hare has a head start of 16 m? 
7 28
Solution: Distance covered by dog in 4 leaps = 4    m 
3 3

www.apparteducation.com   41  info@apparteducation.com  


APPART - Academy of Proficient Professionals for Aptitude Research and Training 

    Distance covered by hare in 9 leaps = 9   1 = 9 m  
1
    Distance gained by the dog in 4 leaps =  m. Hence, for 1 m gain he has to make 12 leaps. 
3
    Number of leaps required by the dog to gain 16 m = 12   16 = 192 leaps. 
Boats and Streams
Downstream motion of a boat is its motion in the same direction as the flow of the river. 
Upstream motion of a boat is its motion in the opposite direction as the flow of the river.  
There are two parameters in these problems. 
1. Speed of the stream or river (R): This is the speed with which the river flows. 
2. Speed of the boat in still water (B): If the river is still, this is the speed at which the boat would be moving. 
The effective speed of a boat while moving upstream = B - R 
The effective speed of a boat while moving downstream = B + R 
1
3. The speed of the boat in still water is given as B =  (d + u) and the speed of the river 
2
1
R (d - u), Where, d and u are the downstream and upstream speeds, respectively. 
2
Example 7: A man rows 27 km downstream and 18 km upstream taking 3 hours each. What is the velocity of the current? 
27
Solution: Rate downstream =  = 9 km/hr 
3
18
    Rate upstream =  = 6 km/hr 
3
    Velocity of current = 0.5 (9 - 6) = 1.5 km/hr. 
Example 8: A man can row upstream at 7 km/hr and downstream at 10 km/hr. Find his rate in still water and the rate of the current. 
Solution: Rate in still water = 0.5 (10 + 7) = 8.5 km/hr  
    Rate of current = 0.5 (10 - 7) = 1.5 km/hr 
Example 9: A man can row a boat 30 km upstream and 44 km downstream in 10 hrs. Also, he can row 40 km upstream and 55 km 
downstream in 13 hrs. Find the rate of the current and the speed of the boat in still water. 
Solution: Let, upstream rate = x k/hr and downstream rate = y km/hr 
30 44 40 55
    Then,    = 10 and    = 13 
x y x y
    or 30u + 44v = 10 
    40u + 55v =13 
1 1
    Where u =  and v =   
x y

     x = 5 and y = 11 
5  11
     Rate in still water =  = 8 km/hr 
2
11 - 5
    Rate of current =   = 3 km/hr  
2
www.apparteducation.com   42  info@apparteducation.com  
APPART - Academy of Proficient Professionals for Aptitude Research and Training 

Races on linear tracks


A contest of speed is called a race. Racing events takes place in various activities like running, riding, driving, walking, rowing, etc. A 
basic assumption is that contestant’s cover the race course at uniform speed unless specified in the problem. 
Let us get used to the various terminologies used in a race on which questions are usually based. 
1. Race course: The path on which race takes place is called racecourse. 
2. Starting point: The point of beginning of the race is called the starting point. 
3. Finishing Point: The point where the race finishes is called the finishing point. 
4. Dead Heat Race: If two or more than two contestants finish the race in exactly the same time and there can be no single winner of 
the race, then the race is said to be a dead heat race. 
5. "A beats B by 20m" means that A has reached the finishing point while B is still 20m behind the finishing point. 
6.  "A beats B  By  5  seconds"  means  that  A  has  reached  the  finishing  point  while  B  will  still  take  another  5  seconds  to  reach  the 
finishing point. 
7."A gives B a start of 40m" means that while A starts the race from the starting point while B starts at a point 40m ahead of the 
starting point. 
8."A gives B a start of 5 seconds" means that A starts the race when B had already ran for 5 seconds. 
Example 10: In a km race A beats B by 25 meters or by 5 seconds. Find the time taken by A to complete the race. 
Solution: A beats B by 25 m or by 5 seconds means that B will take 5 seconds to complete the last 25 m of the race course. 
25
      B's speed =  = 5 m/s 
5
Total race course 1000
  Time taken by B to finish the race =  =  = 100 sec 
Speed of B 5
  Time taken by A to finish the race = Time taken by B - 5 seconds = 200 - 5 = 195 sec 
Example 11: In a race of 100 m, Ajay gives a start of 10 m to Vijay. Despite this, Ajay wins the race by 20 m. What is the ratio of the 
speed of Ajay and Vijay? 
Solution: Time taken by Ajay to cover 100 m = Time taken by Vijay to cover 70 m. 
100 70
 =   Ajay's speed: Vijay's speed = 10: 7 
   
Ajay' s speed Vijay' s speed
Example 12: Sonu beats Munnu by 40 m and Chunnu by 50 m in a 200 m race, while Munnu beats Chunnu by 2 seconds in the same 
race. How long does each take to run 400 m? 
Solution: While Sonu ran 200 m, Munnu ran 160 m and Chunnu ran only 150 m. 

      Sonu’s speed: Munnu's speed: Chunnu's speed  
    = 200: 160: 150 
    = 20: 16: 15 

     Munnu's speed = 16x m/s and Chunnu's speed = 15x m/s 
Also, time taken by Munnu to complete the race = time taken by Chunnu - 2 secs 
200 200
  2 
    16x 15x

www.apparteducation.com   43  info@apparteducation.com  


APPART - Academy of Proficient Professionals for Aptitude Research and Training 

100
     x =  
15  16
100
      Sonu's speed = 20x = 20   m/s 
15  16
400 400  15  16
      Time taken by Sonu to cover 400 m =   = 48 secs 
20x 20  100
Similarly, time taken by Munnu and Chunnu to cover 400 m is 60 secs and 64 secs, respectively. 
Circular Motion
The problems we have encountered till now covered motion in straight line, where the path is not closed i.e. open. In linear motion, 
we observed that, if two bodies moved with different speed in one direction and the body with faster speed overtook the body with 
slower speed, then the bodies never meet again. However, if the same bodies move on a circular track, then the bodies are bound to 
meet again since the track is enclosed (by virtue of being circular). 
The problems in circular motion deal with races on a circular track to calculate the time of meeting at the starting point or anywhere 
else on the track. 
Concepts
1. If two people A and B start from the same point, at the same time and move in the same direction along a circular track and 
take x minutes and y minutes respectively to come back to the starting point, then they would meet for the first time at the 
starting point according to the formula:  
  First time meeting of A and B at the starting point = (LCM of x and y) 
Note: This formula would remain the same even if they move in the opposite directions. 
2. If two people A and B start from the same point with speeds m km/hr and n km/hr respectively, at the same time and move 
in  the  same/opposite  direction along  a  circular  track,  then  the  two  would  meet  for  the  first  time  by  the  formula  given 
below: Time of the first meeting 
Circumference of the track
=   
Relative speed
  First time at the starting 
Direction  First time anywhere on the track 
  point 
SAME 
LCM   (  , )   
Direction  −
Two People 
OPPOSITE 
LCM   (  , )   
Direction  +

SAME  LCM   ( ( ,
Three People  LCM   (  , , )  ) ( ) ) 
Direction 

 
Example 13: A, B and C run around a circular track 1200 m long at respective speeds of 9, 18 and 27 km/hr. If they start at the same 
point and at the same time in the same direction, when will they meet again at the starting point? 

www.apparteducation.com   44  info@apparteducation.com  


APPART - Academy of Proficient Professionals for Aptitude Research and Training 

Solution: L = 1200 m 
5
    Speed of A = 9   = 2.5 m/sec 
18
5
    Speed of B = 18   = 5 m/sec  
18
5
    Speed of C = 27   = 7.5 m/8ec 
18
L L L
They will meet for the first time at a time which is the LCM of  , and  . 
a b c
L 1200
 = 480s 
    a 2.5
L 1200
 = 240s  
    b 5
L 1200
 = 160s 
    c 7.5
LCM of 480, 240,160 is 480s. 
Hence, they will meet for the first time at this starting point 8 minutes from the time they start.  
Example 14: A, B & C run along a circular track having a length of 1.2 km, with speeds of 6 km/hr, 8 km/hr & 9 km/hr respectively. A 
& B run in the same direction but C runs in the opposite direction. If they all start at the same time & from same place, how many 
times will A & C meet anywhere on the track by the time A & B meet for the first time anywhere on the track? 
Solution: Time taken by A & b to meet for the first time anywhere on the track 
1.2
= =  
( ) (8 − 6)
.
=   = 0.6 hours = 36 minutes  

Time taken by A & b to meet for the first time anywhere on the track 
.
=( )
.= ( )
 
.
=   = 4.8 minutes  

The number of times A & C meet anywhere on the track by the time A & B meet each other for the first time = 36 ÷ 4.8 = 7 ½ i.e. 7 
times 
Class Exercise
Q.1  Two persons  start from  P and Q  with the speeds of 25 km/hr  and 49 km/hr respectively towards each other.  After  they 
cross each other, the person from B covers 145 km to reach A. What is the distance AB? 
  (1) 437.1 km    (2) 428.4 km    (3) 429.2 km    (4) 441.2 km 
Q.2  Two cities A and B are 110 km apart. Person A started from city P at 7 a.m. at 20 km/hr and B started from city Q at 8 a.m. 
at 25 km/ hr. At what time will they meet? 
  (1) 10 a.m.    (2) 11 a.m.    (3) 1 p.m.    (4) 12 noon. 
 
www.apparteducation.com   45  info@apparteducation.com  
APPART - Academy of Proficient Professionals for Aptitude Research and Training 

Q.3  Walking at three-fourths of his normal speed, a man is late by 1.5 hr. The usual time is: 
  (1) 4.5 hr    (2) 6hr,      (3) 7.5 hr    (4) 9 hr 
Q.4  If Abhi walks  from his house at 4 km/hr, he reaches school 10 min  early. If he walks at 3 km/hr, he reaches 10 min late. 
What is the distance from his house to school? 
  (1) 2 km     (2) 4 km     (3) 8 km     (4) 16 km 
Q.5  A, who is travelling at 3.5 km/hr, starts 2.5 hr before B who travels at 4.5 km/hr in the same direction as A. In how much 
time will B overtake A? 
  (1) 9.25 hr    (2) 8.75 hr    (3) 9.75 hr    (4) 8.5 hr 
Q.6  A thief, who had  escaped at 7 p.m, was followed by a policeman at 9 p.m. at the rate of 6 km/hr. At what time will  the 
policeman overtake him, supposing the thief runs at 4.5 km/hr? 
  (1) 2 a.m.    (2) 3 a.m.    (3) 4 a.m.    (4) 1 a.m. 
Q.7  Manu goes to school at a speed of 3 km/hr and return to home at a speed of 2 km/hr. If he takes 5 hrs in all, what is the 
distance between home and school?  
  (1) 6km      (2) 7.5 km    (3) 4.5 km    (4) 5 km 
Q.8  Two men A and B walk from P to Q, a distance of 21 km, at 3 and 4 km an hour respectively. B reaches Q,   returns 
immediately and meets A at R. Find the distance from P to R.  
  (1) 18 km    (2) 12 km    (3) 16 km    (4) 21 km 
Q.9  Excluding stoppages, the speed of a bus is 54 km/hr and including stoppage, it is 45 km/hr. For how long does the bus stop 
per hour? 
  (1) 20 min.    (2) 15 min.    (3) 12 min.    (4) 10 min. 
Q.10  The driver of a  car sees a bus 40 m ahead of him. After 20 s, the bus is 60 m behind. If the speed of the car is 30 km/hr, 
what is the speed of the bus? 
  (1) 6 km/hr    (2) 12 km/hr    (3) 9 km/hr    (4) 24 km/hr 
Q.11  Abhishek on a platform notices that a train going in one direction takes 10 s to pass him, and a train of same length going in 
the opposite direction takes 15 s to pass him. What is the time taken by the two trains to pass one another if the length of 
the trains is 200 m each? 
  (1)15s      (2)12s      (3)18s      (4)24s 
Q.12  A train leaves A at 40 km/hr. At the same time, another train departs from B at a speed of 60 km/hr. They  reach the 
respective destinations and turn back immediately towards the starting points. Now if they meet at a distance of 200 km 
from A, what is the distance between A and B? 
  (1) 275 km    (2) 125 km    (3) 250 km    (4) 300 km 
Q.13  The speeds of A and B are in the ratio 3 : 4. A takes 30 min more than B to reach the destination. How much time does A 
take to  reach the destination? 
  (1) 1 hr      (2) 2 hr      (3) 1 hr 30 min    (4) 2 hr 30.min 
Q.14  A man sees a train passing over a bridge 1 km long. The length of the train is half that of the bridge. If the   train  clears  the 
bridge in 2 min, the speed of the train is 
  (1) 45 km/hr    (2) 60 km/hr    (3) 30 km/hr    (4) 90 km/hr 
 

www.apparteducation.com   46  info@apparteducation.com  


APPART - Academy of Proficient Professionals for Aptitude Research and Training 

Q.15  Two trains travel in opposite directions at 36 km/hr and 45 km/hr respectively. A man sitting in the slower train passes the 
faster train in 8 s. The length of the faster train is 
  (1) 180m    (2) 150 m    (3) 210 m    (4) 175 m 
Q.16  A train 110 m long is travelling at a speed of 58 km/hr. What is the time in which it will pass a man walking in the same 
direction at 4 km/hr? 
  (1) 6.33s     (2) 9.33s     (3) 8.33s     (4) 7.33s 
Q.17  A man travels three–fifths of distance AB at a speed of 3a, and the remaining at a speed of 2b. If he goes from B to A and 
back at a speed of 5c in the same time, then: 
  (1) a + b = c    (2) 1/a + 1/b = 1/c  (3) 1/a + 1/b = 2/c  (4) 1/a + 1/b = 5/c 
Q.18   A bus crosses 19 electric poles in 10 seconds. If the distance between any two successive poles is 5 metres, then what is the 
speed of the bus (km/hr)? 
 (1) 32.4      (2) 34.2      (3) 35.2      (4) 36 
Q.19   A  train  consists  of  12  bogies  (inclusive  of  the  engine).  Each  bogie  is  15m  long.  The  train  crosses  a  telegraph  post  in  18 
seconds. Due to some problem two bogies were detached. Moving at the same speed the train now crosses the telegraph 
post in 
(1) 18 seconds     (2) 12 seconds    (3) 15 seconds    (4) 20 seconds 
Q.20  A stream runs at 1 km/hr. A boat goes 35 km upstream and come back again in 12 hr. The speed of the boat in still water is 
  (1) 6 km/hr    (2) 15 km/hr    (3) 9 km/hr    (4) 4.5 km/hr 
Q.21  A man rows a distance downstream in 45 min and the same distance upstream in 75 min. What is the ratio of speed of the 
stream   to the boat in still water?  
  (1) 1: 4      (2) 3 : 5      (3) 5 : 3      (4) 4:1 
Q.22  A man can row at 6 km/hr in still water & a river is flowing at 4 km/hr. How long will the man take to go to a place 1 km 
downstream & return? 
(1) 36 minutes     (2) 24 minutes    (3) 12 minutes    (4) 18 minutes 
Q.23  A man can row at 12 km/hr in still water. He finds that it takes him thrice as much time to row up the river as it takes to row 
down the river. What is the speed of the current? 
(1) 6 km/hr     (2) 24 km/hr    (3) 48 km/hr    (4) 18 km/hr 
Q.24  A boat is moving downstream and reaches its destination in 25 hours while moving at a speed of 50 km/hr   (given 
speed is in still water). One particular day due to  engine problem at mid-point and ship's speed reduction by 20% of the 
original, it reaches its destination 2.5 hours late. Find out the speed of the river. [Assume its speed to be uniform]  
  (1) 12 km/hr    (2) 15 km/hr    (3) 20 km/hr    (4) 10 kn/hr 
Q.25  In a race of 100 metres, Pawan beats Arun by 4 metres and Pawan beats Rahul by 2 metres. By how many  metres  would 
Rahul beat Arun in a 100-metre race? 
  (1) 4.18m    (2) 2.04m    (3) 2.12 m    (4) 3.36 m 
Q.26  A is twice as fast as B. If A gives B a start of 60 metres. How long should the race course be so that both of them reach at 
the same time? 
  (1) 150 m    (2) 120 m    (3) 180 m    (4) 90 m 

www.apparteducation.com   47  info@apparteducation.com  


APPART - Academy of Proficient Professionals for Aptitude Research and Training 

Q.27  Bhim and Arjun were exercising during their Vanvaas. They start running on a circular track simultaneously and in the same 
direction. If Bhim takes 4 min to complete one full round, and Arjun takes 7 min to complete one full round after how much 
time will they meet for the first time? 
  (1) 9 min. 40 sec     (2) 9 min. 10 sec                (3) 9 min. 30 sec       (4) 9 min. 20 sec 
Q. 28  A can run 330 m in 41 seconds & B can run the same distance in 44 seconds. By how many second/seconds will B win, if he 
has a 30 m start? 
(1) 2       (2) 1       (3) 3      (4) 1 ½ 
Q.29  Speed of sound is 330 m/s in air while it is 440 m/s in a liquid. The total time taken by sound to cover a distance of 6.6 km in 
air & 4.4 km in that liquid is 
(1) 20 seconds     (2) 10 seconds    (3) 30 seconds    (4) 35 seconds 
Q.30  A man covers half of his journey at 6 km/hr and the remaining half at 3 km/hr. Find his average speed? 
    (1) 3 km/hr    (2) 4 km/hr     (3) 405 km/hr     (4) 9 km/hr  
Q.31  If Sohail walks from his home to office at 16 km/hr, he late by 5 min. If he walks 20 km/hr , he reaches 10 min before the 
office time. Find distance of his office from his house    . 
  (1) 22 km       (2) 20 km    (3 18 km     (4) 16 km  
Q.32   A certain distance is covered at a certain speed. If half of this distance is covered in 4 times of the time Find the ratio of two 
speeds. 
(1) 1:8 (2) 1:4 (3) 4:1 (4) 8:1
Q.33   A train passes two persons who are walking in the direction opposite to the direction of train art rate of  
    10 m/s and 20 m/s respectively in 12  s and 10 s respectively.  Find the length of the train. 
(1) 500m (2) 900m (3) 400m (4) 600m

TIME AND WORK


If a man can do a piece of work in X units of time then the work done by him in one unit of time will be equal to 1/X of the total 
work. 
If A can complete a work in X days and B can complete it in Y days, then the number of days in which they can together complete the 
XY
work is:  days. 
XY
If A can complete a work in X days, B in Y days and C can complete it in Z days, then the number of days in which they can together 

XYZ
complete the work is:  days. 
 XY  YZ  ZX 
Generally, the following types of questions are asked in various companies. 
Type of question  Example  Approach to question 
Calculate the time taken by two  A can complete a piece of work in 10 days  XY
T , where X and Y are the time 
persons working together to finish a  which B alone can complete in 12 days. In  XY
work.  how many days can they finish the work is  taken by A and B individually. 
they both work together?  10  12 120 5
T   5 days. 
10  12 22 11

www.apparteducation.com   48  info@apparteducation.com  


APPART - Academy of Proficient Professionals for Aptitude Research and Training 

Calculate the time taken by three  A can complete a piece of work in 5 days  XYZ


T , Where X, Y and Z are 
persons working together to finish a  which B alone can complete in 6 days. If C,  XY  YZ  ZX
work.  who can complete the work in 12 days,  the time taken by A, B and C individually. 
joins them, how long will they take to  5  6  12
  
complete the work?  5  6  6  12  12  5
360 2
 2 days 
162 9
If one worker is ‘m’ times as efficient  A is thrice as efficient as B and takes 60  mD
T  
as another worker and takes ‘D’ days  days less than B for finishing a job. Find  m2  1
less than the other person, then in  the time in which they can finish the work  3  60 180 45
   days 
how many days can both finish the  together.  91 8 2
work working together?  = 22.5 days 

One pipe can fill a cistern in T1 mins  A pipe can fill a cistern in 30 min and  T1  T2
T , where T1 and T2 are the time 
and together pipe in T2 mins. If both  another can fill it in 40 min. if both are  T1  T2
the pipes are opened together, find  opened simultaneously, find the time  taken by each pipe individually. 
the time taken to fill the cistern.7  taken to fill the cistern.  30  40 1200 1
   17 mins. 
30  40 70 7
One tap can fill a cistern in T1 min and  A cistern is filled by a tap A in 10hrs and  T1  T2
T , where T1 and T2 are the time 
another tap can empty it in T2 min. If  emptied by tap B in 12 hrs. if both the  T1  T2
both the taps are opened together,  taps are opened simultaneously, find the  taken by each tap individually to fill and empty 
find the time taken to fill the cistern.   time taken to fill the cistern.  the cistern respectively. 
10  12 120
   60 hrs. 
12  10 2
Note: In general, we can say that, If ‘w1’ work is done by ‘m1’ men working ‘h1’ hours per day in ‘d1’ days and ‘w2’ work is done by 
‘m2’ men working ‘h2’ hours per day in ‘d 2’ days, then 
m1 d 1 h1 m 2 d 2 h2

w1 w2  
This is based on the principle that M is inversely proportional to D. Further, H is inversely proportional to D. however, W is directly 
proportional to D. 
Example 1: 5 men can pack 10 boxes in 6 days, working 6 hours a day. Then in how many days can 12 men pack 16 boxes working 8 
hrs a day? 
m 1 d 1 h1 m 2 d 2 h 2
Solution: We know that,    
w1 w2
5  6  6  16
     d2   3 days. 
12  8  10
Example 2: To do a piece of work Ram would take three times as long as Rohan and Raj together, and Raj will take twice as long as 
Rohan and Ram together. The three boys together complete the work in 10 days. How long would each boy alone take to complete 
the work? 

www.apparteducation.com   49  info@apparteducation.com  


APPART - Academy of Proficient Professionals for Aptitude Research and Training 

Solution: 3 times Ram's daily work = (Rohan + Raj)'s daily work. Add Ram's daily work to both the sides. 

       4 times Ram's daily work = (Rohan + Rare + Raj)'s daily work = 10 
       Ram's daily work = 40  
    Also, 2 times Raj's daily work = (Rohan + Ram)'s daily work. Add Raj's daily work to both the sides. 

       3 times Raj's daily work = (Rohan + Ram + Raj)'s 
1
    Daily work    
10
1
       Raj's daily work   
30
1  1 1  1
    Now Rohan's daily work       
10  40 30  24
       Rohan, Ram, and Raj can do the work in 24, 40 and 30 days respectively. 
Example 3: Two men and 3 boys can do a piece of work in 10 days while 3 men and 2 boys can do the same work in 8 days. In how 
many days can 2 men and 1 boy do the same work? 
Solution: Let 1 man's 1 day's work = x  
    Let 1 boy's 1 day's work = y 
1 1
     2 x  3y  and 3x  2 y   
10 8
    On solving the above equations, we get 
7 1
x and y   
    200 100
7 1 16 2
       (2 men + 1 boy's) 1 day's work   2   1    
200 100 200 25
25
Thus, 2 men and 1 boy can finish the work in  days. 
2
Alternative Method: Two men and three boys can do a piece of work in 10 days. 
      Hence, in one day the number of men and boys required to finish the job =  
(2m +3b)    10   ... (1) 
      Similarly, if three men and two boys can do a piece of work in 8 days,  
      then in one day the number of men and boys required to finish the job =  
(3m + 2b)    8.    ... (2) 
By equating eqn. (1) and (2), we get (2m + 3b)    10 = (3m + 2b)    8.  
      20m + 30b = 24m + 16b 
7
      4m = 14 b        m   b 
2
7
      Now, 2m + 3b   2  b + 3b = 10b. 
2
7
      Hence, 2m + 1b   2  b + 1b = 8b 
2
      By unitary method, 10b   10 days 
10
       10 
8b    12.5 days. 
8

www.apparteducation.com   50  info@apparteducation.com  


APPART - Academy of Proficient Professionals for Aptitude Research and Training 

Efficiency
Efficiency is also known as rate of work done. 
If A is taking less number of days with respect to B to complete the same work, we can say that the efficiency of A is more efficient 
than B. 
In general time and efficiency are correlated as follows: 
i.   If efficiency of A is x% more than the efficiency of B and B takes 'D' days to complete the work, then A will take

 D  100   days to complete the same work. 
 
 100  x 
ii.   If efficiency of A is x% less than the efficiency of B and B takes `D' days to complete the work, then A will take 

 D 
  100   days to complete the same work. 
 100  x 
With this, it can also be observed that if work is constant then time taken is inversely proportional to efficiency. 
Example 4: Sony is thrice as efficient as Rakesh and hence completes a work in 60 days less than the number of days taken by 
Rakesh. In how many days will the work finish if both of them work together? 
Solution: Since Sony is thrice as efficient as Rakesh, so the number of days taken by him will be 1/3 rd the number of days 
taken by Rakesh. If Sony is taking x days, then Rakesh will take 3x days to complete the same work. 
    Now,  3x  x  2x  60 days 
    So,  x  30 days and  3x  90 days 
    Let us assume that the total work = 90 units (LCM of 30 and 90) 
    So, the total work done by both of them in one day = 3 + 1 = 4 units of work. 
90
So, the total number of days required to finish the work =   days = 22.5 days  
4
Pipes and Cisterns:
Pipes and cisterns is just another application of the concept of time and work. While we see only +ve work being done in normal 
cases of time and work, in case of pipes and cisterns, -ve work is also possible. 
Given that pipes A and B can fill a tank in 20 mins and 25 mins individually is similar to "A can do a work in 20 mins and B can do the 
same work in 25 mins." 
Example 5: A and B are two taps which can fill a tank individually in 10 min and 20 min respectively. However, there is a leakage at 
the bottom, which can empty a filled tank in 40 min. If the tank is empty initially, how much time will both the taps take to fill the 
tank (leakage is still there)? 
Solution: Let us assume the units of work = LCM of (10, 20, 40) = 40 units  
    Work done by Tap A/min = 4 units/min (Positive work)  
    Work done by Tap B/min = 2 units/min (Positive work)  
    Work done by leakage/min = 1 unit/min (Negative work) 
    Net work done/min = 4 + 2 - 1 = 5 units/min 
40
Hence, time taken to fill the tank =  = 8 mins. 
5

www.apparteducation.com   51  info@apparteducation.com  


APPART - Academy of Proficient Professionals for Aptitude Research and Training 

Example 6: Two pipes can fill a cistern in 14 hours and 16 hours respectively. The pipes are opened simultaneously and it is found 
that due to leakage in the bottom it took 32 minutes more to fill the cistern. In how much time can the leak empty the whole 
cistern? 

Solution: Work done by the two pipes in 1 hour  
 1 1  15

 
 14 16  112
112
     Time taken by these pipes to fill the tank =  hrs = 7 hrs and 28 min.  
15
          Due to leakage, time taken = 7 hrs 28 min + 32 = 8 hrs 
1
     Work done by (two pipes + leak) in 1 hour =   
8
 15 1  1
          Work done by the leak in 1 hour =       
 112 8  112

     Leak will empty the full cistern in 112 hours. 
 
Class Exercise 1 (Time & Work)
Q.1  A can do a piece of work in 7 days working 9 hours a day and B can do it in 6 days working 7 hours a day. How long will they 
42
  take to finish the work, working together  hours a day? 
5
  (1) 3 days    (2) 4 days    (3) 4.5 days    (4) 6 days 
Q.2  A can do a piece of work in 80 days. He works for 10 days and then B alone finishes the remaining work in  42 days. They 
together could complete the work in: 
  (1) 24 days    (2) 25 days    (3) 30 days    (4) 29 days 
Q.3  A and B can do a piece of work in 45 days and 40 days respectively. They begin together but A leaves after some days and B 
completes the rest in 23 days. For how many days did A work? 
  (1) 6 days    (2) 9 days    (3) 8 days    (4) 12 days 
Q.4.  A and B can do a job in 25 days and 20 days, respectively. A started the work and was joined by B  after 10  days. The total 
number of days taken to complete the work is 
  (1) 12.5 days    (2) 14.22 days    (3) 15 days    (4) 16.66 days 
Q.5  A and B can finish a piece of work in 72 days, B and C in 120 days while A and C can finish it in 90 days. In what time can A
  finish it alone? 
  (1) 150 days    (2) 120 days    (3) 100 days    (4) 80 days 
Q.6.  A and B together can do a job in 12 days and B and C together can do it in 16 days. First A and B works for 5 days, then B 
and C works for 7 days and thereafter, C finally finishes the rest of work in 7 days. In how many days can C do the work 
alone? 
  (1) 24 days    (2) 30 days    (3) 36 days    (4) 48 days 
Q.7  Twelve men can do a job in 8 days. Six days after they start, 4 more men join them. How many more days  will it take to do 
the job? 
  (1) 2.5 days    (2) 1 day     (3) 1.5 days    (4) 4 days 

www.apparteducation.com   52  info@apparteducation.com  


APPART - Academy of Proficient Professionals for Aptitude Research and Training 

Q.8  A job can be done by 10 men in 20 days or by 20 women in 15 days. How many days will it take for 5 men  and 10 women to 
  finish the work? 
1 1 1
  (1) 17  days    (2) 17  days    (3) 17 days    (4) 17  days 
2 7 20
Q.9.  R and S can do a job in 8 days and 12 days respectively. If they work on alternate days with R beginning the work, then in 
how many days will the work be finished? 
1 1 1 1
  (1) 9  days    (2) 9  days    (3) 9  days    (4) 10  days 
2 3 24 3
Q.10.  A, B and C can do a job in 11, 20 and 55 days respectively. On the first day A is assisted by B and on the second day by C. 
How soon can the work be completed if A is assisted by B and C on alternate clays? 
  (1) 7 days    (2) 9 days     (3) 8 days    (4) 10 days 
Q.11.  Machines A and B produce 8,000 clips in 4 hr and 6 hr respectively. If they work alternately for 1 hr, A starting first, then 
8,000 clips will be produced in 
  (1) 4.33 hr    (2) 5.66 hr    (3) 5.33 hr    (4) 4.66 hr 
Q.12  A does half as much work as B, while C does half as much work as A and B together in the same time. If C   alone can do the 
work in 40 days, all of them can together will finish the work in 
  (1) 13 days    (2) 15 days     (3) 20 days    (4) 13.33 days 
Q.13  A and B can do a piece of work in 10 days and 20 days respectively. Both starts the work together but A leaves the work 5 
days before its completion time. Find the time in which work is finished? 
  (1) 20 days    (2) 15 days     (3) 25 days    (4) 10 days 
Q.14  Pipes A, B and C can fill a tank in 12 minutes, 36 minutes and 18 minutes respectively. If all the pipes are open, then after 
that time C must be closed so that the tank is full in 8 minutes? 
  (1) 4 mins    (2) 2 mins    (3) 3 mins    (4) 5 mins 
Q.15  3 men and 4 boys can earn Rs. 756 in 7 days. 11 men and 13 boys can earn Rs. 3008 in 8 days. In what time will 7 men with 
9 boys earn Rs. 2480? 
  (1) 15 days    (2) 20 days    (3) 10 days    (4) 5 days 
Q.16  A, B and C together earn Rs. 1350 in 9 days. A and C together earn Rs. 470 in 5 days. B and C together earn Rs. 760 in 10 
days. Find the daily earning of C. 
  (1) Rs. 20    (2) Rs. 10    (3) Rs. 15    (4) Rs. 25 
Q.17  A and B can fill a tank in 6 hrs and 4 hrs respectively. If they are opened for alternate hours and A is opened first, then how 
many hours will it take to fill the tank? 
  (1) 4 hr      (2) 5 hr      (3) 4.5 hr    (4) 5.5 hr 
Q.18  A leak in the bottom of a tank can empty it in 6 hr. A pipe fills in the tank at the rate of 4 litres per minutes. When the tank 
is full, the inlet is opened but due to the leak, the tank is emptied in 8 hr. What is the capacity of the tank? 
  (1) 5760 L    (2) 5670 L    (3) 5846 L    (4) 6970 L 
Q.19  A certain number of men can complete a work in 40 days. If there were 10 men more, the work would be completed in 10 
days less. Initially, the number of men was? 
  (1) 25       (2) 20       (3) 24      (4)30 

www.apparteducation.com   53  info@apparteducation.com  


APPART - Academy of Proficient Professionals for Aptitude Research and Training 

Q. 20  40 boys can finish a work in 60 days. If 40 boys start the work & at the end of every 10 days 40 more boys join them, then 
how long will it take for the work to be complete? 
   (1) 30 days      (2) 20 days       (3) 40 days     (4) None of these 
Q.21  25 men and 15 women can complete a piece of work in 12 days. All of them start working together and after working for 8 
days the women stopped working. 25 men completed the remaining work in 6 days. In how many days can one woman 
complete the job?  
  (1) 60 days    (2) 36 days     (3) 94 days    (4) None of these 
Q.22  10 men and 15 women finish a work in 5 days. One man alone finishes that work in 100 days. In how many days will a 
woman   finish the work? 
  (1) 125 days    (2) 150 days    (3) 90 days    (4) 225 days 
Q.23  A can do a work in 6 days and B can do the same work in 5 days. The contract for the work is Rs. 220. How much shall B get 
if both of them work together?  
  (1) Rs. 100    (2) Rs. 120    (3) Rs. 80    (4) Rs. 140 
Q.24  A man can do a work in 10 days. With the help of a boy he can do the same work in 6 days. If they get Rs. 50 for that work, 
what is the share of that boy?  
  (1) Rs. 10    (2) Rs. 30    (3) Rs. 50    (4) Rs. 20 
Q.25  Two taps A & B can fill a tank in 9 minutes & 12 minutes respectively. In what time will the tank be full, if B is opened two 
minutes after A was opened? 
   (1) 6 min     (2) 8 min     (3) 10 min     (4) 12 min 
Q.26  Taps A and B can fill in a tank in 12 and 15 min respectively. If both are opened and A is, closed after 3 min, how long will it 
take B to fill in the tank? 
  (1) 8 min 15s    (2) 7 min 15s     (3) 8 min 5s    (4) 7 min 45s 
Q.27  A can complete a certain job in 12 days. B is 100% more efficient than A; B can complete the work alone in 
  (1) 6 days    (2) 6.25 days     (3) 7 days    (4) 7.5 days 
Q.28  A is twice as good as a workman as B and therefore A takes 6 days less than B to finish the work individually. If  A and B are 
working together complete the work in 4 days, the how many days are required by B to complete the work alone? 
(1) 12      (2) 18      (3) 8      (4) 6 
Q.29     A certain job is assigned to a group of men to do in 20 days. But 12 men did not turn up for the job and the remaining men 
did the job in 32 days. The original number of men in the group was 
(1) 32      (2) 36      (3) 42      (4) 40 

PERMUTATION & COMBINATION


Introduction
The  concepts  of  higher  maths  are  normally  perceived  to  be  difficult  and  many  students  leave  out  these  topics  without  even 
attempting it. It will make sense to try and understand the basics of these concepts. The level of questions asked from these 
topics in many cases is low, and hence will be easy to crack! 

www.apparteducation.com   54  info@apparteducation.com  


APPART - Academy of Proficient Professionals for Aptitude Research and Training 

Factorial: Factorial of a natural number is defined as the product of all the consecutive natural numbers from 1 to  that particular 


number.  For  example  factorial  of  5  is  1  2  3  4  5.  ‘Factorial’  word  is  represented  with  a  symbol  ‘!’  or  ‘L’.  For  example 
factorial of 5 is written as L5 or 5 !.  
10 !
Example : ? 
8!
10 ! 10  9  8 !
  10  9  90  
8! 8!
Note : Factorial of zero is 1 (0 ! = 1) 
Basic principle of counting: To make the counting simpler there are two basic principles 
1. Basic principle of multiplication : 
Suppose that there are two ways of reaching the railway station from-your home and from the railway station there are three ways 
of reaching the airport. So the total number of ways of reaching the airport from your home is  
2   3 = 6.  
If a function can be done in x ways and for each of these functions the other function can be done in y ways then both the functions 
together can be done in (x  y) ways.  
Example : There are 10 boys and 8 girls in a school. For the post of class monitor the teacher has to select one boy and one girl. In 
how many ways the class monitor can be selected?  
Solution :For every boy the teacher can select any one of the 8 girls. So for one boy the teacher has 8 choices. So for 10 boys, the 
teacher has 10  8 = 80 choices.  
2. Basic principle of addition :
Suppose you. are at the railway station and you want to go to either the airport or your home. So this can be in 2 ways (if you are 
going home) + 3 ways (if you are going airport) = 5 ways.  
If a function can be done in x ways and the other function can be done in y ways then either of the function can be done in (x + y) 
ways.  
Example 1 : There are 10 boys and 8 girls in a class. For the post of class monitor, the teacher wants to select either a boy or a girl. In 
how many ways can he do this function?  
Solution : He can select one boy out of 10 boys in 10 ways.  
      He can select one girl out of 8 girls in 8 ways.  
      He can select either a boy or a girl in 10 + 8 = 18 ways.  
Note : 1. If all the functions are correlated, then basic principle of multiplication is used  
2.  If all the functions are independent, then basic principle of addition is used.  
Example 2 : How many three digit numbers are there?  
Solution : We know that there are 10 digits 0, 1, 2, 3, 4, 5, 6, 7, 8, 9 
    ‘0’ cannot be at the hundreds place  
    So, 100th place can be filled in 9 ways.  
    Tens place can be filled in 10 ways.  
    Units place can be filled in 10 ways.  
    So the total number, of three digit numbers   9  10  10  900  

www.apparteducation.com   55  info@apparteducation.com  


APPART - Academy of Proficient Professionals for Aptitude Research and Training 

Example 3 : How many three digit numbers are there in which all the digits are distinct?  
Solution : 100th place can be filled in 9 ways.  
    10th place can be filled in 9 ways.  
    Units place can be filled in 8 ways because all the digits should be distinct.  
    So the total number of three digitnumbers in which all digits are distinct        
   9  9  8  648  
Example 4 : There are 5 multiple choice questions in an examination. First three questions have 4 choices each and the remaining 
two questions have 5 choices each. How many sequences of answers are possible?  
Solution : Each one of the first three questions can be solved in 4 ways, and each one of the last two questions can be solved in 5 
ways.  
    So the total number of different sequences of answers are  

    4  4  4  5  5  4 3  5 2  1600  
Example 5 : How many  even numbers less than 1000 can be formed by using the digits 2, 4, 3 and 5, if repetition of the digits is 
allowed?  
Solution : All the numbers of one digit, two digits and three digits are less than 1000. So take these cases one by one  
    1.  Single digit even numbers are 2 and 4  
    2.  Two digit even numbers :  
    Unit’s place can be filled in 2 ways, by 2 and 4 because unit's place digit must be an even number.    
  Ten’s place can be filled in 4 ways.  
    So the total number of two digit even numbers   2  4  8  
    3.  Three digit even numbers  
    Unit’s place can be filled in 2 ways.  
    Ten’s place can be filled in 4 ways.  
    Hundred’s place can be filled in 4 ways  
    So the total number of three digit even numbers   2  4  4  32  
    Total number of three digit even numbers (by using the,digits 2, 4, 3 and 5) less then  
    1000   2  8  32  42  

Permutations
Suppose there are three persons A, B and C contesting for the post of president and vice president of an organization and we have 
to select two persons. We can do it in 3 ! ways. For example, (A, B), (B, C), (A, C) (B, A), (C, B) and (C, A). Here, the first person can be 
the president and the second person can be the vice president, means here we are talking about the order of arrangement.  
The arrangements of a number of things taking some or all of them at a time are called permutations. For example, if there are ‘n’ 
n
number of persons and  we have to select ‘r’  persons at a  time,  then  the  total number of permutations  is denoted by  Pr  or by 

P (n , r ) .  
First person can be selected in ‘n’ ways. Second person can selected in ‘n–1’ ways. Third person can be selected in ‘n – 2’ ways.  

www.apparteducation.com   56  info@apparteducation.com  


APPART - Academy of Proficient Professionals for Aptitude Research and Training 

Similarly, the rth person can be selected in ‘n – (r – 1)’ = ‘(n – r + 1)’ ways.  
   Total number of ways of arranging these ‘r’ selected persons  
   n  (n  1)  (n  2 )  ......(n  r  1)  
n  (n  1) (n  2 )  ......1 n!
     
(n  r ) (n  r  r )  ......1 (n  r ) !

n n!
  Pr   
(n  r ) !
Example 6 : There  are  four  persons  A,  B,  C  and  D  and  at  a  time  we  can  arrange  only  two  persons.  Find  the  total  number  of 
arrangements.  
Solution : Total number of arrangements (permutations) is AB, BA, AC, CA, AD, DA, BC, CB, CD, DC, BD and DB or we can say that out 
4
of 4 persons we have to arrange only 2 at a time, so the total number of permutations is  P2 . 

4 4! 4! 4  3  2
P2     12  
   
( 4  2) ! 2 ! 2!
Example 7 : In the above question, if all the persons are selected r at a time, then how many arrangements are possible?  
Solution :We have to arrange 4 persons, so this can be 

4 4! 4! 4 !
P4     4  3  2  1  24  
( 4  4) ! 0 ! 1
   
Example 8 : There are 4 flags of different colours. How many different signals can be given, by taking any number of flags at a time?  
Solution :Signals can be given either taking al( or some of the flags at a time.  

    Number of signals that can be given by taking 1 flag   4 P1  

    Number of signals that can be given by taking 2 flags   4 P2  

    Number olf signals that can be given by taking 3 flags   4 P3  

    Number of signals that can be given by taking 4 flags   4 P4  

    So the total number of signals  

     4 P1  4 P2  4 P3  4 P4  

4! 4! 4! 4!
         
( 4  1)! ( 4  2) ! ( 4  3) ! ( 4  4)!
Example 9 : Find the number of ways in which 5-boys and 5 girls be seated in a row so that : 
I. All the boys sit together and all the girls sit together. 
II. Boys and girls sit at alternate positions.  
III. No two girls sit together: 
IV. All the girls always sit together.  
V. All the girls are never together. 

www.apparteducation.com   57  info@apparteducation.com  


APPART - Academy of Proficient Professionals for Aptitude Research and Training 

Solution :
I. All the boys can be arranged in 5! ways and all the.girls can be arranged in 5! ways.
Now we have two groups (boys, girls) and these 2 groups can be arranged in 2! ways.
[boys-girls and girls-boys]
So total number of arrangements is 5!  5!  2! = 28,800
II. Boys and girls sit alternately, this can be arranged like this
B G B G B G B G B G or G B G B G B G B G B
In the first case boys can be arranged in 5! and girls can be arranged in 5! ways.
In the second case also, the number of arrangement is same as first case
So the total number of arrangement = 5!  5! + 5!  5! Or

 5 P5  5 P5  5 P5  5 P5

 120  120  120  120


 14400  14400  28800 ways
III. No two girls sit together – In this case __B__B__B__B__B__ there are 6 spaces where a girl can find her seat.

5 girls can be arranged in 6 P5

6!
 6  5  4  3  2  720 ways
( 6  5 )!

5 boys can be arranged in 5 P5  5  4  3  2  1  120 ways

Total number of arrangements  720  120  86400


IV. When all the girls are always together, then treat them as one group. So now we have 5 boys and 1 group of 6
girls and this can be permutated in 6! ways at the same time 5 girls in the group can be permutated in 5!
ways, so total number of required ways is 6!  5! = 720  120 = 86400
V. All the girls are never together
Total number of arrangements of 5 boys and 5 girls is 10!
Number of arrangements in which all the girls are always together
= B1, B2, B3, B4, B5 [All 5 girls]
= 6!  5! = 86400
So number of arrangements in which all the girls are never together = total arrangement – number of arrangements when
girls are always together = 10! – (6! x 5!) = 3,54,2400

Example 10 : Find the number of permutation of the letters of the word FOLDER taking all the letters at a time?  
Solution :Number of letters in the word FOLDER is 6 

    So the number of arrangements   6 P6  6 !  

Alternate method : First  place  can  be  filled  by  any  one  of  the  six  letters.  The  second  place  can  be  filled  by  any  one  ofthe  five 
remaining  letters,  the  third  place  can  be  filled  by  any  one  of  the  four  remaining  letters  and  so  on.  So  the  total  number  of 
arrangements is 6  5  4  3  2  1 = 720. 

www.apparteducation.com   58  info@apparteducation.com  


APPART - Academy of Proficient Professionals for Aptitude Research and Training 

Example 11 : How many four digit numbers greater than 5000  can be formed by using the digits 4, 5, 6 and 7? (Repetition of the 


digits is not allowed.)  

Solution :Total number of arrangements possible is  4 P4  4 !  

    Total number of arrangements by using the digits 5, 6 and 7 is = 3!  
    So the total number of required arrangements is 4! – 3! = 24 – 6 = 18 
Alternative method :Thousand's place can be filled in 3 ways. 
      Hundred's place can be filled in 3 ways.  
      Ten's place can be filled in 2 ways.  
      Unit's place can be filled in 1 ways.  
So total number of arrangements   3  3  2  1  18  
Example 12 : In Q.11, find the number of four digit numbers that can be formed if the repetition of digits is allowed. 
Solution :If the repetition is allowed then the total number of arrangements is 4  4  4  4 = 256 ways 
Because on the first place any one of the four number can come, similarly on the 2 nd, 3rd and 4 th place also. 
Total number of arrangements beginning with 4 is 4  4  4 = 64 
So, total number of required arrangements = 256 – 64 = 192 
Alternative method : Thousand's place can be filled in 3 ways Hundred's place can be filled in 4 ways. Ten's place can be filled in 4 
ways. Unit's place can be filled in 4 ways. So the total number of arrangements = 3  4  4  4 = 192 
Combinations
Suppose  three persons A,  B and C are  contesting  for  the post of president and vice  president of an organization and we have  to 
select two persons. We can select either (a, b) or (b, c) or (a, c) = 3 ways because here we are talking about the selection, not about 
the order. Whether ‘a’ is a president or 'b' is a vice president or vice-versa, doesn't matter.  
Suppose  there  are  10  persons  in  class  and  we  have  to  select  any  3  persons  at  a  point  regardless  of  the  order,  it  is  a  case  of 
combination.  
If there are n number of things and we have to select some or all of them it is called combinations.  

n n!
If out of n things we have to select r things (1  r  n), then the number of combinations is denoted by  C r   
( n  r )! r !

n n!
We already know that the number of arrangements of 'r' things out of the 'n' things is given by  Pr   
(n  r )!
Combination does not deal with the arrangements of the selected things. 
    ‘r’ selected things can be arranged in r ! ways. 
 
 (r ! )  n C r  n Pr  
n
n Pr n!
 Cr    
r ! r ! (n  r ) !

Difference between permutations and combinations

www.apparteducation.com   59  info@apparteducation.com  


APPART - Academy of Proficient Professionals for Aptitude Research and Training 

Suppose that there are five persons A, B, C, D and E and we have to choose two persons at a time then in 

Permutation: Number of required ways   5 P2  

5! 5!
   5  4  20 ! 
   
( 5  2 )!   5!
Combinations: Number of required ways   5C 2  

5! 5! 5 4
     10  
   
2 !( 5  2) ! 2 ! 3 ! 2
So it is clear that in permutations (rearrangement) order matters but in combinations (selections) s) order does not matter. 
Example 13 : In a class there 5 boys and 6 girls. How many different committees of 3 boys and 2 girls can be formed? 
5
Solution :Out of 5 boys we have to select 3 boys, this can be done in  C 3 ways. 
6
    Out of 6 girls we have to select 2 girls, this can be done in  C 2 ways. 

    So, selection of 3 boys and 2 girls can be done in   C   C  ways 
5
3
6
2

 5!   6! 
[Basic rule of multiplication]     
  
 3 ! ( 5  3) !   2 !( 6  2 ) ! 
 5 4   6 5 
    10  15  150 ways 
 2   2 
Example 14 : If there are 10 persons in a party, and each person shake hands with all the persons in the party, then how many hand 
shakes took place in the party?  
Solution : It is very obvious that when two persons shake hands, it is counted as one handshake. So we can    
  say that there are 10 hands and every combination of 2 hands will gives us one handshake. So the    
  number of handshakes 
10 ! 10  9  8 !
 10C 2    45  
   
2 ! (10  2 ) !   2!  8!
Example 15 : For the post of Maths faculty in APART there are 6 vacant seats. Exactly 2 seats are reserved for MBA's. There are 10 
applicants out of which 4 are MBA's. In how many ways the selection can be made? 
Solution :There are 4 MBA's and 6 other candidates. 
    So we have to select 2 candidates out of the 4 MBA's and the rest 4 candidates out of 6 other candidates. 

    So the total number of ways of selection    C   C   
4
2
6
4

 4!   6! 
    
2 !  ( 4  2 ) !   4 ! ( 6  4 ) ! 
       
 4  3  2!   6  5  4! 
    
  
     2  1 2!   4!  2  1 
     6  15  90 ways 
Example 16 : There are 10 points out of which no three are collinear. How many straight lines can be formed using these 10 points? 
Solution :By joining any two points we will get one line. So the total number of lines formed 
10  9  8 ! 10  9  8 !
 10C 2    45  
   
2  (10  2 )! 2  8!

www.apparteducation.com   60  info@apparteducation.com  


APPART - Academy of Proficient Professionals for Aptitude Research and Training 

Example 17 : Find the number of diagonals that can be drawn by joining the vertices of a decagon. 
Solution :In decagon there are 10 vertices and by joining any two vertices we will get one line. 
    So in a decagon total number of lines formed 
10 ! 10  9  8 !
 10C 2    45  
2 ! (10  2 ) ! 2 ! 8 !
   
    But out of these 45 lines, 10 lines will be the sides of the decagon. So total number of diagonals =    
  45 – 10 = 35 
Example 18 : In the above question how many triangles can be formed? 
Solution :We know that in a triangle there are three vertices and by joining any three points we will get a triangle. So number of 
triangles formed 
10  9  8  7 ! 10  9  8  7 !
 10C 3    120  
   
3! (10  3)! 3 ! 7 !
Example 19 : There are 5 boys and 6 girls. A committee of 4 is to be selected so that it must consist at least one boy and at least one 
girl? 
Solution : The different possibilities are 
    I.  1 boy and 3 girls  
    II.  2 boys and 2 girls  
    III.  3 boys and 1 girl 

In the first possibility total number of combinations is  5 C 1  6C 3  

In the second possibility total number of combinations is  5 C 2  6C 2  

In the third possibility total number of combinations is  5 C 3  6C 1  

So the total number of combinations are  5 C 1  6C 3  5C 2  6C 2  6C 2  5C 3  6C1  310  

Circular combination
If n persons are seated around a circular table then they can be arranged in (n – 1) ! ways. 
For example:  If three persons are there they can be arranged in (3 – 1) ! 
      = 2 ! ways. [We fix the position of 1 person and then arrange the remaining (n – 1) persons] 

 
PROBABILITY
Suppose a magician approaches you and says that he has a dice and if, he throws that dice number greater than 5 comes on the top, 
he will give you Rs. 20 otherwise you will have to give Rs. 10 to him. What will you do? Here is an application of probability which 
deals with uncertainties. It has nothing to do with actual happenings. It just talks about the possibilities. 
To know about the probability in a better way just take the above example. 
When a dice is thrown then on the top either 1, 2, 3, 4,5 or 6 can come. So we can say there are a total of six possibilities. Out of 
these 6 numbers, greater than 5 is only one number that is 6. So in your favour there is only 1 number that is 6 and against our (or 

www.apparteducation.com   61  info@apparteducation.com  


APPART - Academy of Proficient Professionals for Aptitude Research and Training 

our opponent) favour there are 5 numbers 1, 2, 3, 4 and 5. This indicates' that your chance of losing is five times than your chance of 
winning. It means this game is not in your favour. Probability is defined as 
Number of favourable events
Probability of a event    
Total number of possible events
Like in the example given above 
1
Probability of your winning    
6
5
Probability of your losing    
6
Possible events (sample space) : It means all those events which can occur in that scenario. ,For example: If we have one dice and 
one coin then the possible outcomes are H1, H2, H3, H4, H5, H6, T1, T2, T3, T4, T5, T6 
Note : Probability of an event cannot be less than 0 and at the same time it cannot be more than 1. 
Addition rule
Events : Each possible. outcome is called an event. Like the events of throwing a dice are 1, 2, 3, 4, 5 and 6. 
Mutually exclusive events :  When  a  particular  event  occurs  and  the  other  particular  event  cannot  occur,  then  they  are  called 
mutually exclusive events. 
For example : In the experiment of throwing a dice the event that a possible outcome is an odd number and the event that possible 
outcome is an even number, are mutually exclusive (Because there iss no number which is odd as well as even). Here, the probability 
of such event is given by, P(E) = P(A) + P(B) 
where 
P(E) = Probability of occurence of such mutually exclusive event. 
P(A) = Probability of occurence of event A P(B) = Probability of occurence of event B 
Non mutually exclusive events: When a particular event occurs and a particular event may also occur and vice versa or you can say 
that both events can occur simultaneously, then they are called non mutually exclusive events.  
For example : In  the  experiment of  throwing a dice  the  event  that  a possible  outcome  is  an  odd  and  the  event  that  a  possible 
outcome is a prime number are not mutually exclusive, because there are certain numbers (like 3 and 5) which are both odd as well 
as prime, so the probability of such an event is given by  
P(E) = P(A) + P(B) - P(C) where 
P(E) = Probability of occurrence of any or both of A and B 
P(A) Probability of occurrence of event A P(B) = Probability of occurrence of event B 
P(C) = Probability of occurrence of event C,  
which is an intersection of A and B. (Intersection of A and B means that outcomes in which the number is odd as well as prime. For 
example, the numbers 3 and 5 which are both prime as well as odd). 
Example 20 : In a single throw of a fair dice what is the probability that the number the appearing on the top face of the dice is more 
than 2? 
Solution :In a dice there are 6 faces numbered 1, 2, 3, 4, 5 and 6. 
    So, the total number of possible events are 1, 2, 3, 4,5 and 6 = 6 
    and the total number of favourable events are 3, 4, 5 and 6 = 4 
www.apparteducation.com   62  info@apparteducation.com  
APPART - Academy of Proficient Professionals for Aptitude Research and Training 

4 2
    So, the required probability is    
6 3
Example 21 : If two fair dice are thrown simultaneously, then what is the probability that the sum of the numbers appearing on the 
top faces of the dice is less than 4? 
Solution :Total number of possible events = (1, 1), (1, 2), (1, 3), (1, 4), (1, 5), (1, 6), (2, 1), (2, 2) … and so on. There will be 6  6 = 36 
possible events.  
    Number of favourable events = (1, 1), (1, 2) and (2, 1) = 3 events 
3 1
    So, the required probability     
36 12
Example 22 : If out of the first 20 natural numbers Mr. X selects a number at random, then what is the probability that this number 
will be a multiple of 4? 
Solution :Total number of possible events = 1, 2, 3,..., 20 = 20 such numbers 
    Total number of favourable events = 4, 8, 12, 16 and 20 = 5 such numbers 
5 1
    So, the required probability     
20 4
Example 23 : In the example 22, what is the probability that this number will be a multiple of 4 or 7? 
Solution :Total number of possible events = 1, 2 ... 20 = 20 such numbers 
    Numbers divisible by 4 = 4, 8, 12, 16, 20 = 5 such numbers 
    Number divisible by 7 = 7 and 14 = 2 such numbers 
    Since from 1 to 20 there is no number which is divisible by both 4 and 7.  
    If is a case of mutually exclusive events. 
    So number of possible outcomes = 5 + 2 = 7 
7
    So, the required probability is    
20
Example 24 : In the example 22, what is the probability that the selected number is divisible by 2 and 4? 
Solution :The total number of possible events = 20 such numbers 
    Number divisible by 2 and 4 means the number should be divisible by 4  
    (LCM of 2 and 4 is 4) = 4, 8, 12, 16, 20 = 5 such numbers 
5 1
    So, the required probability is    
20 4
Example 25 : In the example 22, what is the probability that this number is divisible by 2 or 4? 
Solution : The total number of possible outcomes = 20 in number 
    Number divisible by 2 = 2, 4, 6, 8,10,12,14,16,18, 20 = 10 such numbers 
    Number divisible by 4 = 4, 8, 12, 16 and 20 = 5 such numbers 
    There are certain numbers which are divisible by both. 2 and 4, so it is case of non mutually    
  exclusive events. 
    Number divisible by both 2 and 4 are 4, 8, 12, 16 and 20 = 5 such number 
So, the required probability 
  = P(A) + P(B) – P(C) 

www.apparteducation.com   63  info@apparteducation.com  


APPART - Academy of Proficient Professionals for Aptitude Research and Training 

10 5 5 10 1
      
  20 20 20 20 2

Class Excercise
Q.1   In how many ways a team of 5 members out of 9 can be formed such that Sachin is always included? 
(1) 126      (2) 70      (3) 1680     (4) 140 
Q.2    In how many ways a photograph of 5 members out of 9 can be taken such that Sachin is always included? 
(1) 120      (2) 3024     (3) 8400     (4) 15120  
Q.3    Find the number of ways in which the letters of the word BIHAR can be rearranged. 
(1) 99      (2)129      (3) 119      (4) 125 
Q.4  Find the number of ways in which the letters of the word AMERICA can be arranged. 
(1) 2520     (2) 5040     (3) 1250     (4) 2500 
Q.5  Find the number of ways in which the letters of the word CALCUTTA can be arranged. 
(1) 3000     (2) 5009     (3) 5029     (4) 5040 
Q.6  In how many ways can you arrange the letters of the word AKSHAY such that vowels do not start the words? 
6! 6!
(1)  1   (2)  2   (3) 240      (4) 5! 
2!  
2!  
Q.7  How many distinct 4 letter words can be formed by using the letters a, b, c and d? (Repetition of the-letters is allowed).  
(1) 296      (2) 346      (3) 440      (4) 256 
Q.8  How  many  numbers  greater  than  4000  can  be  made  by  using  the  digits  2,  3,  4  and  5?  (Repetition  of  the  digits  is  not 
allowed). 
(1) 12      (2) 14      (3) 20      (4) 24 
Q.9  How many numbers greater than 4000 can be made by using the digits 2, 3, 4 and 5? (Repetition of digits is allowed). 
(1) 120      (2) 128      (3) 138      (4) 130 
Q.10  A bag contains 6 white balls and 4 red balls. Three balls are drawn one by one with replacement. What is the probability 
that all the 3 balls are red? 
8 1 1 1
(1)      (2)       (3)       (4)   
125 20 30 120
Q.11  In the above question, if 3 balls are drawn one by one with replacement, then what is the probability that 2 balls are white 
and 1 ball is red? 
54 1 1 1
(1)      (2)        (3)        (4)   
125 4 3 2
Q.12  In question 10, if the balls are drawn without replacement, what is the probability that 2 balls are red and 1 ball is white? 
(1) 0.1      (2) 0.2      (3) 0.3      (4) 0.4 
1
Q.13  The probability that A will pass the examination is  and the probability that B will pass the examination is 1/2. What is the 
3
probability that both A and B will pass the examination? 
1 2 3
(1)        (2) 1      (3)        (4)   
6 3 2

www.apparteducation.com   64  info@apparteducation.com  


APPART - Academy of Proficient Professionals for Aptitude Research and Training 

Q.14  In Q. No. 13, what is the probability that only one person [either A or B] will pass the examination? 
1 1 2
(1) 1      (2)        (3)        (4) 
2 3 3 
 
Q.15  In Q. No. 13, what is the probability that at least one person will pass the examination? 
1 1 2
(1) 1      (2)        (3)        (4)   
2 3 3
Q.16  In Q. No. 13, what is the probability that no one will pass the examination? 
1 1 1
(1) 1      (2)        (3)        (4)   
2 4 3
Q.17  Two cards are drawn together from a pack of 52 cards at random. What is the probability that both the cards are spades? 
4 13 26 8
C2 C2 C2 C2
(1)  52
  (2)  52
    (3)  52
    (4)  52
 
C2   C2 C2 C2
Q.18  In question number 17, what is the probability that both the cards are kings? 
8 13 26 4
C2 C2 C2 C2
(1)  52
    (2)  52
    (3)  52
  (4)  52
 
C2 C2 C2   C2
Q.19  In question number 17, what is the probability that one card is a spade and one card is a heart? 
13
C 1  13C 2 13
C 1  28C 1 13 13
13
C 1  13C 1
(1)  52
  (2)  52
  (3)       (4)  52
 
C2 C2 52 52 C2
Q.20  In question number 17, what is the probability that exactly one card is a king? 
4
52
C1 4 C 1  48C 1 1
(1)  52
  (2)  58
    (3)  52
   (4)   
C2   C2 C2 2
Q.21  The number of five digit telephone numbers having at least one of their digits repeated is   
(1) 90000    (2) 100000    (3) 30240    (4) 62784 
Q.22  Find the number of words of 5 letters such that each can be formed with the letters of the word   "CHROMATE", if each 
letter may be repeated in any arrangement. (These words need not have meaning) 
    (1) 262144    (2) 4096     (3) 1024     (4) 32768 
Q.23  The letters of the word ENTRANCE are arranged in all possible ways. The number of arrangements having the E's together 
and the N's together is 
    (1) 7!      (2) 6!      (3) 8!      (4) 9! 
Q.24  A picnic party of four persons is to be selected from 8 girls and 3 boys so as to include at least one boy. The possible number 
of ways are 
    (1) 168      (2) 84      (3) 70      (4) 260 
Q.25  4 men and 3 women are to be seated in a row so that the women occupy the even places. How many such arrangements 
are possible? 
    (1) 7!      (2) 144      (3) 30      (4) 6! 
Q.26  In how many ways six boys and six girls can sit alternately along a circle? 
    (1) 5! × 5!    (2) 5! × 6!    (3) 5! × 5! × 2    (4) 5! × 6! × 2 
Q.27  There are four letters and four addressed envelopes. What is the probability that all letters are not placed in the right 
envelopes? 

www.apparteducation.com   65  info@apparteducation.com  


APPART - Academy of Proficient Professionals for Aptitude Research and Training 

    (1) 1/24      (2) 1/6      (3) 23/24    (4) 5/24 


Q.28  Three unbiased coins are tossed. What is the probability of getting at least two heads? 
    (1) 4/8      (2) 3/8      (3) 6/8      (4) 2/8 
Q.29  An urn contains 9 red, 7 white and 4 black balls. If two balls are drawn at random, find the probability that exactly one ball 
is white.  
    (1) 86/95    (2) 36/190    (3) 44/93    (4) 91/190 
Q.30  A card is drawn from an ordinary pack of 52 cards and a gambler bets that, it is a spade or an ace. What are the odds 
against hiswinning this bet? 
(1) 4 : 9      (2) 7 : 6      (3) 9 : 4      (4) 8 : 5 
Q.31.  The total number of ways of answering 5 objective type questions, each question having 4 choices is 
    (1) 256      (2) 512      (3) 1024     (4) 4096 
Q.32.  A "necklace" is a circular string with several beads on it. It is allowed to rotate a necklace but not to turn it over. How many 
different necklaces can be made using 13 different beads? 
    (1) 13!      (2) 13!/2     (3) 12!      (4) 12!/2 
Q.33.  How many parallelograms are formed by a set of 4 parallel lines intersecting another set of  7 parallel lines? 
    (1) 121      (2) 139      (3) 115      (4) 126 
Q.34.  The number of arrangements of the letters of the word SALOON, if the two O's do not come  together, is 
    (1) 360      (2) 720      (3) 240      (4) 120 
Q.35.  In how many ways letters of the word COLLEGE can be arranged such that vowels are always together? 
(1) 360      (2) 180      (3) 720      (4) None of these 

www.apparteducation.com   66  info@apparteducation.com  


APPART - Academy of Proficient Professionals for Aptitude Research and Training 

ANSWER KEY
NUMBER SYSTEM RATION PROPORTION, AVERAGES, MIXTURES & ALLIGATION
 
1 3  12 4  23 3  1 3  18 4  35 4 
 
2 2  13 2  24 1  2 4  19 1  36 3 
 
3 2  14 4  25 2  3 1  20 2  37 2 
 
4 1  15 3  26 3  4 2  21 2  38 4 
 
5 4  16 3  27 3  5 1  22 3  39 4 
 
6 4  17 3  28 3  6 2  23 2  40 2 
 
7 4  18 2  29 2  7 2  24 2  41 4 
 
8 4  19 1  30 2  8 3  25 4  42 3 
 
9 4  20 4  31 4  9 4  26 2  43 4 
 
10 2  21 2  32 2  10 1  27 3  44 3 
 
11 3  22 1  33 2  11 4  28 1  45 3 
 
PERCENTAGE, PROFIT & LOSS, PARTNERSHIPS 12 3  29 4  46 3 
 
1 4  14 4  27 2  13 4  30 2  47 5 
 
2 4  15 2  28 1  14 4  31 3  48 2 
 
3 1  16 2  29 4  15 1  32 2 
   
4 4  17 2  30 3  16 2  33 4 
   
5 2  18 4  31 3  17 2  34 2 
   
6 3  19 3  32 3  TIME SPEED DISTANCE
 
7 2  20 4  33 4  1 3  12 3  23 1 
 
8 2  21 3  34 4  2 1  13 2  24 4 
 
9 2  22 3  35 2  3 3  14 1  25 2 
 
10 3  23 3  36 4  4 2  15 1  26 2 
 
11 2  24 4  37 2  5 2  16 4  27 4 
 
12 3  25 3  38 1  6 2  17 2  28 2 
 
13 1  26 4  39 4  7 1  18 1  29 3 
 
SIMPLE & COMPOUND INTEREST 8 1  19 3  30 2 
 
1 4  6 3  11 4  9 4  20 1  31 2 
 
2 2  7 2  12 2  10 2  21 1  32 1 
 
3 1  8 2  13 4  11 2  22 1  33 4 
 
4 2  9 3  14 3  PERMUTATIONS & COMBINATION, PROBABILITY
 
5 2  10 3  1 2  13 1  25 2 
   
TIME AND WORK 2 3  14 2  26 2 
 
1 1  11 4  21 4  3 3  15 4  27 3 
 
2 3  12 4  22 2  4 1  16 4  28 1 
 
3 2  13 4  23 2  5 4  17 2  29 4 
 
4 4  14 2  24 4  6 3  18 4  30 3 
 
5 2  15 3  25 1  7 4  19 4  31 3 
 
6 4  16 1  26 1  8 1  20 3  32 4 
 
7 3  17 2  27 1  9 2  21 4  33 4 
 
8 2  18 1  28 1  10 1  22 4  34 3 
 
9 1  19 4  29 1  11 1  23 2  35 1 
 
10 3  20 1  12 3  24 4 
     
www.apparteducation.com   67  info@apparteducation.com  
APPART - Academy of Proficient Professionals for Aptitude Research and Training 

LOGICAL REASONING
THEORY BOOKLET
An Orientation

 This section of booklet contains all the concept of LR which are relevant from the campus recruitment test point
of view along with some of the competitive exams (GRE/GMAT) point of view.
 All the major topics are covered & their concepts are discussed along with solved examples & explanations
 In case of any query, feel free to write us at info@apparteducation.com

www.apparteducation.com   68  info@apparteducation.com  


APPART - Academy of Proficient Professionals for Aptitude Research and Training 

CODING – DECODING
Coding is the process of encrypting a word/message into some not–so easily recognizable form which does not make much sense 
until decoded properly into the original message. There is no fixed rule for coding or decoding a message because there are infinite 
numbers of logics which can be applied to code any given word. Candidates must try and solve as many different types of coding–
decoding type questions as possible to get a good deal of coding logics and ideas. 
Having  said  that,  however,  there  are  certainly  some  logics  seen  frequently  when  hundreds  of  problems  are  solved.  They  are 
summarized below –  
1.   Logic of Shifting – Letters in the coded word are totally different than those in the original word. Hence, we can conclude that 
letters are ‘shifted’. 
2.   Logic of Shuffling – Letters in the coded word are collectively same as the letters in the original word. Hence, we can conclude 
that letters are ‘shuffled’ among themselves. 
CLASS ASSIGNMENT 
1. If CRICKET is coded as DQJBLDU, what is the code for FOOTBALL? 
  (A) GPPUCBMM  (B) GNPSCBKM    (C) GNPCSZMKB    (D) GNPSCZMK 
2. If NICE is coded as 15131221, what is the code for CHART? 
  (A) 3811820  (B) 412103445    (C) 413203445    (D) 381019 
3. If ROSE is coded as 7621 and STAR is coded as 2357, what will be the code for ROASTED? 
  (A) 7652418  (B) 7652312    (C) 7652138    (D) 7652319 
4. If COW is coded as 31523, then the code for HORSE is? 
  (A) 81815195  (B) 81518195    (C) 81517195    (D) 81516195 
5. If TIGER is coded as 69 and LION as 60, then what is the code for LEOPARD? 
  (A) 65    (B) 81      (C) 79      (D) 73 
6. If TELEPHONE is coded TEENLOEHP, then the code for TELECOM will be? 
  (A) TMELOEC  (B) TMEOLCE    (C) TMELOCE    (D) TMEOLEC 
7. In a  certain code  language,  ‘dom pul  ta’ means  ‘bring hot food’,  ‘pul  tir sop’  means  ‘food  is  good’ and  `tak da sop’  means 
‘good bright boy’. Which of the following does mean ‘is’ in that language? 
  (A)dom    (B) pul      (C) to      (D) tir      (E) Cannot be determined 
8. In a certain code language, ‘XZM’ means ‘He is bright.’, ‘TCZO’ means ‘Every lawn is green.’ and ‘OQCN’ means ‘Every wall was 
green’. Which of the following means ‘Every lawn is bright’ in that language? 
  (A) ZTOM    (B) CXZT     (C) XOTZ     (D) CZOT    (E) Cannot be determined 
9. If the code for FRONT is 39347, for CHALK is 61867, then what is the code for REVERSE? 
  (A) 9252983  (B) 9454904    (C) 9151981    (D) 9454984 
10. The gate of the top security area of the Lambda Reactor requires any personnel to know a particular code in order to enter 
inside. One day Madan, Mohan and Munna meet the gate when the gatekeeper asks them to tell the codes one by one. The 
following is the conversation between the three and the gatekeeper: 
  Gatekeeper: Twelve. 
  Madan: Six. (and the gatekeeper lets him in.) 
  Gatekeeper: Six. 
www.apparteducation.com   69  info@apparteducation.com  
APPART - Academy of Proficient Professionals for Aptitude Research and Training 

  Mohan: Three. (and the gatekeeper lets him in too.) 
  Gatekeeper: Ten. 
  Munna: Five. (and the gatekeeper does not allow him to enter at all.) 
  What should Munna have said to get inside? 
(A) One    (B) two      (C) three     (D) four      (E) Cannot be determined 
11. If cloud is called white, white is called rain, rain is called green, green is called air, air is called blue and blue is called water, 
where will the birds fly? 
(A) Air    (B) Cloud    (C) Blue      (D) Rain 
12. If FINGER is coded as SGJRNL, what is the code for GROUND? 
  (A) TLSNJE   (B) TGNRSP    (C) EPXSWM    (D) EPNRWM 
13. The code for ORANGE is 3 and the code for BLACK is 4. What is the code for PINK? 
(A) 1    (B) 2      (C) 3      (D) 4 
14. In the city of Lateral Thoughts, the price of items follows a certain pattern. The cost of a CANON machine is Rs. 30 whereas 
the price of an HCL machine is Rs. 12. What is the price of a LENOVO machine? 
(A) Rs. 42    (B) Rs. 48    (C) Rs. 35    (D) Rs. 52 
15. CAN is coded as 414. What will be the code for FAR? 
(A) 618    (B) 817      (C) 871      (D) 718 

NUMBER SERIES
Number Series:It is a  very important topic from aptitude point of view. In such type of questions, a series of numbers is given in 
which you may have to find out the next number or you may have to find the missing number or may even have to spot the wrong 
number.
The number series can be of various types: 
 Arithmetic Series: In this type of series, the difference between the consecutive terms of the series is constant. 
e.g.  5, 13, 21, 29, 37, 45 …    Here the constant difference is 8. 
  203, 226, 249, 272, 295, 312 …  Here the constant difference is 23. 
 Geometric Series: In this type of series, the ratio between the consecutive terms of the series is constant. 
e.g.  4, 12, 36, 108, 324 …    Here the constant ratio is 3. 
  192, –288, 432, –648, 972 …   Here the constant ratio is –1.5. 
 Miscellaneous Series: There can be many series in which the difference or ratios are not constant. In such cases, one may 
have to look at different patterns in the series. 
a) Differences are in Arithmetic Series:
e.g.      3,  5,  10,  18,  29,  43,  60,  80 
 

    +2  +5  +8  +11  +14  +17  +20 


The differences form an Arithmetic Series in which the constant difference is 3. 
b) Differences are in Geometric Series:
e.g.      5,  7,  3,  11,  –5,  27,  –37,  91 
 

    +2  –4  +8  –16  +32  –64  +128 


The differences form a Geometric Series in which the constant ratio is –2. 
www.apparteducation.com   70  info@apparteducation.com  
APPART - Academy of Proficient Professionals for Aptitude Research and Training 

c) Combination of Arithmetic and Geometric Series:


e.g.      1,  3,  13,  51,  205, 
 

    ×4–1  ×4+1  ×4–1  ×4+1 


e.g.      4,  5,  12,  39,  160, 
 

    +1×1  +2×2  +3×3  +4×4 


d) Factorial Series: The numbers in this series oscillate around factorials of numbers. 
e.g.      2,  4,  9,  28,  125,  726,  5047 
 

    1!+1  2!+2  3!+3  4!+4  5!+5  6!+6  7!+7 


e) Others: There  can  be  many  other  series  which  can  be  solved  by  smart  observations.  Such  series  will  be  discussed  in  the 
questions
CLASS ASSIGNMENT
DIRECTIONS for questions 1 to 6: Which number should come in place of the ‘?’ in the following questions? 
1. 2, 6, 21, 88, 445,  ? … 
  (A) 2676  (B) 2686  (C) 2767  (D) 2666 
2. 5, 6, 9, 18, 45,  ? … 
  (A) 116  (B) 126  (C) 136  (D) 146 
3. 7, 24, 75, 228,  ? … 
  (A) 768  (B) 678  (C) 687  (D) 876 
4. 3, 13, 51, 205,  ? … 
  (A) 730  (B) 820  (C) 821  (D) 819 
5. 2, 5, 15, 60, 280,  ? … 
  (A) 1735  (B) 1235  (C) 1275  (D) 1375 
6. 3, 3, 9, 15, 33, 63,  ? … 
  (A) 127  (B) 126  (C) 129  (D) 132 
DIRECTIONS for questions 7 to 8: Which number should come in place of the ‘?’ in the following diagrams? 
7.    
35 27  99  33 
  6
56  ? 
 
  28  76  26  31 
  (A) 31  (B) 71  (C) 50  (D) 28 
 
8.  

  11  281 
 

  2  1406 
 
  (A) 88  (B) 76  (C) 54  (D) 56 

www.apparteducation.com   71  info@apparteducation.com  


APPART - Academy of Proficient Professionals for Aptitude Research and Training 

DIRECTIONS for questions 9 to 12: Find the wrong term in each of the following series. 


9. 4, 6, 14, 16, 24, 34 … 
  (A) 6  (B) 14  (C) 16  (D) 24 
10. 2, 5, 12, 17, 26, 37 … 
  (A) 37  (B) 26  (C) 12  (D) 5 
11. 4, 19, 79, 339, 1279, 5119 … 
  (A) 19  (B) 79  (C) 339  (D) 5119 
12. 5 , 2, –3, –21, –75, –237, –723 … 
  (A) 2  (B) –21  (C) –75  (D) 4 
DIRECTIONS for questions 13 to 15:  In  each of  the following question, five  series  of numbers are given; four of which  follow the 
same pattern and one is different. Find the different series. 
13. (A) 1, 4, 5, 17  (B) 2, 8, 6, 18  (C) 4, 16, 8, 20  (D) 3, 12, 7, 19  (E) 6, 24, 12, 22 
14. (A) 0. 5, 1.5, 1, 2.5  (B) 0.6, 1.2, 1, 4  (C) 1, 3, 2, 5  (D) 2, 6, 4, 10  (E) 0.25, 0.75, 0.5, 1.25 
15. (A) 3, 2, 6, 5  (B) 4, 3, 12, 6  (C) 6, 5, 30, 8  (D) 2, 1, 4, 4  (E) 5, 4, 20, 7 

ALPHABETIC SERIES
Alphabetic Series: Here, a series of letters is given in which you may have to find out the next alphabet, the missing alphabet or have 
to spot the wrong alphabet. Solving a lot questions can give a good idea about the different types of series.
An important thing to remember would be the numerical/place value of every letter. 
i.e. A=1, B=2, C=3, D=4, …, X=24, Y=25, Z=26. 
DIRECTIONS for questions 1 to 7: Find the next term of the following series. 
1. B, D, G, I, L, N, ? 
  (A) O  (B) Q  (C) S  (D) U 
2. X, U, S, P, N, K, I, ? 
  (A) J  (B) K  (C) M  (D) F 
3. A, C, E, G, I, ? 
  (A) H  (B) J  (C) K  (D) L 
4. D, I, L, Q, T, Y, B, G, ? 
  (A) H  (B) I  (C) O  (D) J 
5. Y, W, U, S, Q, ? 
  (A) P  (B) O  (C) M  (D) B 
6. D, F, I, M, R, ? 
  (A) S  (B) U  (C) T  (D) X 
7. LXF, MTJ, NPN, OLR, ? 
  (A) PHV  (B) PPV  (C) PIU  (D) PJW 
DIRECTIONS for questions 8 to 11:Three of the following four are alike in a certain way. Find the one which is different. 
8. (A) GJL  (B) MPS  (C) SVX  (D) LOQ 
9. (A) GIK  (B) LNP  (C) YAC  (D) SUV 
10. (A) SUTR  (B) TVXZ  (C) UWYA  (D) WYAC 
11. (A) TREAT  (B) TEARS  (C) THINK  (D) TRAIT 

www.apparteducation.com   72  info@apparteducation.com  


APPART - Academy of Proficient Professionals for Aptitude Research and Training 

DIRECTIONS for questions 12 and 13: In each of the following questions, one term from different positions of a series are removed. 


Choose the answer option in which the missing terms are written in the same order as in series. 
12. abca_bcaab_ca_bbc_a 
  (A)ccaa  (B) bbaa  (C) abac  (D) abba 
13. _bbca_bcca_ac_a_cb 
  (A)abcba  (B) acbab  (C) bacab  (D) bcaab 
DIRECTIONS for questions 14 and 15:In the following questions, the pair of group of letters given before ‘::’, bear certain relation 
between them. From the answer options, find the group of letters which bears the same relation with the group of letters 
given after ‘::’. 
14. ACE : FHJ :: OQS : ? 
  (A) PRT  (B) RTU  (C) TVX  (D) UWY 
15. AFHO : GBDM :: CHFM : ? 
(A) GBLD  (B) IDBK  (C) GPLD  (D) GBDM 
 
BLOOD RELATIONS
A blood  relation  is  one  related by  blood or origin, especially on  sharing an ancestor with  another.  Following are the well  known 
blood relations: 
Relation Gender Definition
By Ancestors
Father  M  Whom one is born to. Also called a parent. 
Mother  F  Whom one is born to. Also called a parent. 
Grand Father  M  Father of one’s parent. 
Grand Mother  F  Mother of one’s parent. 
Brother  M  Son of one’s parents. Also called a sibling. 
Sister  F  Daughter of one’s parents. Also called a sibling. 
Uncle (Paternal/Maternal)  M  Brother of one’s parent. OR Husband of one’s aunt. 
Aunt (Paternal/Maternal)  F  Sister of one’s parent. OR Wife of one’s uncle. 
Cousin  M or F  Child of one’s uncle or aunt. 
Nephew  M  Son of one’s sibling. 
Niece  F  Daughter of one’s sibling. 
Son  M  One’s child. 
Daughter  F  One’s child. 
By Marriage
Husband  M  Who one marries. Also called a spouse. 
Wife  F  Who one marries. Also called a spouse. 
Father–in–law  M  Father of one’s spouse. 
Mother–in–law  F  Mother of one’s spouse. 
Brother–in–law  M  Brother of one’s spouse. OR Husband of one’s sister. 
Sister–in–law  F  Sister of one’s spouse. OR Wife of one’s brother. 
Son–in–law  M  Husband of one’s daughter. 
Daughter–in–law  F  Wife of one’s son. 

www.apparteducation.com   73  info@apparteducation.com  


APPART - Academy of Proficient Professionals for Aptitude Research and Training 

A candidate must know the definition of each of the above mentioned relations in order to be able to solve questions based upon 
blood relations. 
These  types  of  questions  can  easily  be  solved  by  making  use  of  a  “family  tree”.  A  “family  tree”  is  basically  a  diagrammatic 
representation of the relations following a hierarchal arrangement. 
Following is the simplified procedure for drawing and reading a family tree: 
1. Choose convention for the genders. e.g. Circles for M and Squares for F. 
2. To begin the tree, mark “your” position or the position of the person for which maximum information is given. 
3. Mark all the other relations with respect to your position with proper conventions and hierarchy. 
4. All the relations who are at the immediate higher level than you are; are parents, father/mother–in–law or uncles/aunts. 
5. All the relations at the same level as you are; are either brothers/sisters, brothers/sisters–in–law or cousins. 
6. All the relations at the immediate lower level than you are; are sons/daughters, sons/daughters–in–law, nephews/niece. 
a.   Mark Husband Wife relation by  
 
b.  Mark Siblings relation as  
 
c.   Mark Son, daughter  relation as 
 
d.   Mark cousins as  
  For eg: A is husband of B, B is mother of C, C is sister of D and D is male cousin of E. 
This will be represented as— 
 A                  B 
 
 

      C                    D               E 
CLASS ASSIGNMENT
1. Ashish said to Himani, "Your only brother’s son is my wife’s brother". How is Himani related to wife of Ashish? 
  (A) Sister  (B) Aunt  (C) Mother  (D) Daughter 
2. Pointing towards a person in a photograph, Alka said, "He is the only son of the father of my sister’s brother". How is that 
person related to Alka? 
  (A) Uncle  (B) Mother  (C) Father  (D) Brother 
3. A is the sister of B, C is the father of B, D is the wife of C and E is the father of D. How is E related to B? 
  (A) Uncle  (B) Grandmother   (C) Father  (D) Grandfather 
4. Pointing to a photograph a lady tells Manav, "I am the only daughter of this lady and her son is your maternal uncle". How is 
the speaker related to Manav’s father? 
  (A) Cousin  (B) Wife  (C) Aunt  (D) Daughter–in–law 
DIRECTIONS for questions 5 to 6: Read the following information and answer the questions that follow: 
  A, B, C, D, E and F belong to a family that has two married couples. 
  No one from the third generation is married. 
  C is F’s mother–in–law. 
  D, E and B are A’s teenaged elder sister, father and grand father respectively. 

www.apparteducation.com   74  info@apparteducation.com  


APPART - Academy of Proficient Professionals for Aptitude Research and Training 

5. Which of the following is true? 
  (A) A is C’s wife  (B) C is B’s husband  (C) D is E’s daughter  (D) D is E’s son 
6. Who is C’s husband? 
  (A) A  (B) B  (C) C  (D) E 
7. Pointing to a photograph, a person tells his friend, “She is the grand daughter of the elder brother of my father.” How is the 
girl in the photograph related to this man? 
  (A) Niece  (B) Sister  (C) Aunt  (D) Sister–in–law 
8. Pointing to a photograph, Ankit said, “She is the daughter of my grand–father’s only son.” How is Ankit related to the girl in 
the photograph? 
  (A) Father  (B) Brother  (C) Cousin  (D) Data inadequate 
9.   Introducing a boy, a girl said, "He is the son of the daughter of the father of my uncle." How is the boy related to the girl? 
  (A)  Brother  (B)Nephew  (C) Cousin  (D) Data inadequate 
10.   If A $ B means A is the brother of B; A @ B means A is the wife of B; A # B means A is the daughter of B and A * B means A is 
the father of B, which of the following indicates that U is the father-in-law of P? 
  (A) P@Q$T#W*U  (B) P@W$Q*T#U  (C) P@Q$W*T#U  (D) P@Q$T#U*W 
11.  1. B5D means B is the father of D. 
  2. B9D means B is the sister of D. 
  3. B4D means B is the brother of D. 
  4. B3D means B is the wife of D. 
  Which of the following means F is the mother of K? 
  (A) F9M4N3K  (B) F5M3K  (C) F3M5K  (D) F3M5N3K 
12.   Pointing to a girl in photograph, Aman said, “Her mother’s brother is the only son of my mother’s father, she being the only 
daughter”. How is the girl’s mother related to Aman? 
  (A) Mother  (B) Sister  (C) Aunt  (D) Niece 
13.   Swaty and Sweety are Vivek’s wives. Shweta is Sweety’s step–daughter. How is Swaty related to Shweta? 
  (A) Sister  (B) Mother–in–law  (C) Mother  (D) Step–mother 
14.  Bob has a brother Cob. Bob is the son of Tob. Nob is Tob’s father. In terms of relations, what is Cob of Nob? 
  (A) Son  (B) Grand–son  (C) Brother  (D) Grandfather 
 

DIRECTIONS
Directions are one of the easiest topics in aptitude exams.  While describing the directions sense, two ways are used –  
1.  Universal Sense – In  this  way, direction  is  specified  clearly.  For example,  ‘After  walking 3 km, a  person  turns  to North’.  In  this 
example, the direction is specified explicitly. 
2. Individual Sense – This is more of one’s relative sense. For example, ‘after walking 3 km, a person takes right’. In this example, 
final direction of the person will depend of the direction in which s/he is walking before taking right. 
The distance asked in the question is always the shortest distance unless mentioned otherwise. Shortest distance is obtained when 
we connect two points by a line. The questions based upon direction can be solved easily by making a diagram of the path followed 
by a moving object. The distances can be easily found out by making use of Pythagoras’s theorem.  
www.apparteducation.com   75  info@apparteducation.com  
APPART - Academy of Proficient Professionals for Aptitude Research and Training 

Some of the basic Pythagorean triplets we must know are given as follows 3-4-5, 5-12-13, 7-24-25, 8-15-17, 9-40-41, 20-21-29. Other 
N  
triplets can be derived from them to save the time in exams. 
NE  
The following diagram shows the main and sub-directions.   NW 

W   E  

SW  SE 

The diagram given above depicts four main directions which are North, East, South, West &four sub-directions which can be given  
are N-E, S-E,S-W, N-W 
CLASS ASSIGNMENT
1. I am facing north. I turn left and walk 20 m. Then I turn left again and walk 10 m. Then I turn right and walk 10 m and then 
turning left walk 20 m. Then I turn left again and walk 60 m. In which direction am I from the starting point? 
  (A) East  (B) South–West  (C) South–East  (D) West 
2. A person goes 30 km towards West. Then he takes a right turn and goes another 40 km.  then he turns left and goes 66 km. 
What is his final position with respect to the starting point? 
  (A) 50 km South West(B) 104 km North–west  (C) 50 km North-West  (D) 53 km North-east 
3. Amit goes on a walk. He goes 20 m towards his office and then turns to his right and walks 7 m. Then he takes a left turn and 
walks 18 m after which he turns to his left and goes 21 m. Finally he takes a right turn and walks another 34 m. How far is he 
from the starting point? 
  (A) 73 m  (B) 76 m  (C) 104 m  (D) 112 m 
4. Thomas  wants  to  go  to  see  the  circus.  When he  asks  Vincent  about  the  direction  from  his home,  Vincent  says,  “Go  8 km 
towards the city hospital. From the signal, take a right. After going 6 km, take a left turn and go 3 km further till you reach a 
crossing. From the crossing go 9 km towards your right and then you’ll have to go another 9 km to your left to reach the circus 
ground.” How far is the circus from his home? 
  (A) 35 km  (B) 25 km  (C) 27 km  (D) 34 km 
DIRECTIONS for questions 5 to 7: If you start running from a point towards north and after covering 4 km you turn to your left and 
run 5 km, and then again turn to your left and run 5 km and then turn to left again and run another 6 km and before finishing you 
take another left and run 1 km. 
5.   How many km are you from the place you started? 
  (A) 1  (B) 2  (C) 3  (D) 4 
6.   In which direction will you be running while finishing? 
  (A) east  (B) west  (C) north  (D) south 
7.   After taking the second turn, in which direction will you be running? 
  (A) east  (B) west  (C) north  (D) south 

www.apparteducation.com   76  info@apparteducation.com  


APPART - Academy of Proficient Professionals for Aptitude Research and Training 

DIRECTIONS for questions 8 to 10: Study the given information and answer the following questions. 


I) There are 6 check posts A, B, C, D, E and F. 
II) Check post F is 25 km to the north of D which is 35 km to the north east of B. 
III) Check post A is 15km west of E and 35 km to the south west of C. 
IV) B, A and E are in straight-line. 
V) The check posts B and E are 70kms apart from each other. 
8.   Which check post is the farthest to the south west of D? 
  (A) A  (B) B  (C) C  (D) cannot be determined 
9.   What is the distance between C and D?
(A) 65 km  (B) 110 km  (C) 70 km  (D) None of these 
10.   If a jeep moves from E to F via A, B and D, how much distance it will have to cover (in km)? 
(A) 130 (B) 189 (C) 289 (D) 797

CUBES
A cube is a six faced three dimensional object having length, breadth and height all equal.  
Face: A cube has 6 faces. 
Edge: Edge is an intersection of two faces. A cube has 12 edges. 
Corner: Corner is an intersection of three edges. A cube has 8 corners. 
When a cube is cut into n divisions along each of length, breadth and height, we get in all n × n × n = n3 small cubes. Following table 
illustrates the number of small cubes out of these n 3 smaller cubes and number of colored faces they have. 
Number of faces colored Number of smaller cubes Parameter used
3  8  Corners 
2  12 (n – 2)  Edges 
1  6 (n – 2)2  Faces 
0  (n – 2)3  Core region 
CLASS ASSIGNMENT
DIRECTIONS for questions 1 to 6: Read the information given below and answer the questions that follow: 
Asolid cube of side 8 cm has been painted red, blue and black on pairs of opposite faces. It is then cut into cubical blocks of each side 
2 cm. 
8 cm 
 
 
 
2 cm 
 
 
 
 
1. How many cubes have no face painted? 
  (A) 0  (B) 4  (C) 8  (D) 12 

www.apparteducation.com   77  info@apparteducation.com  


APPART - Academy of Proficient Professionals for Aptitude Research and Training 

2. How many cubes have only two faces painted? 
  (A) 8  (B) 16  (C) 20  (D) 24 
3. How many cubes have three faces painted with different colors? 
  (A) 0  (B) 4  (C) 8  (D) 12 
4. How many cubes have two faces painted red, black, and all other faces unpainted? 
  (A) 4  (B) 8  (C) 16  (D) 32 
5. How many cubes have only one face painted red and all other faces unpainted? 
  (A) 4  (B) 8  (C) 12  (D) 16 
6.  How many cubes have two of their faces painted black? 
  (A) 2  (B) 4  (C) 8  (D) None 
DIRECTIONS for questions 7 to 11: Answer the questions that follow: 
A solid cube painted red on two adjacent sides, black on the side opposite to the red sides and green on the remaining sides is cut 
into sixty-four smaller cubes of equal size. 
7.  How many cubes have at least one side as red? 
  (A) 16  (B) 24  (C) 28  (D) 32 
8. How many cubes are there with one side green and the adjacent side either red or black and painted on two sides only? 
  (A) 8  (B) 16  (C) 24  (D) 32 
9. How many cubes are there which are red on one side and black on the opposite side? 
  (A) 0  (B) 4  (C) 8  (D) 16 
10. How many cubes have two adjacent sides either red or black? 
  (A) 32  (B) 16  (C) 8  (D) 4 
11. How many cubes have at least one side black? 
  (A) 36  (B) 32  (C) 28  (D) 24 
DIRECTIONS for questions 12-15: Read the information given below and answer the questions that follow: 
Six dice with their upper faces erased are as shown: 
 
 
 
 
  (i)(i)  (i)(ii)  (i)(iii) 

(i)(iv)  (i)(v)  (i)(vi) 


The sum of the number of dots on the opposite faces is 7.
12. If the dice (i), (ii), (iii) have even number of dots on their bottom faces, then what would be the total number of dots on the 
top faces? 
(A) 14  (B) 7  (C) 21  (D) 12 
www.apparteducation.com   78  info@apparteducation.com  
APPART - Academy of Proficient Professionals for Aptitude Research and Training 

13. If dice (i), (ii) and (iii), have even number of dots on their bottom faces and the dice (iv), (v), (vi) have odd number of dots on 
the top faces, then what would be the difference in the total number of top face dots between these two sets? 
(A) 0  (B) 1  (C) 2  (D) 3 
14. If odd numbered dice have odd number dots on their bottom faces, what would be the total number of dots on the top faces 
of these dice? 
(A) 4  (B) 6  (C) 10  (D) 12 
15. If even numbered dice have even number of dots on their top faces, what would be the total number of dots on the top faces 
of these dice? 
(A) 18  (B) 14  (C) 12  (D) 10 
16. The faces of the cubes are numbered from 1 to 6. Four different views of the cubes are shown below. What is the number 
opposite to 2? 
 
 
 
 
 
 
 
 

(A) 3  (B) 1  (C) 4  (D) 6  (E) 5 


17. Four different positions of the cubes are given below. Which number is opposite to 6? 
 

(A) 1  (B) 4  (C) 3  (D) 2  (E) 5 


18. Two positions of a die are shown below. Point out which number will be at the bottom if number 5 is on the top. 
 

(A)  6  (B) 3  (C) 2  (D) 1  (E) 4 

www.apparteducation.com   79  info@apparteducation.com  


APPART - Academy of Proficient Professionals for Aptitude Research and Training 

DIRECTIONS for questions 19 and 20: In the following questions, given figures represent four different views of a cube. Study the 


figures and answer the questions that follow: 
19. Which number is on the face opposite to number 4? 
 

(A) 6  (B) 5  (C) 1  (D) 3  (E) 4 


20. Which number is on the face opposite to 3? 
 

(A) 4  (B) 2  (C) 5  (D) 6  (E) 1 


DIRECTIONS for questions 21 to 25: Read the information given below and answer the questions that follow: 
A solid cube of each side 9 cm has been painted green, red and yellow on pairs of opposite faces. It is then cut into cubical block of 
each side 3 cm. 
21. How many cubes have only one face painted? 
(A) 6  (B) 8  (C) 28  (D) 24 
22. How many cubes have three faces painted? 
(A) 0  (B) 6  (C) 8  (D) 4 
23. How many cubes have no face painted? 
(A) 12  (B) 8  (C) 1  (D) 0 
24. How many cubes have only two faces painted? 
(A) 2  (B) 4  (C) 6  (D) 12 
25. How many cubes have all the four faces painted? 
(A) 0  (B) 1  (C) 2  (D) 3 

SET THEORY
Set: A set is a “well defined” collection of objects. Here the term “well defined” is important. For a “well defined” set, one is able to 
tell clearly whether a random element “x” is a member of the set or not. 
e.g. Set of all the vowels is a “well defined” set because one can easily tell whether ‘b’, ‘e’, ‘r’, ‘1’, ‘0’ etc. are a member of the set or 
not. 
Set of seven numbers is not “well defined” since we don’t know which seven numbers are we talking about. 
Sets are represented in two forms: 
1. Roster/Tabular form: All the elements of the set are listed inside curly brackets. 
e.g.  A = {2, 3, 5, 7, 11, 13, 17, 19, 23, 29} 
2. Set Builder/Rule form: Here we define the set by writing the properties which are shared by all or some of the elements of 
the set.

www.apparteducation.com   80  info@apparteducation.com  


APPART - Academy of Proficient Professionals for Aptitude Research and Training 

e.g.  A = {x| x is a prime number less than 30} 
3. Venn Diagrams: Sets can also be represented using geometrical figures like circles, rectangles, triangles etc.
e.g.  Universal Set 

  A 
2, 3, 5, 7, 
Venn diagrams are the most useful tool for solving questions based upon set theory.  11, 
13, 

Interaction between two sets: 


Any two sets A and B can any one of the following interaction between them based on the conditions given: 
1)  Both the sets are some distance apart. There is no element common between them. 
  A  B 
 
 
 
2)  Both the sets intersect each other. There is at least one element which is common to them. 

  B 
 
 
 
The intersection is the region of common elements. 
 
3)  One set is contained inside the other. All the elements of one set are common to the other one. 
  A  B 
  B  A 

  OR 
 
 

Interaction between more than two sets: Take two sets at a time and draw their Venn diagrams as mentioned above. Combine all 


the diagrams and get the final diagram showing all the sets. 
CLASS ASSIGNMENT
DIRECTIONS for questions 1 to 6:  Each  of  these  questions  below  contains  three  groups  of  things.  You  are  to  choose  from  the 
following four numbered diagrams, the diagram that best depicts the correct relationship among the three groups of things in each 
question. 
 
 
 
  a)  c) 
b)  d) 
 
www.apparteducation.com   81  info@apparteducation.com  
APPART - Academy of Proficient Professionals for Aptitude Research and Training 

1. Doctors, Husbands, Male 
2. Students, Teachers, Players 
3. Cows, Buffalo, Cattle 
4. Alphabet, Vowels, Numbers 
5. Natural Numbers, Prime Numbers, Composite Numbers 
6. Tennis Fans, Cricket Players, Students 
7.  In a school, 42 of students like cricket, 5 of students like cricket and football and 10 like none. If 96 students like only 
football, then how many students like at most one game? 
  (A) 170  (B) 90  (C) 180  (D) 190  (E) None of these 
DIRECTIONS for questions 8 to11:The following Venn diagram represents the number of medals won in four sports by students of 
Wonder School of Technology from 2001 to 2011. 
 
Volleyball  Basketball 
  21  61 
13 
 
7  4 
 
51  8  23 
 
3  8 
 
  26  15  98 
Cricket  Football 
 
8. The number of medals won in at most two sports is? 
  (A) 302  (B) 30  (C) 308  (D) 102 
9. The number medals won in at least three sports is? 
(A) 32  (B) 30  (C) 33  (D) 35 
10. The number of medals won in at least one sport is? 
(A) 308  (B) 318  (C) 328  (D) 338 
11. What is the difference between the numbers of medals won in at least one sport and that won in exactly two sports? 
(A) 206  (B) 216  (C) 236  (D) 238 
DIRECTIONS for questions 12 to 15: There are 106 employees in a company each of who likes at least one of Tea, Coffee and Milk. 
48 like Tea, 51 like Coffee and 53 Milk. 16 like Tea and Coffee, 17 like Tea and Milk and 18 like Coffee and Milk. 
12.  The number of employees who like exactly two drinks is? 
(A) 31  (B) 32  (C) 33  (D) 36 
13.  The number of employees who like all the three drinks is? 
(A) 5  (B) 6  (C) 7  (D) 4 
14.  The number of employees who like exactly one drink is? 
(A) 45  (B) 55  (C) 65  (D) 70 
15.  The number of employees who like Tea and Coffee but not Milk is? 
(A) 9  (B) 11  (C) 10  (D) 12 

www.apparteducation.com   82  info@apparteducation.com  


APPART - Academy of Proficient Professionals for Aptitude Research and Training 

DIRECTIONS for questions 16 to 18:In a city, 770 people were surveyed about the type of music they listen to from Rock, Pop and 


Trance.
 500 listen to Rock, 400 Pop and 300 Trance. 
 Number of people who listen to all the three types is 30% of those who listen to Trance, and is10% less than those who listen 
to both Rock and Pop but not Trance. 
 Number of people who  listen to both Rock and Trance but not Pop is 33.33% less than the number of people who listen to 
only Pop. 
 Number of people who listen to Pop and Trance but not Rock is 30. 
16. How many people listen to Rock only? 
  (A) 190  (B) 170  (C) 120  (D) 150 
17. How many people listen to all the three types of music? 
  (A) 190  (B) 100  (C) 90  (D) 130 
18. By  what  percent  is  the  number  of  people  who  listen  to  Trance  only  less  than  those  who  listen  to  Rock  and  Pop  but  not 
Trance? 
  (A) 66.66%  (B) 33.33%  (C) 40%  (D) 62.5% 
DIRECTIONS for questions 19 to 23: From the following five diagrams, choose the one that best illustrates the relationship between 
the three given entities in the questions. 
19. Potato, Vegetables, Eatables.
20. Liquids, Milk, River Water.
a b c
21. Dogs, Pets, Cats.
22. Criminals, Pick - Pocketers, Arsonists.
23. Computer, Software, Hardware.  d e

SYLLOGISM
Syllogism:Syllogism  is  a  form  of  “deductive  reasoning”  in  which  a  conclusion  is  derived  from  two  or  more  “premises”.  The 
“premises” are actually “a  set of statements” given in the question and usually it is asked to find whether the one or more of the 
conclusions given are a logical deduction from the statements or not. 
These types of questions can easily be solved by making use of Venn diagrams. 
Given Statement Deduction Truth-meter Summary
Some A are B  Definitely True 
All' can give only 'Some' as 
Some B are A  Definitely True 
All A are B  a  definitely  True 
Some B are not A  Probably True 
statement 
Some A are not B  Definitely False 
Some B are A  Definitely True  Some'  can  give  only 
Some A are B  Some B are not A  Probably True  'Some' as a definitely True 
Some A are not B  Probably True  statement 
No definitely True deduction possible 
Some A are not B 
Some B are  not A  Probably True    
No A are B   No B are A  Definitely True  No'  can  give  only  'No'  or 

www.apparteducation.com   83  info@apparteducation.com  


APPART - Academy of Proficient Professionals for Aptitude Research and Training 

Some A are not B  Definitely True  'Some+Not' as a definitely 


Some B are not A  Definitely True  true statement. 

Remember that
 No positive statement can give rise to any negative definitely true conclusion. 
 No negative statement can give rise to any positive definitely true conclusion. 
CLASS ASSIGNMENT
DIRECTIONS for questions 1 to 5: In the following questions, two statements are followed by two conclusions. Mark your answer as
(A)  If only conclusion I follows. 
(B)  If only conclusion II follows. 
(C)  If either conclusion I or II follows. 
(D)  If both the conclusions follow. 
(E)  If none of the conclusions follow. 
1. Statement I: All fish are snakes.    II: No snakes are sparrows. 
Conclusion I:  No fish is a sparrow.    II:  No sparrow is fish. 
2. Statement I: Some black is pigeons.    II: All pigeons are yellow. 
Conclusion I: No black is yellow.    II: No yellow is black. 
3. Statement: I: Somecats are dogs.    II: No dogs are rats. 
Conclusion I:  Some cats are rats.    II: No cats are rats. 
4. Statement I: Some comets are stars.    II: Some stars are planets. 
Conclusion I: Some comets are planets.   II: Some planets are comets. 
5. Statement I: No girls are intelligent.    II: Some intelligent are nice. 
Conclusion I: No girls are nice.  II: Some girls are nice. 
DIRECTIONS for questions 6 to 10: In each of the following questions, read the two statements and the four conclusions. Choose
the correct answer option.
6. Statement I: Some newspapers are radios.  II: Some radios are televisions. 
III:  No television is magazine. 
Conclusion I: No newspaper is magazine.  II: No radio is magazine. 
III:  Some radios are not magazines.  IV:  Some newspapers are televisions. 
(A) None follow    (B) III only    (C) I or II only    (D) I or II only 
7. Statement I: Some fowls are insects.    II: All fowls are butterflies. 
III:  All insects are snakes. 
Conclusion I: Some snakes are fowls.    II: Some butterflies are insects. 
III:  Some snakes are butterflies.  IV:  Some insects are fowls. 
(A) None follows    (B) All follow    (C) IV only    (D) I, II and IV only 
8. Statement I: All cats are rats.      II: Some cats are tigers. 
III:  No tiger is red. 
Conclusion I: Some tigers are rats.    II: Some cats are red. 
III:  Some rats are cats.    IV:  Some rats are red. 
(A) II only and either I or III follow  (B) II and IV only    (C) I and III only    (D) I and II only 
www.apparteducation.com   84  info@apparteducation.com  
APPART - Academy of Proficient Professionals for Aptitude Research and Training 

9. Statement I: All artisans are painters.    II: All actors are gentlemen. 


III:  All gentlemen are painters. 
Conclusion I: No artisan is an actor.    II: All painters are actors. 
III:  Some gentlemen are not actors.  IV:  Some artisans are gentlemen. 
(A) I only    (B) Either I or III and IV  (C) II and IV only    (D) None of these 
10. Statement I: All kings are beggars.    II: All beggars are laborers. 
III:  All laborers are careless. 
Conclusion I: Some careless are kings.    II: No laborer is king. 
III:  All kings are laborers.    IV:  All laborers are kings. 
(A) I and II only    (B) I and IV only    (C) I, II and IV only  (D) None of these 
DIRECTIONS for questions 11 to 20:In the following questions, two statements are followed by two conclusions. Mark your answer 
as 
(A)  If only conclusion I follows. 
(B)  If only conclusion II follows. 
(C)  If both the conclusions follow. 
(D)  If none of the conclusions follow. 
(E)  If either conclusion I or II follows. 
11. Statements I: All diamonds are stones.    II:  No stone is glass. 
Conclusions I:  No diamond is glass.    II: No glass is a stone. 
12. Statements I: All fish are hens.      II:  All hens are boys. 
Conclusions I: All fish are boys.      II: All hens are fish. 
13. Statements I: Allcards are notes    II:  No note is a page. 
Conclusions I: Some cards are pages.    II:  Some pages are notes. 
14. Statements  I: All players are females.    II:  No female is smart. 
Conclusions  I: No player is smart.    II: No male is in the team. 
15. Statements  I: Some cows are cars.    II:  'X' is a cow. 
Conclusions I: 'X' is not a cow.      II: Some cars are not cows. 
16. Statements I: All fathers are sons.    II:  All sons are grand fathers. 
Conclusions I:  All grand fathers are fathers.  II: All fathers are grand fathers. 
17. Statements I: Some trains are trees.    II:  No tree is a picture. 
Conclusions I: Some trains are pictures.    II: No train is a picture. 
18. Statements I: Some trays are fans. II:  All fans are bottles. 
Conclusions I: No bottle is a tray.    II:  Some trays are bottles. 
19. Statements  I: Some books are toys.    II:  No toy is red. 
Conclusions  I: Some books are red.    II: Some books are not red. 
20. Statements  I: Some rivers are seas.    II:  All seas are skies. 
Conclusions I: All skies are seas.     II: Some rivers are skies. 

www.apparteducation.com   85  info@apparteducation.com  


APPART - Academy of Proficient Professionals for Aptitude Research and Training 

DATA INTERPRETATION
INTRODUCTION: For  a  professional,  not  a  day  passes  without  coming  across  statistical  data.  Unorganised  and  haphazard  data 
seldom gives  full  and  correct picture. Even  if  it does it is often cumbersome and  tedious. Hence, any  data, be it daily  production 
figures, daily sales figures, financial performance or productivity, will have to be presented in a  concise manner–at the same time 
being precise so that managers can study it in the least time, thus facilitating faster decision making. Study and manipulation of such 
data leads us to an important area–namely Data Interpretation. 
Data can be organised in a number of ways so  that larger volume of data can be presented in a more compact and precise form. 
Data thus presented has to be deciphered correctly by the user of the data. This process of deciphering the data from its compactly 
presented form is called Data Interpretation. 
REPRESENTATION OF DATA:
Numerical data can be presented in one or more of the following ways: 
  i) Data Table  ii) Pie Chart 
  iii) Line Graph  iv) Bar Chart  v) Others 
The "Others" category covers miscellaneous forms like descriptive case format, etc. customised for the situation. Data can also be 
presented by using a combination of two or more of the above forms. 
While some data can be presented in many different forms, some other data may be amenable to be presented only in few ways. In 
real life situations, the style of data presentation is based on the end–objective. In certain situations data has to be presented as a 
combination of two or more forms of data presentation. 
Let us understand each of the above forms of data presentation with an example. 
DATA TABLE: Here data is presented in the form of a table. While any type of data can be presented in tabuiar form, that too in a 
very accurate manner, interpreting the data in table form becomes more difficult and time consuming than the other modes, all of 
which are basically pictorial or graphical in presentation. 
Data tables can be of a number of types. They can be of a single–table variety or combination of tables. One such example of tables 
is given below. 
 

A  B  C  D  E 
Year 
Appeared  Qualified  Appeared  Qualified  Appeared  Qualified  Appeared  Qualified  Appeared  Qualified 
2002  1250  720  1750  460  1000  120  800  120  2000  370 
2003  2750  810  1860  490  1120  200  1000  220  2200  420 
2004  3000  890  2000  520  1250  300  1200  300  2500  510 
2005  3250  910  2100  640  1500  600  1210  340  2750  680 
2006  3720  1050  2400  830  1650  780  1440  480  3240  960 
 

The above table shows the number of students appeared and qualified in an entrance test from five districts A, B, C, D and E of a 
state. 
From the above table, we can obtain the following data: 
 Total number of students appeared and qualified from the given districts in each of the years.  
 Percentage increase in the number of students appeared or qualified in a district over the years.  
 Average number of students appeared or qualified 

www.apparteducation.com   86  info@apparteducation.com  


APPART - Academy of Proficient Professionals for Aptitude Research and Training 

PIE–CHARTS:
This  is  probably the simplest of all  pictorial forms of data presentation. Here, total quantity  to be  shown  is  distributed over  one 
complete circle or 360 degrees. In pie–charts, data is essentially presented with respect to only one parameter (unlike in the 2 and 
3–dimensional graphs described later). This form essentially presents shares of various elements as proportion or percentage of the 
total quantity. Each element or group in the pie chart is represented in terms of quantity (or value, as the case may be) or as  the 
angle made by the sector representing the elements or as a proportion of the total or as a percentage of the total. 
Chart–1 gives distribution of sales different companies. 
  CHART – 1
Total sales: Rs. 6,000 crores
 
Bharat
  Others
Petroleu
  10% m
 
Hindustan
  Petroleum
  18%
 
Castrol India IOC
 
17% 37%
 
 

From the above pie chart, we can calculate the following: 
 Total sales of each of the companies. 
 Sales of a company as a percentage of the other. 
 Conversion of these percentage values into angles for each zone. 
Pie Charts are also very frequently used in combination with other forms of data or along with other pie–charts. 

TWO–DIMENSIONAL GRAPHS:
This is essentially used for continuous data but can also be used for depicting discrete data provided we understand the limitation. 
Also known as Cartesian Graphs, they  represent variation of one parameter with  respect to  another parameter  each shown on a 
different axis. These types of graphs are useful in studying the rate of change or understanding the trends through extrapolations. 
These graphs can be of various types and a few of them are shown below: 
  CHART – 2
70
 
60
Profit in Rs. Lakh

 
50
 
40
  30
  20
  10
  0
2000 2001 2002 2003 2004 2005 2006

www.apparteducation.com   87  info@apparteducation.com  


APPART - Academy of Proficient Professionals for Aptitude Research and Training 

The graph in Chart 2 shows the changes in the profit of acompany during period of time. One can find out trends and the growth in 
the profit over the years.   CHART – 3
 
MOTION GRAPH OF Q1, Q2 and Q3
  16 Seconds
14
  12
10
  8
6
  4
2
  0
  0 20 40 60 80 100 120 140 160
Speed (in meters/second)
 
Q1 Q2 Q3
 
Chart 3 presents another type of two–dimensional graph which is mostly used to depict scientific data like speed, velocity, vectors 
etc. In the graph speed trends of three bodies Q1, Q2, Q3 is given along with their actual path of motion. 
BAR CHARTS: This is a type of graph used mostly to depict data in a discrete way. They are accurate and comparison of variables is 
very convenient.  CHART –
Import and Export 4 of XYZ
 
Ltd.
 
  1400
1200
 
in Rs. Crore

1000
 
800
 
600
  400
  200
  0
2001 2002 2003 2004 2005 2006 2007
 
  Imports Exports
Chart 4 shows Imports and exports of a company over different years. From this graph we can obtain the following: 
 Percentage contribution of imports or exports to the company’s total trade for different years.  
 Relative increase or decrease in the share of imports or exports  
 Percentage growth/average annual growth in imports or exports during given period. 

THREE–DIMENSIONAL GRAPHS:
The data in a triangular graph are given on each side of the triangle. Each point represents three different values, one each in each 
direction. 
 
 

www.apparteducation.com   88  info@apparteducation.com  


APPART - Academy of Proficient Professionals for Aptitude Research and Training 

  CHART 5
 
 
100% 25%
  Horlicks 75% Complan
50%
  A
  50% 75%
 
25% B 100%
  C
 
 
 
100% 75% 50% 25%
  Milo
The graph in chart 5 represents the percentage of people who like the three health drinks–Horlicks, Complan and Milo in three cities 
A, B and C. For example, in city B, 25% like Complan, 50% like Milo and 25% like Horlicks. 

VENN–DIAGRAMS:
Venn–Diagrams  is  one of  the  convenient  and  commonly  used  representation  to  understand  how  two  or  more  sets  have overlap 
among them.  CHART
Chemistry (150) 6
 
Physics
  (175)
  35
25
25 15

Math (225)

Chart 6 represents the number of students who like one or more subjects among Physics, Chemistry and  Maths in 10th class of a 
school. From this Venn–Diagram, we can understand how many students like only Physics, Physics as well as Chemistry and who like 
Physics, Chemistry and Maths etc. The same for Chemistry or Maths can be calculated. 
CLASS ASSIGNMENT
DIRECTIONS for questions 1 to 5: These questions are based on  the  table given below, which  shows  the number of new persons 
(men and women) settling in five residential colonies A, B, C, D and E across five years. 
  Colony Gender  2000  2001  2002  2003  2004 
  Male  151  167  156  163  178 
A
  Female  139  143  141  142  156 
  Male  134  141  150  164  173 
B
  Female  129  133  147  161  169 
Male  167  171  169  176  184 
  C
Female  155  163  161  168  173 
 
Male  183  171  181  188  196 
  D
Female  189  181  182  192  199 
 
Male  106  109  117  126  129 
  E
Female  101  103  111  121  124 
 

www.apparteducation.com   89  info@apparteducation.com  


APPART - Academy of Proficient Professionals for Aptitude Research and Training 

1. During the given years, in which of the following residential colonies is the number of new persons settling, the highest? 
  (A) A  (B) B  (C) C  (D) D  (E) E 
2. In 2003, the ratio of males to females newly settling in which colony is the least? 
  (A) A  (B) D  (C) C  (D) E  (E) None of these 
3. What is the percentage increase in the total number of new persons settling in colony C from the year 2000 to 2004? 
  (A) 10.87%  (B) 11.64%  (C) 12.19%  (D) 13.67%  (E) 15.33% 
4. During the year 2001, in how many colonies is the number of new persons settling more than 320? 
  (A) 4  (B) 3  (C) 2  (D) 1  (E) 0 
5. In colony E, by what percent, average number of male settling is more than that of female settling, for five years? 
  (A) 3%  (B) 10%  (C) 4%  (D) 5%  (E) None of these 
DIRECTIONS for questions 6 to 10: These questions are based on the following line graph. 
Profit of GLASGOWE
 
  50 50
50 45 45
  45 40
40 35
 
35
In Rs. Lakh

  30 25
25 28 21 23
 
20 15
  15
10 7
 
 
5
 

 
0
Expenditure = Income–Profit 
1999 2000 2001 2002 2003 2004
Profit
Profit percentage =   100 Profit Income
Income  
6. In which year is the profit percentage of Glasgowe Limited, the highest? 
  (A) 2000  (B) 2001  (C) 2002  (D) 2004  (E) 2003 
7. In which year is the increase/decrease in the expenditure of Glasgowe Limited, the highest when compared to its previous 
year? 
  (A) 2000  (B) 2003  (C) 2002  (D) 2001  (E) 2004 
8. From the year 1998 to  1999, the profit of Glasgowe  Limited, has  increased by 40% and  the  income has  increased by 25%. 
What was the expenditure of Glasgowe Limited, in 1998? 
  (A) Rs. 21 Lakh  (B) Rs. 22 Lakh  (C) Rs. 23 Lakh  (D) Rs. 24 Lakh  (E) Rs. 25 Lakh 
9. The profit of Glasgowe Limited, in which year is closest to the average profit per year for the given period? 
  (A) 2000  (B) 2002  (C) 2003  (D) 2004  (E) 2001 
10. In which year was the rate of change of expenditure, as compared to previous year, the highest? 
(A) 2001  (B) 2002  (C) 2003  (D) 2004  (E) None of these 
DIRECTIONS for questions 11 to 15:The  following  pie-charts  highlight  the  distribution  of  household  savings  amongst  Indian 
households for  the years  1995-96 and  1996-97. Socials & Security Funds  (SSF) moved up from Rs. 37,200  crore  in 1995-96  to Rs. 
40,000 crore in 1996-97. Study the pie-charts and answer the questions that follow . 
www.apparteducation.com   90  info@apparteducation.com  
APPART - Academy of Proficient Professionals for Aptitude Research and Training 

1995-96 Bank  1996-97


SSF Deposit
s Bank 
31%
29% SSF Deposits
25% 38%

COG
COG 8%
10% Non  S & D
bank  4%
S & D deposits Non 
5% 11% bank 
Currency Currency deposits
14% 10% 15%  
S & D: Shares and debentures; COG: Claims on govt.; SSF: Socials & Security Funds 
11. The savings by households in the form of currency for the year 1996-97 was approximately (in Rs. crore) 
  (A) 10,520  (B) 12,000  (C) 16,000  (D) 15,500 
12. The total savings by households in all kinds of financial assets for the year 1995-96 was (in Rs. crore) 
(A) 120,000  (B) 104,500  (C) 138,000  (D) 140,000 
13. The contribution of shares and debentures in the household savings have increased over the year 1995-96 to 1996-97 by 
(A) 10.6%  (B) 6.7%  (C) 1.5%  (D) 1% 
14. The total savings in the form of deposits (both banks and non-banks) for the year 1996-97 stood at (in Rs. crore) 
(A) 76,720  (B) 84,800  (C) 88,500  (D) 92,100 
15. The financial assets which recorded the highest growth rate in 1996-97 was 
(A) Non-bank deposits (B) bank deposits  (C) claims on government  (D) currency 
DIRECTIONS for questions 16 to 20:The following  graphs show the  production (in  ‘000)  and  the  selling  price per unit  of Gilchrist 
Industries for the first six months of 2001. Refer to the graphs to answer the questions that follow. 

Production of Items (in '000) Selling Price Per Unit (in Rs.)
70 600
60 500
50 400
40
300
30
20 200
10 100
0 0
Jan Feb Mar Apr May Jun Jan Feb Mar Apr May Jun
   
 
16. What was the total production in the six months? 
(A) 125000  (B) 175050  (C) 205000  (D) 220000 
17. If all the items produced were sold, what was the revenue earned in the first six months? 
www.apparteducation.com   91  info@apparteducation.com  
APPART - Academy of Proficient Professionals for Aptitude Research and Training 

(A) Rs. 5.105 crore(B) Rs. 6.4 crore  (C) Rs. 7.255 crore  (D) Rs. 8.25 crore 


18. What percentage of the revenue earned in March is the revenue earned in May, if only 80% of the items produced in March 
could be sold while the entire production of May was sold? 
(A) 117.5%  (B) 177.5%  (C) 182.5%  (D) 187.5% 
19. If all the items produced were sold, find the difference in the revenues earned in April and in May? 
(A) Rs. 50 lakh  (B) Rs. 75 lakh  (C) Rs. 77.5 lakh  (D) Rs. 89.5 lakh 
20. As compared to june there was a 400% increase in the production in July,  but a reduction of 50% in the selling price, what 
was the revenue earned in the month of July? (consider that all the items produced were sold.) 
(A) Rs. 1 crore  (B) Rs. 1.25 crore  (C) Rs. 1.75 crore  (D) Rs. 2 crore 

DIRECTIONS for questions 21 to 25:These questions are based on the pie chart given below. 


  Investment by Raghunandan in Different Companies (in Rs. Thousand)
 
  F A
330 240
 
B
  360
E
  570
C
 
600
 
 
D
 
90
 
21. What percentage of Raghunandan’s total investment is his investment in company B? 
  (A) 12%  (B) 24%  (C) 36%  (D) 48%  (E) 60% 
22. The ratio of investments in company C to that in company A is 
  (A) 1.75  (B) 2.5  (C) 2.75  (D) 3.25  (E) 3.5 
23. By what percent, percentage investment of Raghunandan in company C is more than that in company F? 
(A) 40%  (B) 22.22%  (C) 81.81%  (D) 55%  (E) 11.11% 
24. The return on investments in companies D and E are 11% and 15% respectively for the year 2004. What are the total earnings 
of Raghunandan from these companies for the year 2004?  
  (A) Rs. 154500  (B) Rs. 174500  (C) Rs. 164500  (D) Rs. 184500  (E) Rs. 194500 
25. The investments in how many companies is more than the average investment per company?  
  (A) 1  (B) 2  (C) 3   (D) 4  (E) 5 

DIRECTIONS for questions 26 to 30:The table below shows the reservation position (Number of berths reserved) in trains A, B, C, D 


and E on a particular day under different classes. The trains run from the same place to a particular destination on the same route. 
The total number of berths allocated for a particular class is equal in all the trains and is shown against the particular class of the 
journey in the table. Study the following table carefully and answer the questions that follow. 

www.apparteducation.com   92  info@apparteducation.com  


APPART - Academy of Proficient Professionals for Aptitude Research and Training 

 
 
AC - 1st Class AC Sleeper AC Chair Car 1st Class 2nd Class
Train             Class
Capacity - 200 Capacity - 200 Capacity - 400 Capacity - 300 Capacity - 500
 
A 185 200 312 281 485
  B 190 183 348 215 414
  C 198 191 364 291 484
  D 170 178 290 199 429
E 167 199 377 276 412
 
26. How many berths in all the classes together are vacant in trains B & C? 
(A) 278  (B) 212  (C) 322  (D) 612 
27. What is the approximate difference between the percentage of reserved position of the trains showing lowest in Chair Car 
and highest in 2nd Class? 
(A) 30  (B) 19  (C) 5  (D) 25 
28. What is the difference in the percentages of reserved berths in AC 1st Class of train B and train D? 
(A) 12  (B) 20  (C) 2  (D) 10 
29. The difference between the reservation position of trains A and B in AC  sleeper is equal to  the difference between trains D 
and E in which of the following classes? 
(A) AC Chair Car  (B) 2nd Class  (C) AC 1st Class  (D) 1st Class 
30. Which of the following trains has the highest number of vacant berths in Non-AC classes put together? 
(A) D  (B) B  (C) C  (D) E 
 

DATA SUFFICENCY
The  data  sufficiency  questions  are  designed  to  test  candidates  reasoning  ability.  Basic  knowledge  of  arithmetic,  algebra  and 
geometry  is  prerequisite  to  solve questions on  data  sufficiency.  These  questions  indeed  take  lesser  time  in  comparison  to  other 
questions, but then they can be very tricky. Just keep in mind that you have to  see if the data is sufficient to  get the desired the 
answers, please do not sit down to actually solve them. 
How to solve data sufficiency questions: 
1. Read the question and understand the formula and rule required to solve it. 
2. Take statement I and use the information given in it along with the statement given in the question. Check if you can arrive at 
a solution. Do not try to solve the question; first ensure that a solution can be obtained. The student should be careful not to 
read any more into a statement than what is given. 
3. Take statement II in isolation to statement I. Combine the information given in statement II with that of already given in the 
question. Check if you can arrive at a solution. Remember not to use data from statement I. 
4. If a solution is not arrived at from either statements I or II individually, combine the data available from the two statements 
and check if you can arrive at a solution. 
5. Select the right option. 

www.apparteducation.com   93  info@apparteducation.com  


APPART - Academy of Proficient Professionals for Aptitude Research and Training 

CLASS ASSIGNMENT
In  each of  the questions below  consists of a  question and  two statements numbered  I and II given below it.  You have to  decide 
whether the data provided in the statements are sufficient to answer the question. Read both the statements and 
Give answer 
(a)  If the data in statement I alone is sufficient to answer the question, while the data in statement II alone are not sufficient to 
answer the question 
(b)  If the data in statement II alone is sufficient to answer the question, while the data in statement I alone are not sufficient to 
answer the question 
(c)  If the data either in statement I alone or in statement II alone is sufficient to answer the question 
(d)  If the data given in both statements I and II together are not sufficient to answer the question and 
(e)  If the data in both statements I and II together are necessary to answer the question. 
1. What day is the fourteenth of a given month? 
I. The last day of the month is Wednesday. 
II. The third Saturday of the month was seventeenth. 
2. What is Suman’s rank from the top in the class of 40 students? 
I. Reena is 4 ranks below Suman and is thirty-first from the bottom. 
II. Anuj is two ranks above Suman and is thirty-seventh from the bottom. 
3. What does ‘ta’ mean in a code language? 
I. ‘pa ta ja’ means ‘over and above’ in that code language. 
II. ‘ho ka pa’ means ‘come over here’ in that code language. 
4. Among D, F, J, P and A, who reached office last, if it is known that only one person reached last? 
I. F and J reached office together. 
II. Only D and P reached office ahead of J. 
5. What are the values of m and n? 
I. n is an even integer, m is an odd integer, and m is greater than n.
II. B. Product of m and n is 30. 
www.apparteducation.com   94  info@apparteducation.com  
APPART - Academy of Proficient Professionals for Aptitude Research and Training 

6. Two friends, Ram and Gopal, bought apples from a wholesale dealer. How many apples did they buy?   
I. Ram bought one-half the number of apples that Gopal bought.
II. The wholesale dealer had a stock of 500 apples.  
7. What does ‘$’ mean in a code language? 
I. ‘5$#3’ means ‘flowers are really good’. 
II. ‘7#35’ means ‘good flowers are available’. 
8. How is D related to A? 
I. B is brother of A. 
II. B is D’s son. 
9. What will be the total weight of 10 balls, each of the same weight? 
I. One-fourth of the weight of each ball is 8 kg. 
II. The weighing machine shows 10% error. 
10.  In a hockey match, the Indian team was behind by 2 goals with 5 minutes remaining. Did they win the match?   
I. Deepak Thakur, the Indian striker scored 3 goals in the last 5 minutes of the match. 
II. The opponent team scored a total of 3 goals in the match. 
11. In a row of 5 children A, B, C, D and E, who is standing in the middle? 
I. D is to the immediate right of E and B is to the immediate left of E. 
II. B is at the extreme left of the row. 
12. Anil, Milind, Kiran and Rajesh are four friends. How many of them are married? 
I. True Statement: Kiran and Rajesh are married 
II. False statement: At least one of Anil and Milind is married. 
13. What is the rate of interest if a sum is kept for 5 years? 
I. Compound interest earned in third and fourth year are Rs 220 and Rs 250 respectively. 
II. Total interest earned is Rs 500 calculated at SI. 
14. Members in a club either speak French or Russian or both. Find the number of members in a club who speak only French.    
I. There are 300 members in the club and the number of members who speak both French and Russian is 196.
II. B. The number of members who speak only Russian is 58.
15. What is value of 144 $ 16 * 7 # 9? 
I. $ means ÷, * means x, and # means +. 
II. 16 $ 4 * 2 # 2 = 10. 
16. What percentage of the population of a village comprises of married males? 
I. 4/7 of the population comprises of females. 
II. There are 300 unmarried males. 
17. What is the speed of the train? 
I. It takes 40 seconds to pass a bridge 1200m long. 
II. It takes 8 seconds to pass a telegraph pole. 
18. Three dices A, B and C are thrown together. What is the sum of all the readings of the dice? 
I. Dice A shows 4 more than dice C. 
II. If we multiply the readings of dice B with that of dice C, we get reading of dice A. 
www.apparteducation.com   95  info@apparteducation.com  
APPART - Academy of Proficient Professionals for Aptitude Research and Training 

19. If 2 plates and a spoon cost Rs. 4, how much does a spoon cost? 
I. 6 plates and 6 forks cost Rs. 12. 
II. A fork costs half as much as a spoon. 
20. What is the value of x in 14x433? 
I. The number 14x433 is divisible by 3. 
II. x< 5. 

MATCHING, SELECTION, ARRANGEMENT


No definite rules can be given to solve these problems. Though, some basic guidelines are given below. 
Basic Guidelines:
1.  Tackle each group of analytical reasoning questions as a unit. 
  These questions require you to think through a complicated set of conditions as you answer a group of questions. It, therefore, 
makes sense for you to treat each group of questions as a unit. Do not jump from one group to another before you answer all 
the questions of the first group. 
2.  Simplify the information by using initials and symbols. 
The  questions  are  not  intended  to  be  ambiguous  which  means  that  you  have  to  read  carefully  and  understand  the  precise 
meanings of the conditions and the questions. Rather lengthy expressions are used to specify clearly the various relationships, 
and you are likely to find these confusing. Hence, strip away individual names and irrelevant details that confuse the situation 
and substitute initials for the individual words, and if a dozen words are taken to describe a relationship, try to express it simply 
in letters and symbols as shown in the following examples. 
Wording denoting relationship Symbol
  And   X 
  Or   + 
  Equal to, Same as   =  
  Not equal to, Not same as   
  Greater than (taller, older, etc.)  > 
  Not greater than     
  Smaller than (shorter, younger, etc.)  < 
  Not smaller than   
  Now, consider the following statement: 
  Sohan is taller than Mohan which can conveniently be expressed as S > M. 
Work  with  these  symbols  if  they  seem  useful  to  you.  You  may  prefer other  symbols also  and  add  to  the  list  as  it  suits  your 
convenience OR do your own thing, if it suits you. 
3.  Highlight keywords that limit the situation critically. 
Some keywords are used in problems of this type which are meant to represent the conditions or relationships. You should pay 
particular attention to them by highlighting them. 
   
Given below are examples of commonly occurring keywords: 
www.apparteducation.com   96  info@apparteducation.com  
APPART - Academy of Proficient Professionals for Aptitude Research and Training 

    All  Always 
    At least  At most 
    May be  Cannot be 
    Each  Every 
    If, If and only if  Exactly 
    Consecutive  Immediately 
  It is to be noted that some of these words are ambiguous and admit of more than one possibility. 
  For example: The meaning of the statement ‘Raj follows Rohan’ is that Raj is behind Rohan (but nothing is  clear about the 
exact position of Raj). But when the statement changes to ‘Raj follows Rohan immediately’, then the meaning becomes very 
clear that, Raj is immediately behind Rohan. 
4.  Organize information in list, table, map or diagram form. 
  When you study a set of conditions for its implications, you  wind up with a mass of information. You need to organize this 
information. It is useful to list the basic conditions and to note down their implications in the table form. Map or diagram is 
particularly helpful when you are dealing with a problem involving the physical or temporal order of things. For example, it is 
much easier to tell whether a particular route from point A, to point B is possible or not, when you actually see points A and B 
on the map. 
5.  First, eliminate answer choices, rule out by individual conditions then work through the remaining choices or guesses. Choices 
are ruled out by making use of the conditions given. Then put together the other conditions and arrive at the correct choice 
from among the choices that are left. 
6.  Study conditions, not merely for what they state but for what they imply. 
  When you  read the  conditions you should grasp not only  what they  explicitly  state, but also what  they imply, i.e.  C  is  not 
greater than means C is either less than or equal to D. 
7.  Avoid making unwarranted assumptions. 
  Once you have set up your table or completed your diagram based on a  particular set of conditions, you  should have little 
difficulty with the group of questions based on those conditions. Hence, you should take care to avoid assumptions which are 
not  warranted  by  the  conditions  stated  in  the  problem.  For  example,  in  a  set  establishing  relationship  of  age  and  height 
among the students of a class, you should not assume that a student who is older than another student must be taller than 
that student, as this can be an incorrect inference from it. So, beware of reading too much into a condition. 
Finally, you are advised to follow the above guidelines carefully and practice more and more of these problems. Some solved 
examples have been given here, which would guide you in solving problems based on logical data interpretation. 
Solved Examples:
DIRECTIONS for questions 1 to 5: Answer the following questions based on the information given.  
  i.  There are  seven teachers  A, B, C,  D, E, F and G  in a college.  Each one of  them  teaches a different  subject except  B who 
teaches no subject. 
ii.  The subjects are Social Science, Physics, Chemistry, Math, Zoology and Commerce. 
iii.  There are three female and four male teachers, and out of these, there are two pair of couples.  
iv.  C who teaches Social Science is married to the teacher who teaches Chemistry. 
v.  E and G are female teachers who teach Zoology and Physics respectively. 
www.apparteducation.com   97  info@apparteducation.com  
APPART - Academy of Proficient Professionals for Aptitude Research and Training 

vi.  A teaches Mathematics, and his wife does not teach Physics. 
vii.  F and D are male teachers. 
viii.  F is unmarried. 
1.  Which subject does ‘F’ teach? 
  (A) Mathematics  (B) Chemistry  (C) Physics  (D) Social Sciences   (E) None of these 
2.  Which subject does ‘D’ teach? 
  (A) Physics  (B) Commerce  (C) Social Sciences   (D) Chemistry  (E) Cannot be determined 
3.  Which of the following are two pairs of couples? 
  (A) DC and AE  (B) AC and D  (C) GA and CD  (D) CD and BF  (E)Cannot be determined 
4.  Which subject does A’s wife teach?  
  (A) Chemistry  (B) Zoology  (C) Social Sciences  (D) Commerce  (E) Cannot be determined 
5.  Who among the following are the males? 
  (A) AC  (B) AE  (C) AD  (D) AG  (E) Cannot be determined 
Solution: All of the given information can be tabulated as: 
    Social  Physics  Chemistry  Math  Zoology  Commerce 
  A  x  x  X   x  x 
  B  x  x  X  x  x  x 
C   x  X  x  x  x 
 
D  x  x     x  x  x 
  E  x  x  x  x     x 
Male teachers: A, F, D, B  F  x  x  x  x  x    
Female teachers: E, G, C  G  x     x  x  x  x 

From the above table we can conclude thatbecause F is unmarried, D teaches Chemistry.  Hence, F Teaches Commerce. Now, from 
condition (iii) C is married to one who teaches chemistry. But F is unmarried which in turn implies that C is married to D. 
The correct choices are: 
  1. (E)  2.(D)  3. (A)  4. (B)  5. (C) 
CLASS ASSIGNMENT
1.  Priyanka ranked sixteenth from the top and twenty ninth from the bottom among those who passed an examination. Six girls 
did not participate in the competition and five failed in it. How many girls were there in the class? 
  (A) 40  (B) 44  (C) 50  (D) 55  (E) 58   
Directions for Questions 2 and 3: The University of Mumbai offers eight courses. A student must take up any two of the languages - 
English, French &  German, and any three of  the  subjects-  Psychology,  Philosophy, Sociology,  Geography and  History. French and 
German cannot be taken up together. German and Philosophy cannot be taken up together. Psychology and Geography cannot be 
taken up together. Psychology and History cannot be taken up together. 
2.  If German is selected, then which of the following cannot be taken up? 
  (A) Geography  (B) History  (C) Sociology  (D) Psychology  (E) English 
3.  If Psychology is taken up, then which of the following is true? 
  (i) German cannot be taken up  (ii) French cannot be taken up  (iii) Sociology must be taken up 
  (A)i only  (B) ii and iii  (C) i and iii  (D) i and ii  (E) ii only 
www.apparteducation.com   98  info@apparteducation.com  
APPART - Academy of Proficient Professionals for Aptitude Research and Training 

Directions for questions 4 and 5: A group of seven singers, facing the audience, are standing in a line on the stage as follows: 


 D is to the immediate right of C. 
 F is standing besides G. 
 B is to the immediate left of F. 
 E is to the immediate left of A. 
 C and B have one person between them. 
 A and D have one person between them. 
4.  Who is on the extreme right? 
  (A) D  (B) F  (C) G   (D) E  (E) None of these 
5.  If we start counting from the left, on which number is C? 
  (A) 1 st  (B) 2 nd  (C) 3rd  (D) 5 th  (E) None of these 
Directions for Questions 6 and 7:Answer the following questions based on the statements given below:
There are three houses on each side of the road. These six houses are labeled as P, Q, R, S, T and  U. The houses are of different 
colours,  namely,  Red,  Blue, Green,  Orange, Yellow and White.  The houses are of different heights.  T,  the  tallest house, is exactly 
opposite to the Red coloured house. The shortest house is exactly opposite to the Green coloured house. U, the Orange coloured  
house, is located between P and S. R, the Yellow coloured house, is exactly opposite to P. Q, the Green coloured house, is exactly 
opposite to U. P, the White coloured house, is taller than R, but shorter than S and Q. 
6.  What is the colour of the house diagonally opposite to the Yellow coloured house? 
    (A) White  (B) Blue  (C) Green  (D) Red  (E) none of these 
7.  Which is the second tallest house? 
    (A) P  (B) S  (C) Q  (D) R  (E) Indeterminable 
DIRECTIONS for questions 8 to 9: Read the information given below and answer the questions that follow.
A, B, C, D, E and F are seated in a circle facing the centre. 
D is between F and B. 
A is second to the left of D and second to the right of E. 
9. Who is facing A? 
    (A) B  (B) D  (C) F  (D) Either F or B 
10. Who among the following is facing D? 
    (A) A  (B) C  (C) E  (D)Cannot be determined 
DIRECTIONS for questions 11 to 14: Read the information given below and answer the questions that follow.
P, Q, R, S, T, U, V and W are eight friends. They are sitting around a round table and having coffee. 
1)   P is sitting between U and V. 
2)   Q is sitting between W and T. 
3)   R is sitting to the third left of V. 
4)   W is sitting third right of S. 
11. What is the position of V with respect to S? 
  (A)  Immediate left  (B) Immediate right  (C) Third to the left  (D) Fourth to the right   
12. How many persons are sitting between R and S? 
  (A) 2  (B) 3  (C) 4  (D) 5 
 
13. What is the position of T with respect to W? 
www.apparteducation.com   99  info@apparteducation.com  
APPART - Academy of Proficient Professionals for Aptitude Research and Training 

  (A) Immediate left  (B) Third to the left  (C) Third to the left  (D) Second to the left 


14.  Which of the following is not true? 
  (A) U is sitting immediate right of R    (B) U is sitting immediate left of P 
  (C) P is immediate right of V.    (D) S is to the second left of Q. 
DIRECTIONS for questions 15 to 17: Read the information given below and answer the questions that follow.
1)   Eight friends A, B, C, D, E, F, G and H are seated in a circle facing the centre. 
2)   D is between B and G, and F is between A and H. 
3)   E is second to the right of A. 
15. Which of the following is A’s Position? 
  (A)  Left of F  (B) Right of F  (C) Between E and F  (D)Can’t determine 
16. Which of the following information are not required to ascertain the position of C? 
  (A) 1  (B) 2  (C) 3  (D) either 1 or 2  (E) All are required 
17. Which of the following C’s position? 
  (A) Between E and F  (B) Between G and E    (C) Second to the left of B 
  (D) Cannot be determine  (E)None of these 
DIRECTIONS for questions 18 and 19: Read the information given below and answer the questions that follow.
  Four boys Rajpal, Premanand, Vedanth and Anmol are friendly with four girls Sushma, Kusum, Vimla and Poonam.Sushma and Vimla 
are  friends.  Vedanth’s  girl  friend does  not  like  Sushma  and  Vimla.  Kusum  does  not  care  for  Vedanth.  Premanand’s  girl  friend  is 
friendly with Sushma. Sushma does not like Rajpal. 
18. Who is Rajpal’s girl friend? 
    (A) Sushma  (B) Kusum  (C) Vimla  (D) Poonam 
19. Who is Poonam’s boy friend? 
    (A) Anmol  (B) Vedanth  (C) Premanand  (D) Rajpal 
DIRECTIONS for questions 20 to 22: Read the information given below and answer the questions that follow.
Jaikishan, Kamlesh, Namrata, Abhilasha and Tanmay are five members of a family. 
They have their birth dates from January to May, each member in one of these months. 
Each one likes one particular item for his/her birthday out of Bengali Sweets, Chocolates, Pastries, Ice Cream and Dry Fruits. 
The one who likes Pastries is born in the month which is exactly middle in the months given. 
Abhilasha does not like Ice cream but brings Chocolates for Jaikishan in February. 
Tanmay who is fond of Bengali sweets is born in the next month immediately after Namrata. 
Namrata does not like Dry fruits or Ice cream. 
20. What is the choice of Abhilasha? 
    (A) Pastries  (B) Dry fruits  (C) Bengali sweets  (D) Cannot be determined 
21. What is the choice of Kamlesh? 
    (A) Ice–cream  (B) Bengali sweets   (C) Dry fruits  (D) Cannot be determined 
22. In which month was Kamlesh born? 
    (A) January  (B) May  (C) January or May  (D) Data inadequate 
23. A, B, C, D and E are standing in a ring wearing hats of different colours. A person cannot see two persons standing adjacent to 
him. A sees white and green. B sees black and yellow. The person wearing white hat sees black and red. What is the colour of 
B’s hat? 

www.apparteducation.com   100  info@apparteducation.com  


APPART - Academy of Proficient Professionals for Aptitude Research and Training 

    (A) White  (B) Green  (C) Red  (D) Blue  (E) Black 


DIRECTIONS for questions 24 to 28: Read the information given below and answer the questions that follow. 
i.   P, Q, R, S, T, V and W are seven friends who left for seven different places-Delhi, Chennai, Hyderabad, Bangalore,  Kolkata, 
Chandigarh and Patna each one on a different day of a week. R left for Patna on Monday. On the last day of the week one 
person left for Bangalore. T left on the day after P, who left for Chandigarh two days prior to W’s departure. S left for Kolkata 
on Friday. Q did not leave for either Hyderabad or Bangalore and W left for Delhi. 
ii.  It is also given that the week starts on Sunday and end on Saturday. 
24. On Which day of the week did Q leave? 
    (A)  Sunday  (B) Saturday  (C) Wednesday  (D)Can’t determine 
25. Who left for Bangalore? 
    (A)  T  (B) P  (C) V  (D)Can’t determine 
26. On which day of the week did T leave? 
(A) Tuesday  (B) Thursday  (C) Sunday  (D) Wednesday 
27. Which of the following combinations of person-place is not correct? 
(A) R ---Patna  (B) P ---Chandigarh  (C) T ---Hyderabad  (D)All are correct 
28. Who left on Tuesday? 
(A) P  (B) W  (C) Q  (D) V 
DIRECTIONS for questions 29 to 33: Read the information given below and answer the questions that follow.
A  Quiz  competition  was  organized  in a  school and  the performance  of  students  was  recorded  on  a piece of  paper  with  ink.  But 
somehow some water fell on the paper and the information remained incomplete, as shown below: 
Average Good Excellent Total
Male      12   
Female        36 
Total    33     
However the scorer has some clue which is: 
I.  Half of the students were either excellent or good. 
II. 40% of the students were females. 
III. One third males student were average  
29. How many students are both female and excellent?   
  (A) 0  (B) 8  (C) 16  (D) 20  (E) cannot be determined 
30. What proportion of good students are male? 
  (A) 0.73  (B) 0  (C) 0.4  (D) 0.3  (E) None of these  
31. What proportions of female students are good? 
  (A) 0.25  (B) 0  (C) 0.5  (D) 0.7  (E) Cannot be determined 
 
32.  How many students are both male and good? 
  (A) 16  (B) 24  (C) 27  (D) 32  (E) None of these 
33.  Among Average students, what is the ratio of males to females?   

www.apparteducation.com   101  info@apparteducation.com  


APPART - Academy of Proficient Professionals for Aptitude Research and Training 

  (A) 1:3  (B) 2:3  (C) 3:2  (D) 4:3  (E)None of these   

VISUAL REASONING
Visual Reasoning is all about finding similarities or dissimilarities amongst diagrams by finding the pattern amongst them. 
The  reasoning  involves  the  ability  to  understand  and  analyse  visual  information  and  solve  problems  using  visual  reasoning.  For 
example:  identifying  relationships,  similarities  and  differences  between  shapes  and  patterns,  recognizing  visual  sequences  and 
relationships between objects, and remembering these.  
Various Question patterns can be listed under non verbal reasoning i.e. visual reasoning –  
      1. Complete the series 
      2. Find odd man out 
      3. Mirror images 
      4. Complete the image/figure 
CLASS ASSIGNMENT
DIRECTIONS for questions 1 to 10:Select  a  figure  from  amongst  the  Answer  Figures  which  will  continue  the  same  series  as 
established by the five Problem Figures. 
      Problem Figures:                            Answer Figures 

1.    
      (1)       (2)       (3)       (4)       (5)           (A)        (B)       (C)      (D)        (E)           
      Problem Figures:                            Answer Figures: 

2.    
      (1)       (2)       (3)       (4)       (5)           (A)        (B)       (C)      (D)        (E)           
      Problem Figures:                            Answer Figures: 

3.   
      (1)       (2)       (3)       (4)       (5)           (A)        (B)       (C)      (D)        (E)           
      Problem Figures:                            Answer Figures: 

4.   
      (1)       (2)       (3)       (4)       (5)           (A)        (B)       (C)      (D)        (E)           
      Problem Figures:                            Answer Figures: 

5.   
      (1)       (2)       (3)       (4)       (5)           (A)        (B)       (C)      (D)        (E)           

      Problem Figures:                            Answer Figures: 

www.apparteducation.com   102  info@apparteducation.com  


APPART - Academy of Proficient Professionals for Aptitude Research and Training 

6.   

      (1)       (2)       (3)       (4)       (5)           (A)        (B)       (C)      (D)        (E)           


      Problem Figures:                            Answer Figures: 

7.   
      (1)       (2)       (3)       (4)       (5)           (A)        (B)       (C)      (D)        (E)           
      Problem Figures:                            Answer Figures: 

8.   
      (1)       (2)       (3)       (4)       (5)           (A)        (B)       (C)      (D)        (E)           
DIRECTIONS for Questions 9 to 12: Select a suitable figure from the Answer Figures that would replace the question mark (?). 
      Problem Figures:                              Answer Figures: 

9.   
        (1)        (2)        (3)          (4)                     (A)       (B)        (C)        (D)     (E) 
      Problem Figures:                              Answer Figures: 

10.   
        (1)        (2)        (3)          (4)                     (A)        (B)       (C)        (D)    (E) 
      Problem Figures:                             Answer Figures: 

11.   
        (1)        (2)        (3)          (4)                     (A)        (B)        (C)        (D)     (E) 
   
      Problem Figures:                              Answer Figures: 

12.   
  (1)        (2)        (3)          (4)                     (A)        (B)        (C)        (D)        (E) 
Directions for Questions 13 and 14: Choose the alternative which is closely resembles the mirror image of the given combination. 

www.apparteducation.com   103  info@apparteducation.com  


APPART - Academy of Proficient Professionals for Aptitude Research and Training 

13. 

   
14.  

   
DIRECTIONS for questions 15 and 17: Identify the figure that completes the pattern. 

15.   
        (X)                 (A)                (B)                  (C)                 (D) 

16.   
        (X)                  (A)                (B)                  (C)                 (D) 

17.   

        (X)                (A)                (B)                  (C)                 (D) 

CLOCKS
Clock is system consisting of 12 equal divisions of a  circle,  known as “hours”. Each hour is further divided in 60 minutes and each 
minute is divided in 60 seconds. Hence, in an hour, number of seconds are 60 * 60 = 3600. 
Questions based on clock in aptitude exams can be easily solved if we can relate them to the concept of relative speed of two people 
running along a circular track. 
Before we discuss questions, let’s understand some concepts about clock –  
 Minute space (MS): The smallest division in clock is known as minute space. To make it simple, let’s remember that 1 min = 1 
MS.  

360
 As the total central angle of a circle is 3600, angle between two successive minute spaces =  6 0 . 
60
 So, we can say that, in a minute, minute hand covers 60 and in an hour, hour hand covers 300 
 So, in an hour, distance covered by minute hand is 60 MS and that by an hour hand is 5 MS. So, we can say that minute hand 
takes a ‘lead’ of 55MS over hour hand in an hour. Clearly, ratio of speed of minute hand and hour hand is 12 : 1. 

www.apparteducation.com   104  info@apparteducation.com  


APPART - Academy of Proficient Professionals for Aptitude Research and Training 

Hands of the clock being together: 


At 12 o’clock, hands of the clock are together. As per the concept discussed above,  next time again they will be together is the time 
when minute hand takes a ‘lead’ of 60 MS over the hour hand. 

60 60 5
Hence, after  every  minutes, i.e.  65 minutes both hands of a  true clock  will  be together.  If  they coincide in  less  than 
55 11
5 5
65 minutes, then the clock is gaining time (running faster) and if they coincide in more than  65 minutes, then the clock  is 
11 11
losing time (running slow). 
Angle between hands of a clock at a particular time (say, at H:M o’clock):
Angle between hands of a clock  is the difference the angular distance covered from the hands position at 12 o’clock by both the 
hands.  

10
In 1 min, minute hand travels 6 0 and hour hand travels   
2
M0
So, in M minutes, Minute hand travels 6M0 and hour hand travels . 
2
Also, in H hours, hour hand has already travelled 30H 0 
Hence, When the clock shows H:M o’clock – 
 Minute hand has travelled 6M0.  

M 0
 Hour hand has travelled (30H +   ) . 
2
So, the difference between these displacements will give us the angle between hands of the clock at H:M o’clock. 

M 11M
Angle between hands of a clock at H:M o’clock = 30H +   - 6M = 30H –   
2 2
Sample example:
1.   Find the angle between hands of a clock at 4:12 o’clock 

11
Angle = 30*4 –  *12 = 120 – 66 = 540 
2
2.   Find the angle between hands of a clock at 4:30 o’clock 

11
Angle = 30*4 –  *30 = 120 – 165 = – 450 
2
Sign is negative because minute hand has covered more distance than that covered by hour hand. So, only magnitude of the angle 
should be considered. 
Some important trivia:
In a day both the hands of clock coincide 22 times. 
In a day both the hands of clock make a right angle 44 times. 
In a day they will be opposite 22 times. 

www.apparteducation.com   105  info@apparteducation.com  


APPART - Academy of Proficient Professionals for Aptitude Research and Training 

ASSIGNMENT
1.  What is the angle between the hands of a clock at 10.48 P.M.?  
  (a) 42  (b)35  (c) 36  (d) 41 
2.  What is the angle between the hands of a clock at 3.24 A.M? 
  (a) 31  (b) 66  (c) 12  (d) None of these 
3.  At what time between 4 and 5 o’clock are the hands of the clock together? 

7 9 9
  (a) 4:21  (b) 4:24   (c) 4:31   (d) 4:21  
11 11 11
4.  The minute hand of a clock overtakes the hour hand at intervals of 65 minutes of correct time. How much does the clock gain 
or lose per day? 
  (a) 10 minutes  (b) 1010/11 minutes  (c) 11 minutes  (d) 1010/143 minutes 
5.  A clock is set right at 8 A.M. The clock  gains 10 minutes in 24 hours. What will be the true time when the clock indicates 1 
P.M. the following day?  
  (a) 12:48 PM  (b) 12:47 11/12 PM (c) 12:47 PM  (d) None of these 

CALENDARS
Only two types of questions come on the topic of calendar in aptitude exams. 
1. Find the day on a particular date, without any reference day.  
2. Find the day on a particular date, given a date and day.  
The basic concept is as follows:  
From a reference day and date, calculate the number of days passed till the required date. As any day repeats after a cycle of 7 days, 
divide  the  number  of  days  passed  by  7  and  take  the  remainder.  This  remainder  is  termed  as  “extra days”.  Depending  on  the 
remainder, find the day on a given date.  
More about extra days:
Here, we will explore the concept of extra days for a month, an year and finally for a century. We will use all the notations 
defined here in the subsequent problem solving. 
 The day on 01-01-0001 is always taken as Monday, and is used as a reference day, if not otherwise given in a question. 
 As already discussed above, ‘extra days’ is the remainder when number of days is divided by 7. Hence, we have number of 
extra days for each month as follows –  
Month Number of days Extra days
January  31  3 
February  28/29  0/1 
March  31  3 
April  30  2 
May  31  3 
June  30  2 
July  31  3 
August  31  3 
September  30  2 
October  31  3 
November  30  2 
December  31  3 
 

www.apparteducation.com   106  info@apparteducation.com  


APPART - Academy of Proficient Professionals for Aptitude Research and Training 

 Normal Year (NY):An year consisting of 365 days. Hence, number of extra days in NY is 1. 


 Leap Year (LY): If the year is divisible by 4, then it is called a leap year. It has 366 days and hence, number of extra days in LY is 
2. However, the criterion, that the year should be divisible by 4, is not true in case of century year. Let us define century year 
and understand the exception. 
 Century Year (CY): The year having last two digits as 00. For example, 1500, 1700 etc. 
 Century year which is Leap Year: If CY is divisible by 400 (and not by 4), then only it is a leap year. For example, 1600, 2000 
are leap years and 1500, 1700 etc are not. 
 Normal Century (NC): A century in which CY is not a leap year. For example, 1401-1500, 1801-1900 are examples of a NC. 
 Leap Century (LC): A century in which CY is a leap year. For example, 1501-1600, 1901-2000 are examples of a LC. 
 Now let us calculate number of extra days for a normal and a leap century. 
  Normal Century Leap Century
  NY LY NY LY
Number of years 76  24  75  25 
extra days per year 76  48  75  50 
Total extra days 76 + 48 = 124  75 + 50 = 125 
Effective extra days 5  6 
 

 Now let us calculate number of extra days for first four centuries – (why first four? – keep reading‼) 
Century Number of extra days
001-100 5 
101-200 5 
201-300 5 
301-400 6 
21 
Effective extra days 0 
 

Number of extra days is zero means, last day of the last year is Sunday (remember 01-01-0001 was Monday). It means, after every 
four centuries, calendar repeats itself. Hence, we can ignore set of four centuries appropriately and proceed further. 
Following table gives the relation between the number of effective extra days and day – 
 (A gentle remainder again – 01-01-0001 is Monday)  Extra days Day
1  Monday 
2  Tuesday 
3  Wednesday 
4  Thursday 
5  Friday 
6  Saturday 
0  Sunday 

Sample question:Q. Find the day on 15th August 1947. 
Solution: We will find the total number of odd days till 15th Aug 1947 from the reference day i.e. from 01-01-0001. 
Month/year Extra days Effective extra days
0001-1600  0  0 
1601-1700  5  1 
1701-1800  5 

www.apparteducation.com   107  info@apparteducation.com  


APPART - Academy of Proficient Professionals for Aptitude Research and Training 

1801-1900  5 
1901-1946  57(35 NY and 11 LY)  1 
Jan 1947  3  16 
Feb 1947  0 
Mar 1947  3 
Apr 1947  2 
May 1947  3 
Jun 1947  2 
Jul 1947  3 
Aug 1947 (15 days)  1  1 
  5
Day  Friday
ASSIGNMENT
1. Find the day on the present date if 01-01-0001 was Monday. 
2. Find the day on your birth date assuming 01-01-0001 was Monday. 
3. What was the day on 26th January 1950? 
4. If 1st Jan 1930 was Sunday, find the day on 31 st March 1970. 
5. If 1st Jan 1975 is Sunday, find the day on 31 st May 1950.

INPUT & OUTPUT FLOW CHART


A pattern of random words and numerals, i.e. of alphanumeric form, is given as input. Certain rules are followed for rearranging this 
message to produce sequential outputs. You are therefore required to figure out the pattern followed for rearrangement and then 
answer the given questions. 
Though questions on input-output flowchart come very rarely in campus placements tests, they appear frequently in other aptitude 
exams like Bank Entrance, MBA Entrance etc. 
There are mainly two types of flow charts –  
1.   Numbers arrangement – Under this also, two sub-types are seen – a) numbers shuffled horizontally and b) numbers undergo 
a series of changes vertically. 
2.     Words  arrangement  or  Alphabets  arrangement  –  In  this,  changes  are  in  terms  of  shuffling  of  the  alphabets/words 
horizontally. 
Method to attempt these questions –
1. Observe input and final output carefully. This will tell you whether the words/alphabets/numbers have been shuffled horizontally 
or vertically. 
2. In the output, find the order in which the original set of input is arranged. 
3. Observe the intermediate steps to trace the pattern of changes in the input during each step. 

www.apparteducation.com   108  info@apparteducation.com  


APPART - Academy of Proficient Professionals for Aptitude Research and Training 

CLASS ASSIGNMENT
DIRECTIONS for the questions 1 and 2:Read the information given below and answer the questions that follow: 
A  “number-rearranger”  machine  works  as  follows:  It  takes  the  input  line  and  rearranges  the  numbers  as  per  certain  coded 
instructions. You have to decode the instructions by using the sample given below. 
Input 49  31  17  30  18  97  25  80 
Steps Outcome
I  17  49  31  30  18  97  25  80 
II  17  31  49  30  18  97  25  80 
III  17  31  97  49  30  18  25  80 
IV  17  31  97  25  49  30  18  80 
V  17  31  97  25  49  18  30  80 
 

By using the decoded instructions, solve the following questions. 
1. Which of the following would be the outcome at the second step? 
Input: 13 20 27 43 65 29 
(1) 13 29 43 20 27 65   (2) 13 27 29 43 20 65    (3) 13 29 20 43 27 65    (4) 13 29 27 43 20 65 
2.   If the outcome at the second step is “11 31 71 51 40 28 91”, then what would be the outcome at the end of the fourth step? 
  (1) 11 31 71 51 28 40 91   (2) 11 31 71 51 91 40 28    (3) 11 31 71 51 91 28 40    (4) 11 31 71 51 40 91 28 
DIRECTIONS for questions 3 and 4: Read the information given below and answer the questions that follow: 
A “word-rearranger” machine works as  follows: It takes the input line and rearranges the words as per certain coded instructions. 
You have to decode the instructions by using the sample as given below. 
 

  Steps Outcome
  I:  all on it in if give for of 
  II:  all for on it in if give of 
  III:  all for give on it in if of 
  IV:  all for give if on it in of 
  V:  all for give if in on it of 
  VI:  all for give if in it on of 
 
 

Input: on it in if give all for of  VII:  all for give if in it of on 

By using the decoded instructions, solve the following questions. 
3. Input: “mat not on an for cot”. Which of the following steps would match with “an cot for mat not on”? 
(1) I    (2) II    (3) III    (4) IV 
4. Input: “apt act all ace add”. Which of the following would be the outcome at the fourth step? 
(1) ace act all add apt    (2) act ace add all apt    (3) ace act apt all add    (4) ace act add all apt 
5.   Input: “month women child pay man year”. Which of the following steps would match with: “child man month women pay 
year”? 
(1) I    (2) II    (3) III    (4) IV 

www.apparteducation.com   109  info@apparteducation.com  


APPART - Academy of Proficient Professionals for Aptitude Research and Training 

DIRECTIONS for questions 6 to 10: Read the information given below and answer the questions that follow: 


A  number  arrangement  machine,  when  given  a  particular  input,  rearranges  it  following  a  particular  rule.  The  following  is  the 
illustration of the input and the steps of arrangement. 
Input 17  19  23  7  32  26  13 
I 24  26  30  14  39  33  20 
II 196  256  400  16  841  529  100 
III 55  61  73  25  100  82  43 
IV 289  361  529  49  1024  676  169 
V 256  324  484  36  961  625  144 
VI 26  28  32  16  41  35  22 
6. The third step of a given input is 52, 58, 70, 19, 103, 85, 49. What will be step IV of the input? 
(1) 289, 361, 529, 25, 1027, 784, 196    (2) 256, 324, 484, 25, 1089, 729, 225 
(3) 256, 324, 529, 25, 1027, 787, 196    (4) 289, 361, 529, 25, 1089, 729, 225 
7.  What would be step III for the following input? 11, 12, 16,21,31,24, 18 
(1) 36, 39, 51, 65, 95, 74, 56      (2) 36, 39, 51, 66, 96, 75, 57 
(3) 37, 40, 52, 67, 97, 76, 58      (4) 38, 41, 53, 69, 98, 77, 59 
8.  Step V of a given input is 25, 81, 100, 324, 441, 729, 256. What will be the input? 
(1) 6, 10, 12, 19, 22, 28, 17    (2) 5, 9, 11, 18, 21, 27, 16    (3) 4, 8, 10, 17, 20, 26, 15    (4) None of these 
9.  In how many steps would the following arrangement be yielded by the given input? 
Input: 6, 10, 12, 19, 22, 28, 17 
Arrangement: 22, 34, 40, 61, 70, 88, 55 
(1) 2    (2) 5    (3) 4    (4) 3 
10. What will be the sixth step of the following input? 
Input: 5, 9, 11, 18, 21, 27, 16 
(1) 13, 17, 20, 26, 27, 34, 25  (2) 14, 18, 20, 27, 30, 36, 25  (3) 15, 19, 21, 26, 29, 36, 25  (4) 14, 18, 20, 27, 30, 36, 24 

PUZZLES 
1.  Four camels traveling on a very narrow ledge encounter four camels coming the other way. As everyone knows, camels never 
go backwards, especially when on a precarious ledge. The camels will climb over each other, but only if there is a camel sized 
space on the other side. The camels didn't see each other until there was only exactly one camel's width between the two 
groups. How can all camels pass, allowing both groups to go on their way, without any camel reversing? 
2.  You are trapped in a room with two doors. One leads to certain death and the other leads to freedom. You don't know which 
is which. There are two robots guarding the doors. They will let you choose one door but upon doing so you must go through 
it. You can, however, ask one robot one question. The problem is one robot always tells the truth, the other always lies and 
you don't know which is which. What is the question you ask? 
3.  5 pirates of different ages have a treasure of 100 gold coins.  On their ship, they decide to split the coins using this scheme:  
The oldest pirate proposes how to share the coins, and ALL pirates (including the oldest) vote for or against it. If 50% or more 
of the pirates vote for it, then the coins will be shared that way. Otherwise, the pirate proposing the scheme will be thrown 
overboard, and the process is  repeated with the pirates that remain. As pirates tend to be a bloodthirsty bunch, if a pirate 
would get the same number of coins if he voted for or against a proposal, he will vote against so that the pirate who proposed 
the plan will be thrown overboard. Assuming that all 5 pirates are intelligent, rational, greedy, and do not wish to die, (and are 
rather good at math for pirates) what will happen? 
4.  At a family reunion were the following people: one grandfather, one grandmother, two fathers, two mothers, four children, 
three  grandchildren,  one  brother,  two  sisters,  two  sons,  two  daughters,  one  father-in-law,  one  mother-in-law,  and  one 
daughter-in-law. But not as many people attended as it sounds. How many were there, and who were they? 

www.apparteducation.com   110  info@apparteducation.com  


APPART - Academy of Proficient Professionals for Aptitude Research and Training 

5.  You have a three gallon and a five gallon measuring device. You wish to measure out four gallons. How do you do this? 
6. You want to send a valuable object to a friend securely. You have a box which can be fitted with multiple locks, and you have 
several locks and their corresponding keys. However, your friend does not have any keys to your locks, and if you send a key 
in an unlocked box, the key could be copied en route. How can you send the object securely? 
 
7. A corporate businessman has two cubes on his office desk. Every day he arranges both cubes so that the front faces show the 
current day of the month.  What numbers are on the faces of the cubes to allow this?  
(Note: You can't represent the day "7" with a single cube with a side that says 7 on it. You have to use both cubes all the time. 
So the 7th day would be "07".) 
8. All of my flowers except two are roses. All of my flowers except two are tulips. All of my flowers except two are daisies. How 
many flowers do I have? 
9. A man is caught on the King's property. He is brought before the King to be punished. The King says, "You must give me a 
statement. If it is true, you will killed by lions. If it is false, you will be killed by trampling of wild buffalo." But in the end, the 
King has to let the man go. What was the man's statement? 
10. A bridge will collapse in 17 minutes. 4 people want to cross it before it will collapse. It is a dark night and there is only one 
torch between them. Only two people can cross at a time. "A" takes a minute to cross. "B" takes 2 minutes. "C" takes 5and 
"D" takes 10 minutes. How do they all cross before the bridge collapses? 
11. A high school has a strange principal. On the first day, he has his students perform an odd opening day ceremony: There are 
one thousand lockers and one thousand students in the school. The principal asks the first student to go to every locker and 
open it. Then he has the second student go to every second locker and close it. The third goes to every third locker and, if it is 
closed, he opens it, and if it  is open, he closes it. The fourth student does this to every  fourth locker, and so  on. After the 
process is completed with the thousandth student, how many lockers are open? 
12. You have two strings whose only known property is that when you light one end of either string it takes exactly one hour to 
burn.  The  rate  at  which  the  strings  will  burn  is  completely  random  and  each  string  is  different.  How  do  you  measure  45 
minutes?  
13. I have ten boxes which I want to pack into crates. Each crate can carry a maximum of 25   kg.  But  I  only  have  three  crates, 
and the total weight of the boxes is 75kg: 15 kg, 13kg, 11 kg, 10 kg, 9 kg, 8 kg, 4 kg, 2 kg, 2kg, 1 kg. How can I pack the boxes 
into the crates? 
14. You must cut a birthday cake  into  exactly eight pieces,  but you're only allowed  to make three  straight  cuts, and you can't 
move pieces of the cake as you cut. How can you do it? 

www.apparteducation.com   111  info@apparteducation.com  


APPART - Academy of Proficient Professionals for Aptitude Research and Training 

ANSWER KEY
 
CODING DECODING CUBES
 
1  D  6  B  11  C  1  3  9  1  17  1 
 
2  B  7  D  12  C  2  4  10  3  18  3 
 
3  D  8  E  13  C  3  3  11  3  19  3 
 
4  B  9  D  14  A  4  2  12  2  20  1 
 
5  B  10  C  15  D  5  2  13  3  21  1 
 
6  4  14  4  22  3 
   
NUMBER SERIES 7  3  15  1  23  3 
 
1  A  6  C  11  C  8  2  16  5  24  4 
 
2  B  7  B  12  A  25  1 
   
3  C  8  D  13  E 
   
4  D  9  B  14  B  SET THEORY
 
5  D  10  C  15  D  1  A  9  B  17  C 
 
2  D  10  D  18  C 
   
ALPHABETIC SERIES 3  B  11  C  19  A 
 
1  B  6  D  11  C  4  C  12  D  20  B 
 
2  D  7  A  12  C  5  B  13  A  21  C 
 
3  C  8  B  13  B  6  D  14  C  22  D 
 
4  D  9  D  14  C  7  D  15  B  23  B 
 
5  B  10  A  15  B  8  C  16  A 
   
     
BLOOD RELATION SYLLOGISM
 
1  B  6  B  11  C  1  D  8  C  15  D 
 
2  D  7  A  12  A  2  E  9  D  16  B 
 
3  D  8  D  13  C  3  C  10  D  17  C 
 
4  B  9  D  14  B  4  E  11  C  18  B 
 
5  C  10  D  5  C  12  A  19  B 
   
6  B  13  D  20  B 
   
DIRECTIONS 7  B  14  A 
   
1  C  4  B  7  D 
   
2  B  5  A  8  D  DATA INTERPRETATION
 
3  A  6  C  9  C  1  D  11  C  21  A 
 
10  A  2  B  12  A  22  B 
   
3  A  13  B  23  C 
   
4  C  14  B  24  D 
             
5  A  15  A  25  C 
             
6  C  16  D  26  C 
             
7  B  17  B  27  D 
             
8  C  18  D  28  D 
             
9  C  19  A  29  B 
             
10  C  20  B  30  A 
             
www.apparteducation.com   112  info@apparteducation.com  
APPART - Academy of Proficient Professionals for Aptitude Research and Training

DATA SUFFICIENCY VISUAL REASONING


1  B  8  D  15  A  1  C  7  D  13  B 
2  C  9  A  16  D  2  B  8  D  14  B 
3  D  10  D  17  E  3  D  9  A  15  D 
4  E  11  E  18  E  4  A  10  B  16  D 
5  D  12  E  19  D  5  D  11  C  17  D 
6  E  13  A  20  D  6  C  12  E 
 
7  E  14  E 
   
CLOCKS
 
MATCHING, SELECTION, ARRANGEMENT 1  C  3  D  5  A 

1  D  12  D  23  A  2  4  D 
(420)     
2  D  13  C  24  C   
 
3  C  14  B  25  D  CALENDERS
4  C  15  E  26  D  1  ---  3  THURSDAY  5  SATURDAY 
5  C  16  E  27  A  2  ---  4  SATURDAY 
   
6  D  17  B  28  A 
 
7  E  18  B  29  A  INPUT & OUTPUT FLOW CHART
8  D  19  B  30  A  1  1  5  2  9  4 
9  B  20  A  31  B  2  3  6  2  10  2 
10  A  21  C  32  B  3  3  7  3 
11  B  22  B  4  4  8  4   
 
             
PUZZLES  
Hints for Puzzles
 

1.     
2.   Ask one robot what the other robot would say, if it 
   was asked which door was safe. Then go through the 
other door. 
   3.   The oldest pirate will propose a 98 : 0 : 1 : 0 : 1 split, 
in other words the oldest pirate gets 98 coins, the 
   middle pirate gets 1 coin and the youngest gets 1 
coin. 
Let us name the pirates (from oldest to youngest): A, 
   B, C, D and E 
Workingbackwards: 
   If there are 2 Pirates: D splits the coins 100 : 0 (giving 
himself all the gold). His vote (50%) is enough to 
ensure the deal. 
   If there are 3 Pirates: C splits the coins 99 : 0 : 1. E 
will accept this deal (getting just 1 coin), because he 
knows that if he rejects the deal there will be only 
  
two pirates left, and he gets nothing. 
If there are 4 Pirates: B splits the coins 99 : 0 : 1 : 0. 
   By the same reasoning as before, D will support this 
deal. B would not waste a spare coin on C, because C 
knows that if he rejects the proposal, he will pocket 

www.apparteducation.com 113 info@apparteducation.com


APPART - Academy of Proficient Professionals for Aptitude Research and Training

99 coins once B is thrown overboard. B would also  8.  Try to keep number of flowers as minimum as 


not give a coin to E, because E knows that if he  possible. Again, this puzzle can be   solved using a 
rejects the proposal, he will receive a coin from C in  logic which will give two different answers. 
the next round anyway. 
9.  If king is not able to determine whether the 
If there are 5 Pirates: A splits the coins 98 : 0 : 1 : 0 : 
statement is true or false, he may leave the man. 
1. By offering a gold coin to C (who would otherwise 
Answer: I will be killed by trampling of wild buffalo. 
get nothing) he is assured of a deal. 
(Note: In the final deal A would not give a coin to B,  10.  Either A or B should carry the torch back to the 
who knows he can pocket 99 coins if he votes  earlier end of the bridge. 
against A's proposal and A goes overboard. Likewise, 
11.  The only lockers that remain open are perfect 
A would not give a coin to D, because D knows that if 
squares (1, 4, 9, 16, etc) because they are the only 
he votes against the proposal, A will be voted 
numbers divisible by an odd number of whole 
overboard and B will propose to offer D the same 
numbers; every factor other than the number's 
single coin as A. All else equal, D would rather see A 
square root is paired up with another. So the answer 
go overboard and collect his one coin from B.) 
is thirty one. 
4.  There were two little girls and a boy, their parents, 
12.  Light both the ends of the first string and one end of 
and their father's parents, totaling seven people. 
the second string. 30 minutes will have passed when 
5.  Fill the five gallon container. Pour all but two gallons  the first string is fully burned, which means 30 
into the three gallon container. Empty the three  minutes have burned off the second string. Light the 
gallon container. Put the two remaining gallons from  end of the second string and when it is fully burned, 
the five gallon container into the three gallon  45 minutes will have passed. 
container. Fill the five gallon container one more 
1. Ten possible solutions: 
time. Pour one gallon from the five gallon 
{Crate 1}, {Crate 2}, {Crate 3}  
containerby filling the three gallon container. Now 
{15,10}, {13,8,4}, {11,9,2,2,1}  
the five gallon container contains four gallons. 
{15,10}, {13,11,1}, {9,8,4,2,2}  
6.    Read these sentences carefully:   {15,10}, {11,8,4,2}, {13,9,2,1}  
{15,10}, {11,9,4,1}, {13,8,2,2}  
You have a box which can be fitted with multiple 
{11,10,4}, {15,8,2}, {13,9,2,1}  
locks…Your friend does not have any keys to your 
{11,10,4}, {15,9,1}, {13,8,2,2}  
locks. 
{13,8,4}, {15,9,1}, {11,10,2,2}  
7.  The 6 doubles as a 9 when turned the other way  {13,10,2}, {15,8,2}, {11,9,4,1}  
around. There is no day 00, but you still need the 0  {13,10,2}, {15,9,1}, {11,8,4,2}  
on both cubes in order to make all the numbers  {13,11,1}, {15,8,2}, {10,9,4,2} 
between 01 and 09. There are multiple solutions for  An horizontal or vertical cut doubles the existing number of 
this puzzle.  regions. 
 

www.apparteducation.com 114 info@apparteducation.com

Das könnte Ihnen auch gefallen